.
. centre de ressources dilemmes et doutes le visage humain de mathématiques Qui sommes-nous Problème de mois activités de promotion babillard
Centrale des maths - centraledesmaths.uregina.ca
Dilemmes & doutes
« D & D »
. .
topic card  

Sujet:

ul

liste de
sujets
. .
nouvelle recherche

1464 articles trouvés pour ce sujet.
 
Page
1/1
calcul de durée 2012-09-03
laila pose la question :
un bus démarre de paris à 8h 45mn s'il fait 42h 20 mn de trajet entre paris et la ville suivante à quelle heure il arrivera à la ville suivante. est ce qu'il ya un shema pour calculer ou comment procéder. je sais qu'il faut additionner 8h 45 mn à 42h 20mn, mais comment on trouve le résultat rapidement. merci
Claude Tardif lui répond.
Les dérivées partielles 2012-05-16
Haddouche pose la question :
Bonjour,
Je vous demande de bien vouloir m'éclaircir comment on calcule les dérivées partielles généralement étudiées dans les modèles de la concurrence fiscale à titre d'exemple celui de Wildasin ""NASH EQUILIBRIA IN MODELS OF TAX COMPETITION "", dont j'ai essayé toutes les méthodes et aucune d'elle n'est arrivée à trouver le résultat des dérivées de ce dérnier papier, à cet effet, je vous serais reconnaissant de m'éclaicir la méthode afin de résoudre ce problème qui pour moi un vrai mistère à percer. Je vous remercie infiniment à l'avance. Et comptant sur votre compréhension, mes salutations les meilleures. HA

Claude Tardif lui répond.
Les calcules avec les heures 2011-09-09
johanna pose la question :
bonjour je voudrais pouvoir expliquer a mon fils ces calcules avec les heures, mais franchement je n'y comprend rien, pouvez vous m'aider.
voilà le premier calcul:
1h35min29s - 46min37s

deuxiéme exercice:
calculer la durée d'une emission télévisé qui débute à 20h40min et se termine à 22h17min.
troisiéme exercie:
je suis partie en vacances le 25 juillet. Mes vacances ont durées 2 semaines. Quel jours suis-je revenu?
quatroéme exercice:
je pars de st-pierre, je prend le bus. je veut arrivé à st-denis à 10h et demi. je sais que la durée du trajet st-pierre /st-denis est de 1h50min. a quel heure dois-je partir de st-pierre.

Claude Tardif lui répond.
Le volume d'une boule 2009-10-22
ve.lu.ch pose la question :
Pour calculer le volume d'une boule, il faut utiliser la formule :

4/3 x Pi x R au cube

Pourquoi 4/3 ?

Merci d'avance

Pierre-Louis Gagnon et Claude Tardif lui répond.
Preuve du volume d'une boule 2009-04-06
Geneviève pose la question :
J'aimerais avoir l'explication du volume de la boule, qui est (4pi R au cube) divisé par 3. J'aimerais savoir le pourquoi de l'équation, la preuve de celle-ci. Merci!
Pierre-Louis Gagnon lui répond.
Une multiplication 2009-04-03
josee pose la question :
24/6x5+40-10=

le x est une multiplication et il n'y as pas de parenthèses.

Merci Josée

Antoine Letarte and Claude Tardif lui répond.
Calculs de minutes en heures 2009-02-23
Denis pose la question :
Je suis en train de suivre un cours en navigation maritime et je dois changer souvent des minutes en heures. ex: 495 minutes = ?h??

Je désire avoir le cheminement le plus simple a faire pour ce type de calcul. Je ne travail pas avec excell. Je veux une formule a faire seulement avec une calculatrice élémentaire. Merci pour votre aide, j'apprécie beaucoup votre coup de main. Denis.

Claude Tardif lui répond.
Un volume d'une capacité minimal 2009-01-05
Pauline pose la question :
je doit faire un volume d'une capacité minimal de 2500 cm 3 en quell matiere que ce soit
* un prisme a base pentagonale non réguliere
* une sphere
* stella octangula

Pierre-Louis Gagnon lui répond.
Un tetraèdre régulier 2008-05-05
delabroye pose la question :
La figure ci-contre est un tetraèdre régulier :
AB = AC = BC = AD = 6cm

1) Représente un patron de ce tetraèdre
2) Calcule le volume de ce tétraèdre

Thierry Moisan lui répond.
Arithmétique modulaire 2008-01-10
hausknecht pose la question :
ma grand mère a entre 60 et 80 ans.il y a un an son âge était un multiple de 3.dans deux ans son âge sera un multiple de 2,de 3 et de 9.cette année , c'est un multiple de 5.quel est l'âge de ma grand mère?
Claude Tardif lui répond.
800 poules pondent 800 oeufs en 8 jours 2007-12-17
LINDA pose la question :
800 POULES EN 8 JOURS PONDENT 800 OEUFS, COMBIEN 400 POULES VONT-ELLES PONDENT D'OEUFS EN 4 JOURS?
Claude Tardif lui répond.
F=4a-3c+6b 2007-11-22
olive pose la question :
comment calcule ton ?
F=4a-3c+6b
mon fils et en 4ème et je suis dépassée, je ne peut pas l'aider merci de le faire à ma place.

Claude Tardif lui répond.
pente nulle 2007-08-31
stephanie pose la question :
si la pente dune droite est nulle alors cette droite est quoi ???
Claude Tardif lui répond.
boulier français 2007-01-19
Stéphanie pose la question :
Je prépare une exposition sur le centième anniversaire du vieux couvent de Château-Richer dans la province du Québec qui s'ouvrira en mai 2007. Dans ce couvent les soeurs de la Congréagation du Perpétuel Secours ont enseigné pendant de nombreuses années. Pour mettre en valeur, le site et l'exposition, nous souhaitons mettre les visiteurs à l'épreuve dans l'ambiance d'une classe d'antan (début 20ème siècle) avec, entre autres, les bouliers français que nous possédons et leur faire faire des exercices de mathématiques avec. Malheureusement, nous ne savons pas comment faire les quatre opérations de base (addition, soustraction, multiplication et division) avec. Pouvez-vous nous l'expliquer de façon détaillé, s'il vous plaît?
Claude Tardif lui répond.
Ordre des opérations 2006-05-15
Chantal pose la question :
2324/4-153+619= je sais pas comment calculer merci
Claude Tardif lui répond.
problème de trigonométrie 2005-11-01
Manon pose la question :
Soit un triangle ABC, de cotes a , b et c, avec A=45° et B=60°.
Apres avoir demontre que 3a²-2b²=0 (que jai fais en utilisant la regle du sinus) et a²-2c²+2ac=0 (que jai fait en utilisant la regle du cosinus), vous montrerez que laire du triangle ABC peut sexprimer: { [ 3 + racine carre 3 ] / 12 } b² unites.

Je ne sais pas comment arriver a cette expression de laire. Je narrive pas a exprimer c² et ac en fonction de b².. et je pense que ca pourrait vraiment aider..
puisque je suis arrive a letape:
ac[(racine carre 3) / 4]
avec ac = a² + c² - b²
et a² = 2/3 b²
ms c²= 1/3 b² + ac

Claude Tardif lui répond.
6 poules pondent 6 oeufs en 6 jours 2005-03-01
Nathalie pose la question :
SACHANT QUE 6 POULES PONDENT 6 OEUFS EN 6 JOURS, COMBIEN 12 POULES PONDENT D'OEUFS EN 12 JOURS?
Claude Tardif lui répond.
le système RSA 2005-02-19
Un eleve pose la question :
Je suis actuellement en classe préparatoire MP au lycée Henri Poincaré de Nancy et je souhaiterais obtenir des informations sur l'exponentiation modulaire car je réalise un TIPE sur la cryptographie et plus particulièrement le système RSA. pourriez-vous m'indiquez en quoi consiste l'exponentiation modulaire et comment cela fonctionne.
Claude Tardif lui répond.
il y a 1/6 de voitures bleues, 2005-01-24
Facrice pose la question :
il y a 1/6 de voitures bleues, 2/9 de voitures rouges, 5/12 de voitures blanches et 1/8 de voitures vertes. Sachant que 15 voitures sont d'une autre couleur. Combien y a t il de voitures de chaque couleur.
Claude Tardif lui répond.
Hauteur d'un tétraèdre irrégulier 2003-08-29
Thibault pose la question :
Je cherche à connaître comment calculer la hauteur d'un tétraèdre irrégulier (aucune arêtes égales) en ne connaissant que la longueur des arêtes.
Claude Tardif lui répond.
le matricule 2003-01-12
Sirkka pose la question :
"Dans un pays totalitaire, tous les habitants ont un matricule composé de 5 chiffres. Les policiers se reconnaissent au fait que le premier chiffre de leur matricule est strictement supérieur à la somme des quatre autres. ex: 90124 9>0+1+2+4=7 Combien y a-t-il (au plus) de policiers?"
Claude Tardif lui répond.
Boulier 2001-02-13
Un élève pose la question :
Comment se sert-on d'un boulier (non asiatique) comportant 10 rangées de 10 boules chacune?
Claude Tardif lui répond.
isomorphisme 2000-08-10
Romain Kroes pose la question :
Pour les beoins d'un ouvrage d'économie que je suis en train de terminer, pouvez-vous me dire qui est (sont) l'inventeur de l' "isomorphisme" en mathématiques (calcul tensoriel)?
Claude Tardif lui répond.
Derivées partielle 1999-10-19
Arnaud Flandin pose la question :
Quel est la definition des derivées partielle
Claude Tardif lui répond.
Le sang humain 1998-10-06
Golden pose la question :
Le sang humain contient approximativement 2,5 x globules rouges. Chaque globule a un rayon de 0,004 mm. Si on le place ces globules bout a bout, quelle est la longeur de la chaine en millimetres? En kilometres? Compare toi avec la circonference de la terre (24 000 km) (Travail avec exposants)
Claude Tradif lui répond.
The area of a lot 2022-12-31
Brian pose la question :
To calculate lot size, do you use arc length or chord length? Lot Survey shows: Length 1 = 120.0' Length 2 = 120.0' Width 1 = 61.0' Width 2 = ARC Length 125.58' / CHORD Length 124.10' / Radius = 236.0'
Harley lui répond.
Velocity and acceleration 2021-01-09
yen pose la question :
The motion of a stone projected upward is given by the relation s=112t - 16t^2. What are the values of velocity and acceleration when t=3 sec?
Penny Nom lui répond.
Running around a circular track 2020-10-31
kaylee pose la question :
Keiko and Kendra are training for an international track competition. Kendra is running one lap on the outside of a circular track with a diameter of 1000 feet. Her constant rate of speed is 12 mi/h. Keiko is running one lap on the outside of a nearby circular track with a diameter of 1125 feet. She is also running at 12 mi/h. Find the time, in minutes, each runner needs to complete one lap.
Harley Weston lui répond.
Two equations with fractions 2020-10-22
mia pose la question :
I am supposed to look for x and y. my teacher didn't explain very well.
please help : ((
2x - y = 5
x/4 + y/3= 2

Penny Nom lui répond.
Factoring an interest formula 2020-09-29
Kenneth pose la question :
Hello

P + Pr has a common factor of P so it can be expressed as P(1 + r) after the P is factored out.. How does the "P" get in front of (1 + r)?

P/P + Pr/P = 1 + r What step is used to show that P is added in front of (1 + r)?

I thank you for your reply.

Harley Weston lui répond.
Simultaneous equations 2020-09-05
Paa pose la question :
a+b=8
a+c=13
b+d=8
c-d=6
find the value of each letter

Penny Nom lui répond.
The angular speed of a wheel 2020-09-03
Catrina pose la question :
A car is moving at a rate of 75 miles per hour, and the diameter of its wheels is 2.6 in. Find the angular speed of the wheels in radians per minute.
Penny Nom lui répond.
4sin(2x)cos(2x)+1=0 2020-07-02
Sheila pose la question :
4sin(2x)cos(2x)+1=0
Penny Nom lui répond.
A triangular garden 2020-05-24
yoonji pose la question :
the 3 sides of a triangular garden measure 200 ft., 250 ft., and 300 ft., respectively. the garden is to be divided by a line bisecting the longest side and drawn from the opposite vertex. what is the length of the bisecting line?
Penny Nom lui répond.
A system of equations. 2020-04-27
solomon pose la question :
xy + x =28
x + y +4

Penny Nom lui répond.
A rectangular garde 2020-04-10
Denise pose la question :
In a rectangular garden a person says "if I made that bed 2 feet wider and 3 feet longer it Would have been 64 sq,ft bigger. But if it had been 3 feet wider and two feet longer It would have been 68 sq.ft bigger.
What is the length and width of the garden bed?

Penny Nom lui répond.
Form a square and a triangle from a wire 2020-04-08
Raahim pose la question :
2. A 2 meter piece of wire is cut into two pieces and once piece is bent into a square and the other is bent into an equilateral triangle. Where should the wire cut so that the total area enclosed by both is minimum and maximum?
Penny Nom lui répond.
A times table in base 5 2020-03-11
Felix pose la question :
Design a time table for numbers 1-10 in base 5
Penny Nom lui répond.
Multiplication base 8 2020-02-19
Tamesha pose la question :
Let’s count with the symbols 0,1,2,3,4,5,6,7 (base 8)
A). Create a multiplication table for this base
B). Compute the following products using the multiplication algorithm
I). 7605 times 3713
II). 63725 times 40627

Penny Nom lui répond.
Sipping juice from a Tetra Pak 2020-02-11
Anjhelic pose la question :
Karen is sipping juice from a 1 in. by 3 in. by 6 in tetra pack at the rate of 0.5in³/sec. How fast is the height of juice in the pack decreasing?
Harley Weston lui répond.
Investigating y = (-2)^x 2020-01-13
Gonzalo pose la question :
This is not precisely a maths question, but it is formulated based on my maths curiosity. Fidgetting with my new graphic calculator, I started graphing things and had the idea to graph $y=(-2)^x.$ The result surprised me, and I thought a little bit about it, stored it on the back of my brain, and promised myself to look deeper into it someday.
Harley Weston lui répond.
Two people are clapping their hands 2019-12-06
Yukti pose la question :
Two people are clapping their hands, but with different frequencies. Ramya claps her hands 17 times per 3 minutes, and Kumar claps his hands 31 times per 5 minutes. They start clapping at the same time. How many times does each one clap before they clap together at the same time again?
Penny Nom lui répond.
cos (3pi/2 -x) 2019-11-20
sam pose la question :
1. express the following as a trigonometric function of angle x for cos (3pi/2 -x) the answer in the textbook is -sin x but i am unsure of the process to get the answer.
Penny Nom lui répond.
Simultaneous equations 2019-10-16
deepak pose la question :
This is too complicated please help me
8/x - 10/y = 1 and x+y=9

Penny Nom lui répond.
Packing chocolates in boxes 2019-07-22
CHESKA pose la question :
1.levy can pack the chocolate bars either with 35 bars in a box,with 42 in a box, or with 49 in a box. what is the least number of chocolate bars that he has?
2. the blue light of a Christmas light blinks every 2 second. the red light blinks every 3 second and the yellow light blinks every 5 second. how many times will the three lights blink at the same tims in one hour?
3.a wooden straightedge can exactly measure an 825 cm rope, a675 cm rope, and 375 cm rope. what is the least possible length of the straightedge? how many times will it measure each rope?

Penny Nom lui répond.
The area of a triangle in acres 2019-06-23
Karen pose la question :
if I have a triangle that measures 78 ft x 54 ft x 50 ft, how do I figure out what part of an acre that is??
Harley Weston lui répond.
Maximize monthly revenue 2019-05-23
a student pose la question :
A real-estate firm owns 100 garden type apartments. At RM400 per month, each apartment can be rented. However, for each RM10 per month increase, there will be two vacancies with no possibility of filling them. What rent per apartment will maximize monthly revenue?
Penny Nom lui répond.
Constructing a hexagonal pyramid 2019-04-25
REBECCA pose la question :
I'm doing a project for my Honors Geometry class and I have to build a regular hexagonal pyramid. I was given the volume of .75 gallons, but I don't know how to figure everything else out. Please help?
Thanks,
Rebecca

Harley Weston lui répond.
A circle with a circular hole 2019-03-11
Sue pose la question :
My little one is wondering if a circle with a circular hole could be described as an irregular semicircle.
As it has 2 sides but is not the standard shape. Could you share your thoughts ??

Penny Nom lui répond.
Running around a circular track in opposite directions 2019-02-24
carmi pose la question :
how many times will lita and rose pass each other in 15 minutes if lita and rose start jogging on a 110 meters circular track. they begin at the same point but in opposite direction. lita at 8/3 meters per second and rose at 7/3 meters per second.
Penny Nom lui répond.
Loan payment formula 2019-02-24
Kenneth pose la question :

I have a question regarding the loan payment formula shown below.

Calculating the Payment Amount per Period
The formula for calculating the payment amount is shown below.

Simple Amortization Calculation Formula

A = P X r(1 + r)n over (1 + r)n - 1

where

A = payment Amount per period

P = initial Principal (loan amount)
r = interest rate per period

n = total number of payments or periods

Is this formula/calculation a condensed version of a longer calculation? I am curious to know how the (1 +r)n - 1 was developed from the longer calculation. For example, r(1 + r)n may have been (r + rn)n. The n's are exponents.

I thank you for whatever helpful explanation that may be provided.

Kenneth


Harley Weston lui répond.
cos(2x) = sin(x) 2018-12-18
Liz pose la question :
Determine all values of x in [0,2pi] that solve the equation Cos(2x)=sinx
Penny Nom lui répond.
Simultaneous equations with fractions 2018-12-14
zaheer pose la question :
solve simultaneous equations and give answer in fractional form
3x - 2 = 4y +5/3
y + 7 = 2x + 4
would really appreciate some help on this please

Penny Nom lui répond.
Angular velocity 2018-12-05
Kolade pose la question :
Krusty the Clown and Sideshow Bob have covered themselves in velcro, and shot themselves out of a large cannon so that they stick to a huge rotating velcro wheel (don’t try this at home). As shown in Figure 1(a), Krusty is now stuck to a point 3.00 m from the center of the wheel while Bob is stuck to another point 5.00 m from the center. The wheel is rotating counterclockwise at 10.0 rpm. Conveniently a set of axes is suspended just in front the wheel (Krusty asked for axes, like the ones you would use to chop wood, so that he and Bob could chase each other with them in the usual wholesome entertainment he is known for. Unfortunately, the set designer misunderstood.) The axes do not rotate with the wheel.
(a) Find Krusty’s and Bob’s angular velocities, in radians per second.

Penny Nom lui répond.
Two cones formed from two circular sectors 2018-11-30
APARNA pose la question :
The central angles of two sector of a circle are 60° and 120° . Using these sectors two cones are made. If the radius of the smaller cone is 5 cm, find the radius and base area of the larger cone.
Penny Nom lui répond.
Angular speed 2018-10-18
Akidima pose la question :
A bicycle wheel rotates uniformly through 2.0 revolutions in 4.0 seconds . What is the average angular speed of the wheel
Penny Nom lui répond.
The volume of a hotel soup pan 2018-09-15
Melanie pose la question :
If the hotel pan measures 12 inches long x 5 inches wide x 6 inches tall, how many quarts of soup will the hotel pan hold?
Penny Nom lui répond.
Working with percentages 2018-08-26
Kenneth pose la question :
16 is 4% of what amount? The answer can be determined by the following unusual manner: 4% = 16 1% = 4 100% = 400 Can the following solutions for the following be determined by using the same type of calculation? 16 is ?% of 400 ? is 4% of 400 I thank you for your reply.
Penny Nom lui répond.
Amy and Joey on a circular track 2018-08-13
Zoe pose la question :
Amy and Joey are situated on a circular track 400 feet around.
Joey starts running at a rate of 10 feet per second.
Amy waits for a full minute, then starts running from the same point (in the same direction) at 12 feet per second.

How many seconds elapse before Amy passes Joey on the track?

Penny Nom lui répond.
A sulphuric acid solution 2018-07-12
Tan pose la question :
How much 96% h2so4 to add into sulphuric acid 30% in order to make it to 50% sulphuric acid?
Penny Nom lui répond.
A circle inscribed in a regular polygon 2018-07-12
Naveen pose la question :
The radius of inscribed circle for n sided regular polygon of a side a is? Please with proof
Penny Nom lui répond.
The base for a circular pool 2018-06-10
Adam pose la question :
How many sheet of 4ft by 8ft foam would I need to lay under a pool 15dt in diameter
Penny Nom lui répond.
Subdividing a rectangle into squares 2018-05-13
jeverzyck pose la question :
A rectangular board is 108cm wide and 156 cm long Equal squares as large as possible are ruled off this board.Find the size of the square.How many squares are there?
Penny Nom lui répond.
A rectangular prism with rounded corners 2018-04-25
Jackie pose la question :
Rectangular prism that is 40 by 20 with rounded corners with a radius of 2.5.

What would the surface area be?

Penny Nom lui répond.
A cone formed from a circular sector 2018-04-18
Jessica pose la question :
A circle has a radius of 7.5cm. A sector with an angle of 240 degrees is cut out from the sector. If the sector is folded to form a cone. Find the length of the cone.
Penny Nom lui répond.
Covering a region with mulch 2018-04-13
Roberta pose la question :
I have a fall zone area of approx. 700 sq ft. that I have to cover with a 6in depth of mulch. How much mulch would I need to purchase?
Penny Nom lui répond.
0.366 x cos square (02 degree 17 mins 27 seconds) 2018-03-12
michael pose la question :
what is 0.366 x cos square (02 degree 17 mins 27 seconds)
what is 0.366 x cos square (88 degree 26 mins 45 seconds)

Penny Nom lui répond.
The limit of (e^x-1)^(1/x) as x tends to 0 2018-02-27
ARGHA pose la question :
Find the limit of (e^x-1)^(1/x) as x tends to 0.
Penny Nom lui répond.
Multiplies and factors 2018-02-24
Lil pose la question :
Is a multiple the same as a factor?
Penny Nom lui répond.
Dividing a region in half 2018-02-24
mandy pose la question :
There is a line through the origin that divides the region bounded by the parabola y=4x−5x2y=4x−5x2 and the x-axis into two regions with equal area. What is the slope of that line?
Penny Nom lui répond.
3 yards 6 inches times 7 2018-02-08
Tamara pose la question :
3 yards 6 inches × 7
Penny Nom lui répond.
Simplify 1/2^i 2018-01-28
Deepak pose la question :
How do I simplify this complex equation:

z=1/(2^i)

Penny Nom lui répond.
The equation of a line 2018-01-23
Gloria pose la question :
Write the equation of a line in standard form through the point (1, 6) and perpendicular to the line 2x –y = 9.
Penny Nom lui répond.
When are the hands of a clock perpendicular? 2018-01-11
Nazrul pose la question :
How many times in a day are the hands of a clock perpendicular to each other? How can I find the times? Please help me.
Penny Nom lui répond.
Needing the area of an irregular Pentagon 2017-12-06
Dale pose la question :
I'm needing to find the square footage of an irregular pentagon.
I've seen the other questions similar to this one, but without a great deal of difficulty can't get you the angle degrees, or the length between opposing angles.
I'm hoping that a different bit of information can help. So here goes:
The longest side is 67ft long.
One of its legs is 39ft long, and the other is 18ft long.
The leg off of the other end of the 39ft leg is 35ft long.
The leg off of the other end of the 18ft is 49ft long, and meets the other end of the 35ft side.
The information that I DO have, which I'm hoping makes the difference, is that the 39ft and the 18ft sides are parallel.
Also, while the angles at each end of the 67ft side are not right angles, they are fairly close to it, approximately 80 degrees at the 39 ft side corner, and 100 degrees at the 18ft side corner.

Dale.

Harley Weston lui répond.
Formula when i know the GST total 2017-10-31
Duncan pose la question :
HI - I've added up the GST from a pile of receipts and would like to know what the total cost of goods would be without GST. I don't want to add up 100's of receipts again with the cost before GST but I do need this total cost. i.e.) I know I paid $400 GST, what would the formula be to find out the cost of what I purchased before taxes? Your help is much appreciated!
Penny Nom lui répond.
formula when i know the GST total 2017-10-31
Duncan pose la question :
HI - I've added up the GST from a pile of receipts and would like to know what the total cost of goods would be without GST. I don't want to add up 100's of receipts again with the cost before GST but I do need this total cost. i.e.) I know I paid $400 GST, what would the formula be to find out the cost of what I purchased before taxes? Your help is much appreciated!
Penny Nom lui répond.
A tangent to a curve 2017-10-22
Jasem pose la question :

Suppose that
f(x)=(3x-3)^1/2.

(A) Find an equation for the tangent line to the graph of f(x) at x=2

(B) Find all values of xx where the tangent line is horizontal, and enter them as a comma-separated list (e.g., 2,-3,6). If there are none, enter none.

Values of x


Penny Nom lui répond.
The distance between the origin and a moving point on a graph 2017-10-16
Paulina pose la question :
Find the rate of change of the distance between the origin and a moving point on the graph of y=x^2 +1 if dx/dt=2 centimeters per second
Penny Nom lui répond.
An enclosure for some remote control boats 2017-10-03
Roberto pose la question :
I am trying to build a "ring" for our remote control boats, using irrigation pipe. I'd like to use full 20' long sticks for the long sides and use eight of what we call 45s, which are actually 135' fittings to make the six short sides. If I know the length of the two long sides, what would be the formula to find the exact lengths of the short sides?

Thank you.

Penny Nom lui répond.
The volume of a cone without calculus 2017-10-02
Akash pose la question :
How to find the volume of a cone without the knowledge of calculus?
Penny Nom lui répond.
Four multiples of 10 2017-09-23
Laudacir pose la question :
Four multiples of 10 are added together.the total is a 3 digit number with three consecutive digits. What could the four number be?
Penny Nom lui répond.
Simultaneous equations with fractions 2017-09-09
Farah pose la question :
Hi, my name is Farah. I hope you can help me with this question . X/2 + g/5= 3 and 2g - f = 10
Penny Nom lui répond.
The volume of an attic 2017-09-07
paul pose la question :
trying to determine volume of an attic base is rectangular 29 ft by 37 ft peak runs parallel to longest base side and is 7 ft long roof lines run from each end of the peak to the respective corners the peak is 3.5 ft above the base
(so seen from above there are 5 visible lines: the peak and 1 line to each of the 4 corners)

Harley lui répond.
Simultaneous equations 2017-09-02
keto pose la question :
x-y=2,x^2+xy=12
Penny Nom lui répond.
A rectangle and a circle 2017-08-12
Ramzan pose la question :
a diagram shows the shape of rectangular framework with length (2x+20)cm and with (y+10)cm . The outline is made of wire . If a circle with diameter 56 cm is to be made from the wire and the area of the rectangle is 420 square centimeter ,find the possible lengths and width of the rectangle
Penny Nom lui répond.
An arched window 2017-07-24
Gerry pose la question :
Looking to make a full size template for an arched window. Need increments every 16". The radius is 138' 0 9/16" , the chord is 226" and the rise at center of chord is 43"
Penny Nom lui répond.
3 consecutive multiples of 11 2017-07-22
nisha pose la question :
using the multiples formula shown at ypur site how can we solve finding 3 consecutive multiples of 11 whose sum is 363
Penny Nom lui répond.
Parallel and Perpendicular lines 2017-07-17
Karen pose la question :
Find an equation for the line with the given properties. Perpendicular to the line 7x + 2y = -6; containing the point (-2, -1)

Find an equation for the line with the given properties. Parallel to the line 5x - 3y =-6; x intercept = 3

Penny Nom lui répond.
Simultaneous equations with fractions 2017-06-02
Jamal pose la question :
1/x + 1/y =5 and 1/y - 1/x =-1
Penny Nom lui répond.
What is the regular price? 2017-05-11
Saira pose la question :
If the sale price is 55.50$ and the discount is 12% what is the regular price
Penny Nom lui répond.
Canada's population as a percentage 2017-05-11
bethmarie pose la question :
What percentage of the world population (7.5 billion) is Canada's population (36 million). Thank you very much!
Penny Nom lui répond.
Why does 10x10x10 give 1,000.0000000000001? 2017-04-12
Randolph pose la question :
Hi, On your calculator I found that a box 10 by 10 by 10 inches has a volume of 1,000.0000000000001 cubic inches. Can you explain the numeral 1 thirteen places past the decimal? Thanks, Randy
Penny Nom lui répond.
Quadratic Equations 2017-03-28
Natalie pose la question :

Hi Maths Central,

I am having trouble with quadratic equations. The question is:

Calculate the length 'l' of a rectangular pool by creating and solving a quadratic equation.

1. The rectangular pool is of equal area to a square pool, and a circular pool (5.70m^2).
2. The length of the rectangular pool must be 2m longer than the width 'w'.

Thank you so, so much for your help, Maths Central!


Penny Nom lui répond.
Differentiate y = x^x^x 2017-03-19
Nafis pose la question :
differentiate y = x^x^x
Penny Nom lui répond.
A regular tetrahedron 2017-03-12
Jeramie pose la question :
When we have a regular tetrahedron, is it always true that all the angles of the triangles measure 60 degrees?
Penny Nom lui répond.
A block base for a plywood rectangle 2017-02-18
Joe pose la question :
How many cinder blocks 16x8 would i need to build a solid base for a plywood base 54inches wide and 30 inches deep Thank you
Penny Nom lui répond.
Vinculum 2017-01-27
Dick pose la question :
What is the line or bar over a Roman Numeral called?

Eg. V with the bar over it representing 5,000 .

Penny Nom lui répond.
The volume of a slab 2017-01-21
Russell pose la question :
Working with a segment of a circle. The O.D. is 3.440" the length of the chord is 3.130 I need information on the volume of the largest area. I would like to know (in thousands of an inch) how tall would the largest area need to be to equal .1143 cubic inches If you need more info I can send it Thanks in advance
p lui répond.
Angular speed 2017-01-21
Kenzi pose la question :
A wheel of radius 14 inches is rotating 45°/sec. What is the linear speed in in/sec and the angular speed in RPM? (Round your answers to three decimal places)

I already found the linear speed. I got 10.996 in/sec. I don't know how to find the angular speed.

Penny Nom lui répond.
The average rate of change of cot(t) 2017-01-18
Brianna pose la question :
Hello! It's been a while since I've taken a math course, and I'm stuck on a problem in my calculus course.
The question is this:

Find the average rate of change of the function over the given interval.
h(t)=cot(t) a) [5pi/4, 7pi/4]

Penny Nom lui répond.
Five equations 2017-01-16
Muhammed pose la question :
A x 4 = E
B ÷ 4 = E
C + 4 = E
D - 4 = E
___________
& A + B + C + D = 100
what is the value of E

Penny Nom lui répond.
8^3/2(2+2) 2017-01-13
Mary pose la question :
8^3/2(2+2)
Penny Nom lui répond.
Simultaneous equations with fractions 2016-12-21
Kimi pose la question :
I am stuck on this,can someone please help me????

1/2x+1/3y=11
8x+2/5y=102
Can someone please explain how to solve questions like these?? Or else I will never learn...
Thank you

Penny Nom lui répond.
Two concentric circles 2016-12-21
shrestha pose la question :
Two concentric circles have radii of 14 cm and 7 cm respectively. Find the area of space between them.
Penny Nom lui répond.
Two boys on the same route 2016-12-06
Abbey pose la question :
Starting from the same spot, one boy completed a route in 9 minutes and another boy completed it in 15 minutes. When will both boys be at the same starting point at the same time again?
Penny Nom lui répond.
Simultaneous equations 2016-11-25
Rohan pose la question :
x-2/3 + y - 1/4 = 13/12
2-x/2 + 3 + y/3 = 11/6

Penny Nom lui répond.
Multiplication and repeated addition 2016-11-23
Anandmay pose la question :
Hello I was looking closely at early arithmetic where I found how we discovered properties of Arithmetic. Like:2 x 3 = 3 x 2. This can be proved by considering a 2-D figure(actually,quadrilateral) having length consisting of 2 boxes of 1-by-1 dimensions and breadth of 3 boxes of the same dimensions. Now,consider it again,but,this time,length of 3,and breadth of 2 of such 1 by 1 boxes. We now notice that we can fit the 2 types of rectangles formed on each other precisely. So the multiplicative property of commutativity is true for all natural numbers as we can generalize the result(in our mind,for self satisfaction).

Now,can you find me a nice satisfactory reason of why a fraction times a natural number equals the number times the fraction? I mean, for example,i can understand the meaning of 3 x 2/3 to be three times 2-3rd,that is, 2/3+2/3+2/3.Fair enough. But here is the problem:By definition and actual meaning of multiplication, a x b means the repeated sum of b,done 'a' times. So what is the meaning of doing 2/3 x 3?The repeated addition of 3 how many times??2/3 times??Not making sense,right?And even we have not proved yet the commutative property of numbers INCLUDING fractions.So how can we resolve this problem and make these things meaningful?
Thanks

Penny Nom lui répond.
Volume of liquid remaining in a tilted cylinder 2016-11-08
Brian pose la question :
I am trying to determine the amount of a liquid remaining in a 55 gallon drum when it is tilted at 45 degrees and the liquid level is low enough so that the liquid does not completely cover the bottom of the drum.

Your help is greatly appreciated.

Harley Weston lui répond.
A pattern rule for a sequence 2016-10-28
Grace pose la question :
Here's a Grade 7 question, we need to find the pattern rule:

1 - 1
2 - 3
3 - 6
4 - 10
......
It may involve exponents, but we can't seem to figure it out.
thank you.
Grace.

Penny Nom lui répond.
An airplane schedule 2016-10-14
Stephanie pose la question :
Starting at 8:00a.m., a passenger plane leaves from an airport every 6 minutes and a cargo plane leaves every 7 minutes. When is the next time these kinds of planes will leave at the same time?
Penny Nom lui répond.
Hexagonal pyramid bevel angles 2016-09-30
Peter pose la question :
I have seen your response to a similar question from Steve which Chris and Harley responded to, however I am not familiar with some of the terms. Is there a formula that I can enter my details in which will give me the specific angles I require. The item I am constructing is much larger than the one you responded to previously. Thank you for any assistance you are able to provide.
Penny Nom lui répond.
2,006-1-2-3-4-...-48-49-50 2016-09-24
Mimi pose la question :
Compute the following:

2,006-1-2-3-4-…-48-49-50

Penny Nom lui répond.
The width of a 300 foot long rectangular acre 2016-09-21
Debbie pose la question :
If 2 sides of a 1 acre plot is 300 feet each what are the other 2 sides? The acre is a rectangle.
Penny Nom lui répond.
The distance around a warehouse 2016-09-21
Chuck pose la question :
How far do I walk when I walk around a 751,000 sq. ft. warehouse?
Penny Nom lui répond.
Integrate (x^2 - 4x + 4) ^4/3 2016-09-15
Ifah pose la question :
Hi i have questions please answer
Integral 2 sampai 3 dari (x² - 4x + 4) ^4/3 dx

Penny Nom lui répond.
The modulus of a complex number 2016-07-29
Cheyenne pose la question :
There's a question on my Summer Assignment I cant figure out. Here it is:

Find the absolute Value of the complex number. -5i

Please help?

Penny Nom lui répond.
Two equations with fractions 2016-07-22
kanesh pose la question :
p/2+q/3=3

p/4+2q/3=3

Penny Nom lui répond.
Simultaneous equations 2016-06-25
sena pose la question :
2x/3+3y/2=-1
4x-5y=22

simultaneous equation

Penny Nom lui répond.
The difference between the ares of two rectangles 2016-06-09
Ingrid pose la question :
I am trying to help my son with an area question.
I have the answer, from the solutions, but I cannot figure out how to teach him.

Question:

Two rectangles have lengths 13cm and 19cm respectively.
Their total area is 376cm squared.
If both their widths are whole numbers, what is the difference in their areas?

I know that this is solvable once I determine the widths of the rectangles ,
but how do I go about finding that?

Thanks for your help

Chris Fisher and Harley Weston lui répond.
Implicit differentiation 2016-06-06
Pranay pose la question :
Is a circle x^2+y^2=2 a function? If it is not a function, why is it possible to do implicit differentiation on it?
Thanks.

Penny Nom lui répond.
A barn roof 2016-05-29
Joe pose la question :
Is it possible to build a barn roof (irregular pentagon?) with a 12' base and the other 4 sides 4' each? Thanks.
Penny Nom lui répond.
Two pieces of rope 2016-05-21
Render pose la question :
Fred cuts a 12- inch piece of rope. Then he cuts a second piece of rope that is 3/2 longer than the first piece. Is the second piece shorter or longer than the first piece? Explain.
Penny Nom lui répond.
A trough with a triangular cross section 2016-05-21
Clarice pose la question :
A trough having an equilateral triangle end sections has sides equal to 0.4 m and 7m long.what is the volume of the liquid in the container if the depth of the water is one half the depth of the trough?
Penny Nom lui répond.
Euler's polyhedron formula 2016-05-20
Antonia pose la question :
why does not cylinders follow Euler's polyhedron formula ( V-E+F=2) ?? if a cylinder has 3 faces, 2 edges and 0 vertices it doesn't follow Euler's formula???
Penny Nom lui répond.
The volume of a truncated rectangular pyramidal pond 2016-05-13
Paul pose la question :
How do you calculate a partially filled truncated rectangular pyramid if you always know the bottom rectangle, the maximum height top rectangle perimeter, but have a varying height. Similar to filling up a pond you know the current height and dimensions at the max rectangle how do you calculate it half full i.e. 10x30 outside perimeter with a 2x8 base and a max height of 6ft how do you calculate it at 3ft without re-measuring the top perimeter. Thanks, Paul
Harley Weston lui répond.
A pair of equations with fractions 2016-05-11
tiya pose la question :
hello, i want to know how to solve this question.

m/6+2n/3=6
-m/10=2n/5=2

Penny Nom lui répond.
A rectangular field 2016-05-07
Jan pose la question :
A rectangular field is 63m long and 21m wide. A fence of 2m in height is needed to fence the field and fencing is also required to divide the field in three square sections. There are three wooden gates measuring at 1.5m to the three sections of the field. What is the total surface area of the fence needed for fencing and diving the field?
Penny Nom lui répond.
Solve 2^2x + 3(2^x) - 4 = 0 2016-04-27
Lloyd pose la question :
Solve the equation 2^2x + 3(2^x) - 4 = 0
Penny Nom lui répond.
sin 2x - sin x = 0 2016-04-24
lilly pose la question :
sin 2x - sin x = 0 0
Penny Nom lui répond.
600(1+0.05)^7 2016-04-21
chelsey pose la question :
I am unsure on how to solve this problem, 600(1+0.05) 7 ?
Penny Nom lui répond.
15.86 acres 2016-04-21
Ronald pose la question :
If a rectangular piece of land is 1320 ft. on one side, how many feet would be required on the other side to equal 15.86 acres?
Penny Nom lui répond.
External and interior angles of a regular polygon 2016-04-19
pearl pose la question :
a polygon has n sides.The exterior angle is 8 times the interior angle
find the value of the interior angle
find the value of n

Penny Nom lui répond.
Heights at various points along an arc 2016-04-15
Isiah pose la question :
So I am working on a problem with a few friends; you have an arc with the center of its chord at 0,0. We also have a known sagitta and a known radius of curvature. How do we calculate the height extending in both the positive and negative directions?
Central sag: 2.48
Chord length: 9.6

Penny Nom lui répond.
Sloping my yard away from my house 2016-04-15
Harold pose la question :
I need to slope a area of my yard away from my house and garage. The length of my house is 80 feet long and i need to slope it 22 feet away. I want to raise the dirt at the foundation by 6 inches and slope it down to nothing at the width of 22 feet. How much soil would i require?
Penny Nom lui répond.
Pizza delivery 2016-04-03
Andrew pose la question :
Suppose you own a local pizza restaurant, Pizza with Pizzazz, and in order to compete with the big pizza chains in your area you are considering an advertising campaign offering customers a free pizza if their pizza is not delivered in 30 minutes or less. Even though your pizza restaurant is known for its fast and friendly delivery service, you are not sure if you can afford to give away too many pizzas for free. Looking over your past 12,421 pizza deliveries you find out 97% of the pizzas were delivered in 30 minutes or less.

Calculate the probability that all 100 randomly selected pizza deliveries will have been made in 30 minutes or less? Hint: Use the multiplication rule. (Round 4 decimals)

I'm really stumped on how exactly to do this problem. I don't need the answer just the steps broken down to show me how to get it! It would be gladly appreciated if you could help!

Penny Nom lui répond.
Maximizing the area of a two lot region 2016-04-03
yousef pose la question :
A man wishes to enclose two separate lots with 300m of fencing. One lot is a square and the other a rectangle whose length is twice its width. Find the dimensions of each lot if the total area is to be a minimum.
Penny Nom lui répond.
A rotating schedule for a softball team 2016-03-25
Caroline pose la question :
I couldn't find a similar question, hopefully there isn't one!

I have a softball team of 18 people (7 girls and 11 boys), but am limiting each game to 13 players (6 girls and 7 boys). The season is 10 games long. How do I create a rotating schedule which allows members to play evenly? I tried separating the boys and girls to create rotation for each but I got confused. Some are unable to attend all games but for the simplicity of a rotation schedule can you please help?

Robert Dawson and Victoria West lui répond.
A percentage problem 2016-03-02
jehanzaib pose la question :
if x is a percentage: e.g. 40% than what does "x/(1+x)" = 29%. what is the explanation.
Penny Nom lui répond.
A rectangular prism 2016-03-02
Paula pose la question :
Trying to help with a math problem. I don't know how to set up the equation for this.

A rectangular prism has a volume of 7.875m3rd and a height of 3.5 m. What is the area of the base of the prism?

Thanks for your help!

Penny Nom lui répond.
Solve for x and y 2016-02-27
ntshidi pose la question :
Y=1/2x+4and1/4x-6
Penny Nom lui répond.
The sum of the angles of a triangle 2016-02-24
Sophia pose la question :
Does every triangle add up to 180 degrees? (Such as a unique triangle)
Penny Nom lui répond.
The interior and external angles of a regular polygon 2016-02-17
percy pose la question :
a regular polygon has n sides .The size of each interior angle is eight times the size of each exterior angle .
1.find the size of each exterior angle
2.calculate the value of n

Penny Nom lui répond.
Perpendicular lines 2016-02-14
Brendon pose la question :
points A (-4; 6) , B (1;-2), C (5;-3) and D (4; 1) are given prove that AC and BD are perpendicular to each other
Penny Nom lui répond.
Calendar arithmetic 2016-02-14
Jenalee pose la question :
January 1, 2001 is Julian Day 2 451 911 (the number of days that have passed since Day 0, January 1, 4713 BC).

If Julian Day 0 was a Monday, what day of the week was January 1, 2001?

Victoria West lui répond.
Two equations in two unknowns 2016-02-13
Anumba pose la question :
4x + 2y = 4
7x - y = -11

Penny Nom lui répond.
I was paid 67% of the total owed 2016-01-25
Julie pose la question :
I was paid $27000. This is 67% of the Total Owed. The Total Owed is unknown. How do I calculate the amount of the Total Owed?
Penny Nom lui répond.
A system of linear equations 2016-01-24
kareem pose la question :
my name is kareem
and i am a student i have a math puzzle and i tried to solved it but it always have same mistake
x-y=9
x+z=12
z-n=14
y+n=2

Penny Nom lui répond.
A Max/Min problem with an unknown constant 2016-01-17
Guido pose la question :
Question:

The deflection D of a particular beam of length L is

D = 2x^4 - 5Lx^3 + 3L^2x^2

where x is the distance from one end of the beam. Find the value of x that yields the maximum deflection.

Penny Nom lui répond.
A rectangular enclosure 2016-01-15
Amber pose la question :
A rancher wants to use 300ft of fencing to enclose a rectangular area of 4400 square feet. What dimensions should the rectangle be?
Penny Nom lui répond.
A triangular shaped garden 2016-01-13
Demetrie pose la question :
Dwayne's garden is triangle-shaped with two equal sides and a third side that is 4 ft more than the length of an equal side. If the perimeter is 49 ft, how long is the longest side?
Penny Nom lui répond.
The perimeter of a triangular plot of land 2016-01-11
Janelle pose la question :
I need to fence off 6.5 acres. The plot is triangular shape. How many feet would I be fencing off?
Penny Nom lui répond.
Water flowing through a rectangular drain 2016-01-08
Tiffany pose la question :
the cross section of drain is a rectangle 30 cm wide. If water 3.5 cm deep flows through the drain at a rate of 22 cm/s, how many litres of water will flow through in one minute?
Penny Nom lui répond.
Integration of dx/(x^2+1)^3 2016-01-07
Ishank pose la question :
Integration of dx/(x^2+1)^3
Penny Nom lui répond.
A relative maximum and a relative minimum 2015-12-28
kemelo pose la question :
show for the following function f(x)=x+1/x has its min value greater than its max value
Penny Nom lui répond.
Constructing simultaneous linear equations 2015-12-28
Deborah pose la question :
Linda thinks of a two-digit number. The sum of the digits is 8. If she reverses the digits, the new number is 36 greater than her original number. What was Linda's original number?

Thank you!

Penny Nom lui répond.
Linear equations in two variables 2015-12-13
priya pose la question :
I have problem in solving these equations please help me today itself very urgent:
I)2x+y=y
II)pie*x+y=9

Penny Nom lui répond.
Constructing a cross country district schedule 2015-12-13
David pose la question :
Retired math teacher trying to help son-in-law schedule a cross country district schedule. The would like to have four tri-meets each week for five weeks and one week of dual meets. The score the tri-meets as three duals, i.e. 1v2, 2v3, 1v3 constitutes one tri-meet. After many hours, not sure its possible, but I have forgotten a great deal! Thanks
Victoria West lui répond.
A schedule for a league with 15 teams 2015-12-05
Zac pose la question :
Hello,

I have a league schedule question. We have a league where we have say have 15 teams. Each team is to play each other twice during a 10 week session.
However each team tries only to play 3 matches(teams) per night(week).
How would we set up a random schedule for the 10 week session?

Victoria West lui répond.
A tangent line to a parabola 2015-12-02
pei pose la question :
Given that the line y=mx-5 is a tangent to the curve y=2x^2+3 find the positive value of M.
Penny Nom lui répond.
Archimedes, Euclid and "Circular Reasoning" 2015-11-15
Ron pose la question :
I have read about Archimedes and his work with sphere in cylinder and cone in cylinder and the volume relationships. Did he or any others also extend this to regular based polygon based regular like pillars, and columns? The ratio of 1/3 to 1 whole holds true with all regular based columns as example: a regular pyramid having a regular hexagon base inside a regular hexagon column of equal height.
Chris Fisher lui répond.
The area of a domed roof 2015-11-13
Brandon pose la question :
I have a tank with a 13' radius that has a domed roof of 3.5' tall How do I figure out the area in SQFT?
Penny Nom lui répond.
Fencing around a rectangular field 2015-11-11
Darlene pose la question :
Question from Darlene, a parent: A farmer has 10,000 meters of fencing to use to create a rectangular field. He plans on using some of the fencing to divide the rectangular field into two plots of land by constructing a fence inside the rectangle that is parallel to one of the sides. Let X be the width of the rectangular field. Write an equation to express the area of the field as a function of X. Find the value of X that maximizes the area of the field.
Penny Nom lui répond.
An 8 Team Basketball League Schedule 2015-10-28
Janice pose la question :
We have an eight team league that play at four time slots each week. It's a seven week season. How do we schedule so that each teams plays each other at least once?
Victoria West lui répond.
A schedule for a pool league 2015-10-21
Hazel pose la question :
I have to make a schedule for my pool league. I have twenty teams each of which must play all other teams. I have ten tables which must also be rotated to all teams play on all tables.
Victoria West lui répond.
Simplify -3(x-7) 2015-10-21
geraldine pose la question :
simplify: -3(x-7)
Penny Nom lui répond.
Scheduling a social curling league 2015-10-20
Tyler pose la question :
I'm scheduling a social curling league where all skips will play with all thirds, all seconds, all leads. And all thirds will do the same and so on. We have 6 teams of 4 and will be doing 6 rotations. Is it possible that all skips will play with all other players from other positions without anyone doubling up?
My initial thoughts were (1,1,1,1) (2,2,2,2) (3,3,3,3) (4,4,4,4)(5,5,5,5)(6,6,6,6)... then rotate the thirds up and seconds down but don't know what to do with the leads and even with just 3 positions there's doubling (1,2,6,x)(2,3,5,x)(3,4,4,x)...4s have doubled (4,5,3,x)(5,6,2,x)(6,1,6,x) as you can see I'm having problems
If you could let me know if this is even possible it would be greatly appreciated

Victoria West lui répond.
6 teams randomly arranged in groups of 3 2015-10-19
Anthony pose la question :
im trying to make a game scheduled for 6 teams using a 1vs2vs3 ; 4vs5vs6 format. all 6 teams randomly arranged in groups of 3. each team will play all teams at least twice, im trying to see how to make the number of times each team play another the same. schedule should be arranged as such 1vs2vs3 4vs5vs6 so that (1vs2, 1vs3, 2vs3) and (4vs5, 4vs6, 5vs6) i am limited to having only 2 sets of 3 at a time with 6 teams.
Victoria West lui répond.
An 8 team schedule 2015-10-19
Dean pose la question :
I need an 8 team schedule for 2 time slots where 3 teams are on the ice per slot. I understand that there will be 2 teams on a bye per day. How do make sure that the byes are fair? Can you help me out?
Victoria West lui répond.
A six team league to play a 20 game schedule 2015-10-19
Kelly pose la question :
Hi there. Some of your posts are close, but my problem is evening out the home and away games. We have 6 teams in our league and we are going to play a 20 game schedule, playing everyone 4 times. We will play Fridays and Saturdays, each weekend. Of course the ideal would be to have each team have a home and an away game each weekend, but I realize that is impossible, but I would like it as close as possible. Any help would be much appreciated!! We also were considering opening with a home and away - Friday then Saturday with the same teams. Then carrying on with playing everyone else and then possibly finishing up with the same home and home
Victoria West lui répond.
Six teams playing five games 2015-10-18
Aidan pose la question :
I have six teams that will be playing five games.
I want each team to play each game once and each team to play each other team once.
Is this possible or should I add an extra round and let some teams play each other twice?

Victoria West lui répond.
4 games in 4 time slots with 8 teams 2015-10-16
Paul pose la question :
I'm trying to set up a game matrix for my kids that will have 4 games in 4 time slots with 8 teams. And every team is playing a different game in a different time slot. I provided the initial matrix and I am trying to ensure that no 2 teams play each other more than once. That's my problem.
       8 pm 9 pm 10 pm 11 pm
Crokinole 1 vs 2
Trivial Pursuit 3 vs 4
Darts 5 vs 6
Pictionary 7 vs 8
Please help, I've been at it for hours.

Robert Dawson and Victoria West lui répond.
Squash scheduling 2015-10-01
Ed pose la question :
I have 5 teams and 2 courts. Every Thursday we play at four different times (6,7,8,9 o clock)

Each team will play each other 5 times over 25 weeks.

Everyone should get to play evenly at the best time (6:00) and the worst time (9:00)

Victoria West lui répond.
A 6-person team that has 9 players 2015-09-21
Sheri pose la question :
I am organizing a 6-person team that has 9 players. How do I schedule them fairly over a 9 week league?
Victoria West lui répond.
A hockey roster of 19 kids 2015-09-21
Brian pose la question :
We have a hockey roster of 19 kids. We only want to play 15 kids for each game. 4 players will not play each game. There are 16 games total. We want to rotate who doesn't play evenly. Can this be done? How many games will each player sit during the season?
Robert Dawson and Victoria West lui répond.
20 golfers for 6 days 2015-09-11
Dave pose la question :
have 20 golfers for 6 days. Each player should pair with 18 golfers used a system on your website which is good for 5 days, but cannot get the 6th day. Thanks for your help Dave
Victoria West lui répond.
Scheduling for a bridge club 2015-09-09
margot pose la question :
Can you help me with scheduling our bridge club?
8 rounds are played with four teams together.
There are 16 teams in total, so our schedule should look like eight 4x4 boxes.
(Each round the four teams play each other over the course of the evening)

Victoria West lui répond.
A schedule for eight teams and four stations 2015-09-04
Joel pose la question :
We are having a youth activity where we have eight teams and four stations. At each station there will be two teams competing against each other. We need each team to go through all four stations, but without ever competing against the same team, and, of course, without ever doing the same station twice.

If this possible? Thank you!

Victoria West lui répond.
A schedule for an 8 person team 2015-08-27
Julie pose la question :
We have 8 people on our team, but only 4 can play every week. We have 32 weeks, total. How can we divide this so everyone plays evenly ?
Robert Dawson lui répond.
Reinforced steel mesh in a circular foundation 2015-08-22
Padmesh pose la question :
in a circular foundation we are laying a reinforced steel mesh . i.e., like chords in both ways. the circle diameter is 2.91m and rods are placed 0.1m spacing. so I want to find the number of rods present in that circle. here by I am attaching an auto cad drawing for your reference
Penny Nom lui répond.
Rotating speakers through a 10 week schedule 2015-08-03
Timothy pose la question :
I want 15 students to speak 7 times each over the course of 10 weeks.
They each will have 5 minutes.
Thanks

Victoria West lui répond.
1 + 1 + 1 + 1 + 1 + 1 + 1 + 1 + 1 + 1 + 1 + 1 x 0 + 1 = ? 2015-06-18
Sharon pose la question :
1 + 1 + 1 + 1 + 1 + 1 + 1 + 1 + 1 + 1 + 1 + 1 x 0 + 1 = ?

I got 1 as my answer despite BODMAS making it 12 because logic tells me I ought to place brackets around the first set of repeated addition. Could you please clarify this for me? Thank you 😊

Harley Weston lui répond.
The surface area of a round bar 2015-06-06
chirag pose la question :
Please tell me the formula for finding round bar surface in square inches/ mm
Penny Nom lui répond.
Manipulating an equation 2015-06-05
Leigh pose la question :
Hi, I'm having trouble trying to find out how the equation y=mx+b for b comes up with the answer b=y-mx. Could you please assist me with this
Penny Nom lui répond.
The surface area of a triangular prism 2015-06-03
Alex pose la question :
Find the surface area of a triangular prism with Isoceles triangles and dimensions 4 feet by 6 feet
Penny Nom lui répond.
A tangent to y = x^3 2015-05-31
Brayden pose la question :
Show that a tangent line drawn to the curve y=x^3 at the point (d,f (d)), where d>0, forms a right triangle with the x and y axes in quadrant 4 whose area is (2/3)d^4.
Penny Nom lui répond.
A pentagon inscribed in a circle 2015-05-30
Victoria pose la question :
find the area of a regular pentagon inscribed in a circle with radius 3 units
Penny Nom lui répond.
A pile of mulch 2015-05-15
Justin pose la question :
I have a pile of mulch that measures 19 feet wide at the bottom and has a height of 6 feet. I need to know how many yards of mulch I have.
Please help and help with the formula for further use. Thank you!

Penny Nom lui répond.
Two lorries approaching an intersection 2015-05-15
Nuraini pose la question :
Two straight roads intersect at the right angles. Lorry A, moving on one of the roads, Approaches the intersection at 50mi/h and lorry B, moving on the other roads, approaches the intersection at 20mi/h. At what rate is the distance between the lorry changing when A is 0.4 mile from the intersection and B is 0.3 mile from the intersection?
Penny Nom lui répond.
A calculus optimization problem 2015-05-14
Ali pose la question :
Given an elliptical piece of cardboard defined by (x^2)/4 + (y^2)/4 = 1. How much of the cardboard is wasted after the largest rectangle (that can be inscribed inside the ellipse) is cut out?
Robert Dawson lui répond.
The method of elimination 2015-05-01
oreanna pose la question :

Question from oreanna, a student:

How do u solve 2x+9y=3

7x-4y=-25 in elimination


Penny Nom lui répond.
Constructing a box of maximum volume 2015-04-14
Margot pose la question :
I need to do a PA for maths and I'm a bit stuck. The PA is about folding a box with a volume that is as big as possible. The first few questions where really easy but then this one came up.

8. Prove by differentiating that the formula at 7 does indeed give you the maximum volume for each value of z.

Penny Nom lui répond.
Team selection 2015-04-13
Peter pose la question :
I manage a football team with a list of 28 players.
Only 24 can play each game.
The season is 16 games.
How do I fairly select 24 players each week.

Robert Dawson lui répond.
A word problem with fractions 2015-04-09
Lorraine pose la question :
If the numerator of a certain fraction is doubled and the denominator is increased by 1, the fraction becomes 1/2.

If the numerator of the original faction is squared and the denominator is decreased by 2, the fraction becomes equal to 1.

Let x be the numerator and let y be the denominator of the original fraction. Write down two simultaneous equation in x and y.

Solve these equations to find two possible values for the given fraction.

Penny Nom lui répond.
The area of the ring between two concentric circles 2015-04-08
Conner pose la question :
The area of the ring between two concentric circles is 25pi/2 square inches. The length of a chord of the larger circle tangent to the smaller circle is?
Robert Dawson lui répond.
n^2 is a multiple of 100 2015-03-30
Rahul pose la question :
I have to prove that n^2 is a multiple of 100 is necessary or Sufficient condition (or both) for n being multiple of 10
Penny Nom lui répond.
A lot on a cul-de-sac 2015-03-06
ana pose la question :
Julie lives on a cul-de-sac and wants to know if the city calculated the area of her plot of land correctly (see diagram below). We can assume the cul-de-sac is a circle and it has radius 40 feet.The boundary of the lot along the cul-de-sac (the arc from A to B) is 78.21 feet. The city found the area to be 7012.5 square feet–are they correct?
Penny Nom lui répond.
Two equations with fractions 2015-02-26
Pulane pose la question :
Hi math centre I've been trying to solve these equations for days now please help (6/x)-(1/y)=4
(9/x)+1=(-2/y)

Please help me solve them simultaneously
Thank you

Penny Nom lui répond.
f(x)=(x^2-1)/(x-1) 2015-02-21
Ahmed pose la question :
Is f(x)=[(x^2-1)/(x-1) and x=2 at x=1] differentiable at x=1 ? Why ?
Penny Nom lui répond.
Two equations 2015-02-16
nigel pose la question :
2x+1/2y=1
6x-3/2y=21

Penny Nom lui répond.
Bearings and a rectangular plot 2015-02-14
Mubashir pose la question :
A,B,C,D are the four corners of a rectangular plot marked on level ground. Given that the bearing of B from A is 040 degrees and that the bearing of C from A is 090 degrees. Calculate the bearing of
(a)B from C
(b)A from C
(c) D from C

Penny Nom lui répond.
A parallelogram inscribed in a circle 2015-02-04
narendra pose la question :
how can we prove that a circular parallelogram is a rectangle?
Chris Fisher lui répond.
128/(-16)/(-2) 2015-01-28
jackie pose la question :
128/(-16)/(-2) I was wondering if you can show me how to work this question out
Harley Weston lui répond.
A schedule for observing our peers 2015-01-23
Jennifer pose la question :
I am a sales trainer. Each month we need to observe one of our peers and give feedback. What this means is every month, not only am I observing someone, but someone is also observing me. For example, in January, Amy observes Tracy, Tracy observes Rachel, Rachel observes Ryan, and so on. The last person would then observe Amy, bringing it full circle. There are 10 trainers on my team. How do I set it up to where each trainer observes another trainer only once over the course of the year (and in return, each trainer is only being observed once by each person)? I know they will eventually repeat but would that start in the 10th month?
Robert Dawson lui répond.
A schedule for 5 employees working 2 at a time 2015-01-23
Cynthia pose la question :
We have 5 employees and need only 2 to work on Saturdays. what is the best way to fairly schedule everyone. PS. from time to time a sixth person will be thrown in to make things really interesting.
Victoria West lui répond.
Scheduling meetings with pairs of people 2015-01-22
Jacey pose la question :
I am trying to figure out a formula/system to pair up a list of people so they meet with each other every month, but they rotate who they meet with. Right now I have 13 people and I would like to just type in their names and then have the system put each person with someone else every month and rotate so no one gets the same person twice. Can this be done?
Victoria West lui répond.
Scheduling 12 for lunch 2015-01-05
Sabrina pose la question :
I have 12 staff members that need to have lunch together 1 time each week. I am trying to pair them in a way that we each have the same amount of lunches in the same amount of weeks. Please Help! Thanks!
Victoria West lui répond.
Revolutions per minute 2014-10-24
Edward pose la question :
Hello; I have a 28.2 inch diameter tire; do not worry about engine RPM or gear ratios, please tell me what the RPM of that tire is at 8 MPH and 64 MPH. Thank you.

Sincerely; Edward

Penny Nom lui répond.
A linear system 2014-10-14
pheter pose la question :
4/x - 1/y = 3 .... equation (1)
6/x - 2/y = 5 .... equation (2)

Penny Nom lui répond.
A fact family containing 4, 4 and 16 2014-10-14
Leslie pose la question :
I am helping my son and the question is 4,4,16 in fact family what does that mean
Penny Nom lui répond.
A tennis ball in a triangular prism 2014-10-06
Anna pose la question :
Hi, I need to find the the base and the height of a triangular prism, but I only know the diameter of a tennis ball that fits exactly in the triangular container.. The diameter for the ball is 8.50cm. Can you help me find the formula? Thank you!
Penny Nom lui répond.
Two equations in x and y 2014-09-25
seyilogo pose la question :
solve y=2x - 3 and (4x - 2y) / x + y = 1 simultaneously
Harley Weston lui répond.
Continuity on a closed interval 2014-09-21
Pragya pose la question :
The trouble I'm having is as follows :
a continuous function is most of the times defined on a closed interval, but how is it possible to define it on a closed interval ,because to be continuous at endpoints of the interval the function's limit must exist at that endpoint,for which it has to be defined in its neighborhood,but we don't know anything about whether the function is always defined in the neighborhood.
Please help...

Penny Nom lui répond.
Meetings for 77 students 2014-09-18
Adam pose la question :
We have a new group of students starting, and would like to set up a series of small group meetings so each student meets each other, hopefully without meeting the same people twice.

We have 77 students. In theory I would like to have 11 meeting slots, with 10 groups in each slot, with up to 8 students in each group. Is there a way to do this?

I have wondered if it might be easier to do 9 meeting slots, with 9 groups per slot, with 9 students per group. Thanks!

Victoria West lui répond.
Two equations in two variables 2014-09-18
Susan pose la question :
(28x + 36y) - [20000 - .75(28x + 36y) + 60000] = 5000
x + y = 10000
solving two equations involving variables

Penny Nom lui répond.
A tangent to a curve passing through a point not on the graph 2014-09-15
Aquilah pose la question :
For the curve y = x2 + 3x, find the equations of all tangent lines for this graph that also go through the point (3, 14).
Penny Nom lui répond.
A six team track schedule 2014-09-15
Paul pose la question :
I need to make a six (6) team track schedule. It needs to be five (5) triangular (3 team) meets with each team playing each team twice
Victoria West lui répond.
A barge of triangular cross section 2014-08-18
tushar pose la question :
a barge of triangular cross section is 20m long 12 m wide and 6m deep.its floats in SW at a draft if 4m find its displacement
Penny Nom lui répond.
Multiplication in base five 2014-08-07
Alise pose la question :
Hi Maths Central!

I was wondering if you could help me with multiplying in base 5. I have read a question previously answered by you in regards to this but got fairly confused by it and was hoping you could help me with my question.

My question is: Multiply 1422 base 5 by 21 base 5 keeping these numbers and answer in base 5.

Would you also be able to explain how to get to the answer?

Thanks!

Penny Nom lui répond.
A 6 team sports schedule 2014-08-02
Daniela pose la question :
Hello,

My husband and I are volunteering at our church's Jrs camp and we were given the task of making the schedule for sports. There will be 6 teams and a total of 3 games for two days. We would like for every team to play each other but we cannot figure it out. Pls help!

Thanks.

Victoria West lui répond.
60% as much as $30. 2014-07-18
Kenneth pose la question :
Question from Kenneth:

Hello:

Do you know why some "as...as" phrases indicate multiplication?

For example, ? is 60% as much as $30. This is equivalent to 60% X $30 = $18. How does this make sense?

I saw this example in an old textbook on business mathematics, but the author did not explain why it is equivalent to multiplication.

I thank you for your reply.

Robert Dawson lui répond.
A rotation schedule for 14 teams 2014-07-08
Daniel pose la question :
I am organizing team activities for a summer camp. We have 14 teams and 7 rotations and 7 different games. every team will play a different game each rotation. Where I am having trouble is I would like each team to play a different team each rotation without a repeat. Is that possible? if so what is the formula?
Victoria West lui répond.
The method of elimination 2014-07-05
leo pose la question :
please explain how can i solve this problem

3x-6y=-38
6x-9y=44

using elimination and simultaneous method thank you :)

Penny Nom lui répond.
Angular speed 2014-06-29
andrea pose la question :
a wheel having a radius of 10cm rotates such that the linear speed at its rim is 30mls. what is the angular speed of the wheel in rpm?
Penny Nom lui répond.
Differentiate ln[x(2x-4)^1/2] 2014-06-28
Igwe pose la question :
If y=In[x(2x-4)^1/2],find dy/dx at x=3
Penny Nom lui répond.
Simultaneous equations 2014-06-20
rana pose la question :
solve the simultaneous equations
a)3x=7y
12y=5x-1

Penny Nom lui répond.
20 teams and 10 competition stations 2014-06-04
Joel pose la question :
I have 20 teams. There are 10 competition stations. Each station will accommodate exactly 2 teams to compete with each other at a time.

Each of the 20 teams needs to compete at each of the 10 stations exactly one time.

Here's the catch I cannot solve: how to schedule it so that each team plays EVERY OTHER TEAM once and only once, while never repeating a station.

Victoria West lui répond.
10 golfers, playing 7 rounds in 2 x 3balls and 1 x 4ball 2014-06-04
adrian pose la question :
Hi, We are off to Scotland on our yearly golfing trip soon. Question is we have 10 golfers, playing 7 rounds in 2 x 3balls and 1 x 4ball. I would like to ensure that each golfer plays with the other guys an equal amount of times, guess 3ish? and we share the amount of times each player plays in the 4ball. Hope you can help?
Victoria West lui répond.
A rotating schedule for 9 individuals 2014-05-29
Josh pose la question :
I need to set up a rotating schedule for 9 individuals. The individuals will be pairing each week in groups of two. Thus, every week there will be 4 groups of two individuals and one individual not paired with anyone (i.e. a "bye week"). The rotating schedule should not repeat any pairing and it will continue until each individual has been paired with every other individual once.
Thank you

Victoria West lui répond.
A hexagon with different side lengths 2014-05-16
Grace pose la question :
Sorry if this is a simple question, but I can't seem to understand how to solve it. I am Trying to find the area of a hexagon with measurements 6x7x6x6x7x6, or 2 sets of opposing sides with 6, and one set with 7. Thank you.
Penny Nom lui répond.
A 12 week poker tournament 2014-05-12
Jamie pose la question :
Looking to schedule a 12 week poker tournament with 18 players ...playing 2 separate 9 person tables...I'm looking to have a balanced schedule where everyone player with each other an equal amount of time
Victoria West lui répond.
A schedule for 2 teams of 10 2014-05-07
John pose la question :
How can I set up a schedule for 2 teams of 10 on a team to play each individually against each other over a 10 day span at 10 different course with 10 different start times each day. I was looking to not repeat any competition. As an example, Team A would consist of players 1-10, Team B would consist of players 11-20. I would want to have 10 start times a day for 10 days where a member of team A plays a member of team B each day without repeating any matches and any of the same courses and not repeating any of the same start times. Any help in this would be GREATLY appreciated!!!!
Victoria West lui répond.
The derivative of sin(x) 2014-04-26
Lucky pose la question :
f(x)=Sin(x), by first principle its f'(x)...show me how to solve such problem.
Penny Nom lui répond.
A schedule for 9 teams on 3 fields for 18 days 2014-04-25
Scott pose la question :
I need to schedule 9 teams on 3 fields for 18 days with no buys. Two teams play while other practices in field(little guys). Two fields have backstops and other is open field. Would like to have even time on each diamond if possible. Thank you
Victoria West lui répond.
Solve for theta 2014-04-25
ALASTAIR pose la question :
Hi, The question asks Solve for 'theta' cos2theta=sintheta x costheta. Substituting for cos2th either 1-2sin^2th or 2cos^2th does not give an equation in either sine or cosine alone how do I solve this please?
Penny Nom lui répond.
Two 9 hole golf leagues 2014-04-23
Sally pose la question :
I need help scheduling two 9 hole leagues. Here are the variables:
- Leagues alternate each week starting on the front and back
- League A starts at 4:30 with 7 groups and League B starts at 5:15 with 8 groups (all use carts and must be teed off by 5:30)
- Regular play must continue, customers who have started before league play begins have preference to continue their round whether they are playing 9 or 18 holes.
Thank you!!!

Victoria West lui répond.
Golf with 3 teams of 20 2014-04-21
Joshua pose la question :
Thank in advance for this great service

Golf - new - 3 teams of 20 - to create 20 unique groups of 3

For example use 3 teams of 3

1a 2a 3a
1b 2b 3b
1c 2c 3c

So groups are
1a 1b 1c
2a 2b 2c
3a 3b 3c

Groups can only have one a, one and one c - and that combo should be unique

Victoria West lui répond.
Golf League Schedule for 15 weeks and three flights 2014-04-21
bigdog pose la question :
I need help setting up a 12 team (3 flights with 3 teams in each flight) 15 week league play schedule. Each team must play teams in there perspective flight twice during the 15 week season.
Victoria West lui répond.
Simultaneous equations with fractions 2014-04-19
Maryam pose la question :
I looked at your example of simultaneous equations with fractions and applied it to my question from an educate exam papers but I couldn't get it to work. The question is:

x/8 - y = -5/2
3x + y/3 = 13

Penny Nom lui répond.
The area bounded by the X-axis and y=x^(2)-4 from -5 to 0 2014-04-15
Lexie pose la question :
Determine the area that is bounded by the following curve and the x-axis on the interval below. (Round your answer to three decimal places)

y=x^(2)-4, -5 ≤ x ≤ 0

The answer is 32.333 but I have no idea how to get there.

Penny Nom lui répond.
A tangent of the curve (x/a)^n+(y/b)^n =2 2014-04-15
sudhir pose la question :
the equation of tangent of the curve (x/a)^n+(y/b)^n =2. at(a,b) is
Penny Nom lui répond.
The surface area of a circular dome 2014-04-10
Shafiqah pose la question :
Is this a dome's surface area formula??
{{2 × π r × h square units}}
Is the surface area of the floor for the dome is calculate too in this formula?

Thanks for answering. =)

Robert Dawson and Penny Nom lui répond.
Two rectangular prisms 2014-04-10
hannah pose la question :
two rectangular prisms have a combined volume of 432 cubic feet Prism A has half the volume prism b has what are the volumes of prism a and prism
Penny Nom lui répond.
The surface area of a rectangular prism 2014-04-08
Manraj pose la question :
Find the surface area of a rectangular prism with dimensions 4, 5, and 7 units.
Penny Nom lui répond.
A schedule for 6 people 2014-03-29
John pose la question :
How do I set up a schedule where six people are here for ten of twenty days. Arranged in rotating groups of three, so everyone works with everyone else. Everyone works with everyone else at least once and everyone works ten days.
Victoria West lui répond.
Percentage increase of the population of a town 2014-03-28
hunter pose la question :
The population of a certain town in 1984 was 2900 people. The percent increase was 2.5% each year. What is the town's population in 2002?
Penny Nom lui répond.
14 teams divided into two equal divisions 2014-03-18
C pose la question :
We have 14 teams divided into two equal divisions. We want each team to play each team in their division one time and four teams from the other division one time. What is the schedule
Victoria West lui répond.
A circle which is tangent to two perpendicular lines 2014-03-09
MJ pose la question :
I'm a College Student taking up Bachelor of Secondary Education on Math Subject. And I'm struggling for my research about Circles. I done solving the said topic particularly on this question:

"What are the possible equations of a circle being tangent to a pair of perpendicular lines, having the origin as the Point of Intersection and the C (h, k), where h, k ∈ℤ"

But I can't get what would be the process that I must do in order to jive to my idea/goal for that problem.
Please check my idea that the numerical coefficients of the equation is equal to the radius of the circle. Thanks in advance! :)

Penny Nom lui répond.
A schedule for 6 teams over 12 weeks 2014-03-09
Paul pose la question :
So I have a question. I noticed you came up with ways to create schedules and we are having a really hard time making our lineup correctly. Basically we have 6 teams that will play over a 12 week period every Monday night. The time slots are 1 hr each playing at 6, 7 and 8. We would like each team to play 3 games at each time slot over the 12 weeks. Is this something that can actually be accomplished?
Thanks so much for your help!

Eric Venables lui répond.
8 team schedule over 7 days 2014-02-21
Andrea pose la question :
I need to organize 8 teams. Each team has to go through 8 games against another team, but not against the same as before.
Victoria West lui répond.
A 7 person golf schedule 2014-02-21
Claude pose la question :
Golf Holiday: 7 guys, play a 4-some and a 3-some each day for 10 days. Want to have the best balance of each player playing with every other player the same number of times and each player playing in an equal number of 4-somes and 3-somes.
Victoria West lui répond.
Simultaneous fractional equations 2014-02-15
benjamin pose la question :
hi math central. benjamin here. during class, i had problem with this topic. normally i wont have problems with math but this topic i just too hard for me. please help i am having exam and test next week on this topic

here is the question:
using substitution method, solve the simultaneous equation.
(x+1)/(y+2)=0.5

(x-2)/(y-1)=1/3

Penny Nom lui répond.
Can a cylinder be considered a circular prism? 2014-02-08
Davina pose la question :
Can a cylinder be considered a circular prism?
Penny Nom lui répond.
Two nonlinear equations 2014-01-26
Naryn pose la question :
(1÷x) + (1÷y) = (7÷12)
xy = 12

Penny Nom lui répond.
A rectangular shaped property 2014-01-17
Donna pose la question :
If I I have property that is rectangle shaped. I I know the ends are 100 feet how many feet would the sides be to make 5.4 acres
Penny Nom lui répond.
When would one flip the inequality sign? 2014-01-09
Natasha pose la question :
Would one flip the inequality symbol in this equation: (explain why) (-9a)/(-9) > 81
And please explain in what circumstances one would flip the inequality sign
THANKS!

Penny Nom lui répond.
Schedule for a 4 team basketball league 2013-12-08
Sommer pose la question :
I need a schedule for my kids basketball league. There are 4 teams. We will play every Saturday for 4 weeks with 4 games per Saturday. Week 5 will be the playoffs where 1st will play 4th and 2nd will play 3rd. Week 6 will be winners of week 5. Please help!!
Victoria West lui répond.
Problem solving with fractions 2013-12-03
Jennie pose la question :
Paul has 2/3 as many postcards as Shawn. Shawn has 3/5 as many postcards as Tim. If the 3 boys have 280 postcards, how many more postcards does Tim have than Paul?
Penny Nom lui répond.
Base 5 arithmetic 2013-11-27
samuel pose la question :
Good day sir, please i don't understand when you say 4x4=13 in base five? In fact, am always having difficulties in addition, subtraction, division and multiplication of number in the same base other than base ten. Please can you give me one example each with details explainations?
Penny Nom lui répond.
A schedule for 32 golfers 2013-11-22
Scott pose la question :
I would like a schedule for 32 guys playing in 8 foursomes for 13 week. I would like it so that no one plays in the same foursome more than once or twice. Is this possible to do??? Can you let me know at bogeysm@yahoo.com if you can do it and if you will do it.
Victoria West lui répond.
Admiring handblown glass ornaments 2013-11-21
Ruby pose la question :
We are admiring hand blown glass ornaments 5% are unmarketable due to defects what is the probability the 3rd ornament is the first defective one
Penny Nom lui répond.
Scheduling 10 people for 5 rounds 2013-11-20
Keith pose la question :
Using golf as an example, I would like to schedule 10 people for 5 rounds with 2 groups of 4 and 1 twosome each round. I would like to minimze the number of times any one player plays with another. Can you provide an optimal schedule? Thank you for your help.
Victoria West lui répond.
The area of an irregular octagon 2013-11-18
zafar pose la question :
if i have a rectangular octagon then how i can find its area 1. The length of rectangle is 42 and the width is 21 and and the base length is 42-12 =30 i mean the length of base =30
Penny Nom lui répond.
A lever 2013-11-09
Durgesh pose la question :
A farmer uses a 2 m long lever to lift large rocks from the ground of his field . He places a fulcrum 20cm from the rock end of the lever and then stands on the other end, levering out the rock. If the farmer weighs 100 kg , what is the heaviest mass of the ro ck that he can lift using this method? (how to solve)
Penny Nom lui répond.
The popcorn box problem 2013-11-07
Dave pose la question :
We know that calculus can be used to maximise the volume of the tray created when cutting squares from 4-corners of a sheet of card and then folding up.

What I want is to find the sizes of card that lead to integer solutions for the size of the cut-out, the paper size must also be integer. EG 14,32 cutout 3 maximises volume as does 13,48 cutout 3.

I have done this in Excel but would like a general solution and one that does not involve multiples of the first occurence, as 16, 10 cutout 2 is a multiple of 8,5 cutout 1.

Walter Whiteley lui répond.
Substitution type simultaneous equations 2013-11-03
Kayla pose la question :
I am having problems with substitution type simultaneous equations, when the variable you are substituting is a algebraic one:
y=x^2-3x+4 and 3x-2y=1
I have rearranged 3x-2y=1 to get x=(1+2y)/3 but when I substitute this x value into the other equation, I get the wrong answers!
Would appreciate any help! Thank you.

Penny Nom lui répond.
Water flowing out of a tank 2013-11-03
Carolyn pose la question :
The flow of water out of a hole in a tank is known to be proportional to the square root of the height of water above the hole. That is,

dV/dt (proportional to) sq root (h)

The tank has a constant cross-sectional area A, show that the height of water in the tank is given by

h = ((-kt+C)/2)^2

If the tank is 9 metres high, and it takes 5 hours for it to drain from full to half full, how much longer will we have to wait until it is completely empty?

Penny Nom lui répond.
Is-5/-11 a positive rational number? 2013-10-30
tazneem pose la question :
Is-5/-11 a positive rational number
Penny Nom lui répond.
A square inscribed in a circle 2013-10-14
Jenn pose la question :
Hello! I am about to buy a 7'9" round rug, but I want to have it cut down into a square. What's the largest square I can obtain from this? Thank you!
Penny Nom lui répond.
Maximize the volume of a cone 2013-10-09
Conlan pose la question :
Hi I am dong calculus at school and I'm stumped by this question:

A cone has a slant length of 30cm. Calculate the height, h, of the cone if the volume is to be a maximum.

If anyone can help me it would be greatly appreciated.

thanks.

Penny Nom lui répond.
A schedule for 2, 7 team divisions 2013-09-24
Sampson pose la question :
I'm looking to set up a weekly schedule within a 22 week segment of 2 7 team divisions where every team plays every other team in their divison 3 times and have 4 cross over games with 4 of the other 7 teams in the other division. I've yet to figure this out. Thanks for your time.
Victoria West lui répond.
25 people are attending a 5 day meeting. 2013-09-20
Lisa pose la question :
25 people are attending a 5 day meeting. There are 5 tables, with each table seating 5 (or 6). I want each person to sit at a different table every day with completely different people.
Is this possible. Can do I figure this out?
I have each of the people a letter designation from A to Y. Each table have a number from 1 to 5.
Thank you!

Victoria West lui répond.
A rectangular hyperbola 2013-09-19
Soumya pose la question :
Is the graph of an inverse variation a RECTANGULAR HYPERBOLA? If it is, then how can be the equation of a rectangular hyperbola be xy=constant , whereas in the books it is written that the graph of a rectangular hyperbola is- x^2-y^2=a^2?
Chris Fisher lui répond.
The volume of an attic 2013-09-18
David pose la question :
I have small equipment hut that has a small attic space that I want to blow insulation into it.
The attic size is shaped like a triangle that is about 8 feet by 10 feet by 7 inches at the highest point.
I need to estimate the volume in square feet or cubic feet to know how many bags of insulation to blow into the space.
Thanks

Penny Nom lui répond.
Intersection points and diagonals 2013-09-11
Kenneth pose la question :
I was wondering, is there a relationship between the number of sides of a regular n-gon and the number of times its diagonals intersect?
Ie:
4 sides=1 intersect (center, diagonals form an 'X' )
5 sides=5 intersects (diagonals form a star)
6 sides=13 intersects
And so on.

Chris Fisher lui répond.
A schedule for 18 golfers 2013-08-29
Brian pose la question :
After searching the data base which helped me solve at least 3 other issues I have one more un answered question. I have 18 golfers split into 6 threesomes for 4 days of golf. What is my best option for a schedule mixing them all up as much as possible over the course of the 4 days
Victoria West lui répond.
Euclid's Parallel Postulate 2013-08-20
Justin pose la question :
Hello there,

I was wondering is Euclid's Fifth Parallel Postulate of parallel lines never intersecting, undecidable or essentially undecidable?

Thank you so much for any help you can provide!

Robert Dawson lui répond.
Equal ordinate and abscissa 2013-08-15
sonit pose la question :
the slope of tangent to the curve y=(4-x^2)^1/2 at the point, where the ordinate and abscissa are equal, is
Penny Nom lui répond.
Two perpendicular lines 2013-08-15
Hanna pose la question :
The problem says, "Write the equation in standard form for the line that is perpendicular to the graph of y=5x+1 with a y-intercept of 4." I have no clue how to do this. Please help me!
Penny Nom lui répond.
Differentiate x^x - 2^sinx 2013-08-09
tarun pose la question :
derivative of x^x - 2^sinx
Penny Nom lui répond.
16 golfers 2013-07-31
Pat pose la question :
16 golfers...3 nine hole rounds a day...for 4 days. what are the best pairings for the fewest repeats of partners?
Victoria West lui répond.
What is the value of |2((i)^(1/2))|? 2013-07-22
Delilah pose la question :
What is the value of |2((i)^(1/2))| ?
i.e. absolute value of 2 multiplied by square root of i.

Penny Nom lui répond.
10 guys playing 8-9 hole rounds 2013-07-19
Jason pose la question :
I have 10 guys playing 8 9 hole match-play rounds. I need to have everybody play the other 9 men, but need to limit them to only a 9 hole match between them. If we need to go 3-3-4 or 2-4-4 for a better solution, that may be possible.
Victoria West lui répond.
Simultaneous equations 2013-07-10
Warren pose la question :
solve this simultaneous equation:
xy=4
2x+3y=14

Penny Nom lui répond.
Scheduling a 12 person fishing 2013-07-09
Don pose la question :
Greetings Math Central!
I searched the index, but could not find a similar quandary.
Here's my predicament:
I am seeking help in scheduling a 12 person fishing outing.
We have four boats.
I would like to schedule three fishermen to a boat.
We fish 2 1/2 days, split into morning and afternoon sessions.
Thursday afternoon, Friday morning, Friday afternoon, Saturday morning, and Saturday afternoon
We switch boats after lunch, with each fisherman moving to a different boat, with different partners.
I would like to have each fisherman fish with as many others in the group as possible, with a minimum of partner duplication.
I would like to have each fisherman fish in all of the four different boats over the course of the trip at least once.

Would you please suggest a boat / fishing partner schedule that fits my parameters as closely as possible.

Thank you!

Don

Victoria West lui répond.
4 couples golfing 2013-07-06
Brian pose la question :
We have 4 couples going on a 4 day golf vacation playing 4 rounds of golf. I have spent hours trying to set up a schedule that allows the 4 spouse to play together, and then each spouse to play with one of the other spouses (men with women) for the] other 3 rounds.
I would like the foursomes to be different as possible. Also, no-one should RIDE in a cart with the same person more than once.
I am not a math guy so I try to do this by working it out on paper, over and over again. It ain't working!!
If you can help, I am very thankful.
Brian

Victoria West lui répond.
8 golfers playing 8 games 2013-07-03
Johan pose la question :
We have 8 golfers playing 8 games, and we want to schedule them so that they play together equal amount of times, or as close as you can get to that. They will be playing in two four balls each day. Anyone sorted this one out yet, I will really appreciate a solution? Thanks Johan
Victoria West lui répond.
13 golfers playing 5 rounds 2013-07-03
Alan pose la question :
i have 13 golfers playing 5 rounds. want everyone to play together at least once.
Victoria West lui répond.
A league schedule for 6 hockey team 2013-07-02
Giovanni pose la question :
Hello, I try to make a league schedule for 6 hockey team. Each team will play 36 games during the regular season.
I cannot find a proper schedule generator. Can you help me please?
Thank you

Victoria West lui répond.
Fantasy football league 2013-06-18
Chris pose la question :
Hello, I am the commissioner of a fantasy football league. In this week, we play all 17 weeks of the season. It's a 12 team league, and we play each team twice. To do that, we have 5 "double header" weeks, which means you play 2 different teams. I am struggling to come up with a schedule to make this work. Does anyone have a solution?
Victoria West lui répond.
Compound interest 2013-05-10
Kyla pose la question :
I am doing correspondents and cannot figure out how to explain the solution to this question, I do not understand how they came to this answer and need it explained step by step so I can complete the following questions in this unit, please help!!! Calculating compound Interest

Johnny invest $800 that pays 8% compounded quarterly for 5 years. How much is the investment worth at the end of the 5th year?

Penny Nom lui répond.
The rings of a bull's eye 2013-04-28
Math pose la question :
The bull’s eye of a target is a circle 20 cm in diameter. It is surrounded by four rings, each 15 cm wide. Find the area of each ring.
Penny Nom lui répond.
How do i reverse this formula? 2013-04-20
MK pose la question :
I have 22,000,000 worth of gold and wants to want to resell the gold back with a profit. Each transaction of gold will have a transaction fee of 15%, so if i resell the gold back at 22,000,000, I will only get back 18,700,000 after 15% transaction cut. The current formula is X-15%X = Y where X = 22,000,000 and Y = 18,700,000 Will u be able to come up with a new formula if we don't know the value of X but we know the value of Y
Something like this X-15%X = 22,000,000. How do i reverse this formula to find the value of X
Thanks

Penny Nom lui répond.
4 linear equations with 3 unknowns 2013-04-12
Marian pose la question :
how to solve for 3 unknowns in 4 simultaneous equations
Penny Nom lui répond.
A gravel pile in the shape of a triangular pyramid 2013-04-04
Casey pose la question :
Hello
Right now I am stuck and I feel embarrassed because I feel like the answer is so easy I should know it.

I am working on a project and need to find a volume of gravel it will take to occupy this triangular prism like area. I am not sure what formulas I should use whether it be that for the volume of a pyramid or something more complex? Basically it forms a right triangle at one side then from there all points slope to one singular point about 10412mm away.
I am attaching a picture drawn up in paint with the actual dimensions to clear up any confusion.

Thank you for any help. Casey

Penny Nom lui répond.
Question 2013-04-04
Casey pose la question :
Hello
Right now I am stuck and I feel embarrassed because I feel like the answer is so easy I should know it.

I am working on a project and need to find a volume of gravel it will take to occupy this triangular prism like area. I am not sure what formulas I should use whether it be that for the volume of a pyramid or something more complex? Basically it forms a right triangle at one side then from there all points slope to one singular point about 10412mm away.
I am attaching a picture drawn up in paint with the actual dimensions to clear up any confusion.

Thank you for any help. Casey

Penny Nom lui répond.
Simultaneous equations with fractions 2013-03-31
Terence pose la question :
5/x-6/y=1 17/x+30/y=16 I been spending whole day to solve this question. Would be very grateful if you can help I try The denominator value is a equations term which make is simultaneous equations so hard.
Penny Nom lui répond.
Tangents to the curve y = x^3 2013-03-24
Ethan pose la question :
How many tangent lines to the curve y = x^33 pass through the point (2, 4)? For each such line, and the exact coordinates of the point of tangency on the curve.
Penny Nom lui répond.
Cumulative Frequency Math Question 2013-03-20
Primadonna pose la question :

Hi,

Please help me solve this question.

Thank you so much.

The cumulative frequency table below shows the length of time that 30 students spent text messaging on a weekend.

Minutes Used Cumulative Frequency
31-40 2
31-50 5
31-60 10
31-70 19
31-80 30

Which 10-minute interval contains the first quartile?

(1) 31–40

(2) 41–50

(3) 51–60

(4) 61–70


Penny Nom lui répond.
Five players in a 4-player partnership game 2013-03-19
Dee pose la question :
I have 5 players who are playing in a 4-player partnership game & want to rotate the 5th person in in such a way that each person is each other person's partner for a game. How can such a rotation be done?
Penny Nom lui répond.
Six-digit passwords 2013-03-19
Judy pose la question :
Hello:
Can you please explain why the answer to the following question is 10^6? What is the number of possible six-digit passwords when using the digits 0 through 9, with repetition allowed.
Thank you.

Penny Nom lui répond.
Related rates 2013-02-17
Ishaak pose la question :
A hemispherical bowl is filled with water at a uniform rate. When the height of water is h cm the volume is π(rh^2-1/3 h^3 )cm^3, where r s the radius. Find the rate at which the water level is rising when it is half way to the top, given that r = 6 and the bowl fills in 1 minute.
Penny Nom lui répond.
A word problem involving toys 2013-02-14
sandy pose la question :
Each boy gets 5 toys.Each girl gets 3 toys.There are 150 pupils.
The boys had 74 more toys than girls.
How many boys?
How many girls?

Penny Nom lui répond.
Simultaneous equations 2013-02-10
Michael pose la question :
2P + 1/3V =8
3P - 2/V=5

Penny Nom lui répond.
We have 10 golfers and four rounds of golf in two days 2013-02-05
Steve pose la question :
We are having a golf tournament. We have 10 golfers and four rounds of golf in two days. We are going to break the group of 10 into two flights. Flight A and Flight B. We would like all members in Flight A to play one nine hole match against each other. All players in Flight B to play one nine hole match against each other. Could we play in groups of 4,3 and 3? What is the best way to make the pairings work?
Robert Dawson lui répond.
Integration from 0 to 2pi of 1/(3cos x + 2) dx 2013-02-04
ankit pose la question :
Integration from 0 to 2pi of 1/(3cos x + 2) dx
Harley Weston lui répond.
Multiplication in base two 2013-02-01
Michael pose la question :
multiply in the indicated base

110two*11two

Penny Nom lui répond.
The fourth side of an irregular polygon 2013-02-01
Emran pose la question :
I have a irregular polygon. I know 3 of the 4 sides, and 2 of the angles. A-B is 285, B-C is 149, and C-D is 310. Angle B is 135 degrees. and Angle C is 45 degrees. Is there a formula to solve for the final side? Thanks.
Penny Nom lui répond.
The multiplication table for the different bases 2013-02-01
sylvia pose la question :
I am having a difficult time trying to figure out how to fill in the multiplication table for the different bases. i don't know how to get the numbers.
Penny Nom lui répond.
We have 2 teams of 6 couples each that play 6 games 2013-01-25
Denise pose la question :
We put together a game night where we have 2 teams of 6 couples each that play 6 games. We haven't been able to figure out an arrangement that allows each couple to play each game with a different couple from the opposite team (i.e. Team A couples play every game with a different couple from Team B). Is this possible? It works with 2 teams of 5 couples each.
Chris Fisher lui répond.
A railroad embankment 2013-01-22
clint pose la question :
the pennsylvania railroad found it necessary,owing to land slides upon the roadbed,to reduce the angle of inclination of one bank of certain railway cut near pittsburgh,pa.,from an original angle of 45 degrees to a new angle of 30 degrees. the bank as it originally stood was 200 ft.long and had a slant length of 60 ft.. find the amount of the earth removed, if the top level of the bank remained unchanged.
Penny Nom lui répond.
Maximize profit 2013-01-19
Chris pose la question :
A firm has the following total revenue and total cost function.
TR=100x-2x^2
TC=1/3x^3-5x^2+30x
Where x=output
Find the output level to minimize profit and the level of profit achieved at this output.

Penny Nom lui répond.
The weight of a pipe full of water 2013-01-16
joe pose la question :
how do you figure out the weight of a pipe full of water
Harley Weston lui répond.
A large ditch 2013-01-11
steve pose la question :
hi
i have a large ditch to make a road entrance accross, the ditch is a green field on one side and a highway on the other side the ditch depth is 1.8m on the field side and 3.2 on the highway side the culvert pipe is 1,2m in diameter are you with me so far? the entrance from the highway will be 26m wide tapering in to 6m - ,20 m in from the highway there will be a concrete slab of 150mm over the top

the fall from highway to field must not exceed 1/8

any ideas how much stone it will need

Harley Weston lui répond.
A trig identity 2013-01-04
Tehmas pose la question :
Prove sinC+sinD=2sin(C+D/2)cos(C-D/2)
Harley Weston lui répond.
The quadratic formula 2013-01-03
itsel pose la question :
Find the discriminant ans use it to determine the use the quadratic formula to solve the equasion -2x^2+3x+2=0
Penny Nom lui répond.
Mod versus Rem in Turing 2013-01-01
Eric pose la question :
I am a teacher teaching computer science using Turing. I am having difficulty understanding why one would use the mod operator versus the rem remainder operator.

Mod seems to make the resulting sign depend on the sign of the divisor, whereas rem makes the resulting sign depend on the dividend.

Examples:

11 mod 5 = 1 and 11 rem 5 =1
-11 mod 5 = 4 and -11 rem 5 = -1
11 mod -5 = -4 and 11 rem -5 =1
-11 mod -5 = -1 and -11 rem -5 = -1

What I can't understand is why this would matter. For example, -11 / 5 = -2.2 and 11 / -5 = -2.2 get the same result.
So how is a remainder dependent on the sign of one of the parts? What benefit would using one over the other have?

Any insight would be most helpful!

Eric

Harley Weston lui répond.
An integral 2012-12-16
Slavena pose la question :
integration of (lnx)^2 / x dx
Penny Nom lui répond.
An area bounded by lines 2012-12-16
sidra pose la question :
find area bounded by functions:
y=x
y=2x
and y=5-x

Penny Nom lui répond.
A max/min problem 2012-12-14
bailey pose la question :
A right angled triangle OPQ is drawn as shown where O is at (0,0). P is a point on the parabola y = ax – x^2 and Q is on the x-axis.

Show that the maximum possible area for the triangle OPQ is (2a^3)/(27)

Penny Nom lui répond.
The derivative of y = sin (30º + x) 2012-11-07
Saskia pose la question :
derivative of y = sin (30º + x)
Harley Weston lui répond.
birdhouse in the shape of a rectangular prism 2012-10-29
Tiffany pose la question :
Hello, I am Tiff M. from NYC.
I was solving a math problem about adding,subtracting,multiplying or dividing fractions and mixed numbers. I am not getting this question below.
* Laura is building a birdhouse in the shape of a rectangular prism. The base of the birdhouse has an area of 5 square inches. The height of the birdhouse is 2 7/12 inches. Laura calculated the volume of the bird house to be 10 7/12in. Is she correct?
If not, what did she do wrong?

Robert Dawson lui répond.
An implicit differentiation problem 2012-10-26
Katie pose la question :
find y' of x^2y-2y^3=3x+2y
Harley Weston lui répond.
How fast is the distance between the aircraft and the car increasing? 2012-10-24
Steven pose la question :
At a certain instant an aircraft flying due east at 240 miles per hour passes directly over a car traveling due southeast at 60 miles per hour on a straight, level road. If the aircraft is flying at an altitude of .5mile, how fast is the distance between the aircraft and the car increasing 36 seconds after the aircraft passes directly over the car?
Penny Nom lui répond.
Differentiation rules 2012-10-23
Morgan pose la question :
Use the derivative rules to differentiate each of the following:
1. f(x)=1/x-1
2. f(x)= sqrt(x)

Penny Nom lui répond.
Revolutions per second 2012-10-18
Crystal pose la question :
Suppose a car wheel is 26 inches in diameter.
a) If the tire completes one rotation, how far would the car travel?
b) If the car is traveling at 60 miles per hour, how fast is the car wheel spinning in revolutions per second?

Penny Nom lui répond.
A word problem involving a fraction 2012-10-12
Derrick pose la question :
If the numerator and denominator of a fraction are both decreased by 1 the fraction becomes 2/3. If the numerator and denominator are both increased by 1 the fraction will be 3/4. Find the original fraction. How to do?
Penny Nom lui répond.
Changing the sign of an inequality 2012-09-19
Bryauna pose la question :
Why do you change the signs in inequalities?!
Penny Nom lui répond.
An equilateral triangle and a regular hexagon in a circle 2012-09-11
Heidemarie pose la question :
The vertices of an equilateral triangle with side length of 10 sqrt 3 cm lie on a circle. Find the side length of the regular hexagon whose vertices lie on the same circle.
Penny Nom lui répond.
Angular velocity 2012-09-05
Kyra pose la question :
A belt connects a pulley of radius 8cm to a pulley of radius 6cm. Each point on the belt is travelling at 24 cm/sec. Find the angular velocity of each pulley.
Penny Nom lui répond.
A tangent to f(x) = 1/x 2012-09-04
Steven pose la question :
Consider the graph of the function f(x) = 1/x in the first quadrant, and a line tangent to f at a point P where x = k. Find the slop of the line tangent to f at x = k in terms of k and write an equation for the tangent line l in terms of k.
Penny Nom lui répond.
12 golfers, 4 rounds 2012-08-29
Paul pose la question :
We have 12 golfers who will play four rounds of golf - how can we organize the foursomes so that each player has the most exposure to the other 11 players?
Victoria West lui répond.
A schedule for 6 teams and 37 games 2012-08-29
Giovanni pose la question :
Hello, i try to prepare a schedules for 6 teams, and every teams will play 37 games before the playoff. How to make a schedules? thanks
Victoria West lui répond.
A rotating schedule 2012-08-16
Dusty pose la question :
Ok, Im trying to create a rotating schedule. There are four jobs and four people but, one of four people cant preform one job. How can I figure out a rotating schedule for this issue?
Victoria West lui répond.
A volume of revolution 2012-07-15
Tewodros pose la question :
Let f(x) = e^x and g(x) = x^1/2 both be defined on [0,1]. Consider the region bounded by f(x), g(x), x = 0, x = 1. Rotate this region about the y-axis and determine the volume using the shell method.
Harley Weston lui répond.
Scheduling an academic competition 2012-06-12
Mike pose la question :
I need to schedule 18 high school teams to play each other in a "Jeopardy!"-like academic competition. Teams compete against each other 6 at a time (simultaneously) in 3 different rooms. What will the minimum amount of matches (I believe 6 ?) we would have to play to ensure that each team plays every other team exactly twice?

And more importantly: what would that schedule look like?

The best I've come up with is having most teams playing twice, but with 2 pairs of teams having to play either only once or sometimes 3 times.

Thanks in advance for any help you can give! :-)

Mike

Chris Fisher lui répond.
A multinomial theorem 2012-06-08
Anuj pose la question :
if (1+x+xsquare) whole raise to 20 is a0 + a1x + ........ + a20xraise to 40, then find a0 + a2 + a4 + .......... + a38.
How do you solve this???????

Robert Dawson lui répond.
The volume of a wedge 2012-05-28
olwethu pose la question :
volume of wedge that have side 3,3 cm and 6,2 cm and 20 cm long?
Penny Nom lui répond.
Multiples 2012-05-28
Kenneth pose la question :
If I understand correctly , a multiple is a product of two numbers. For example some of the multiples of 6 are 6, 12, 18, 24, 30, etc. I just multiplied 6 by 1, 2, 3, 4, 5, etc.

Are the multiples of a fraction, for example, 2/3, determined in the same way? Are they 2/3, 4/3, 6/3, 8/3, 10/3, etc., or are they instead, 2/3, 4/6, 6/9, 8/12, 10/15, etc.?

Or do fractions have no multiples?

Penny Nom lui répond.
Multiplying binomials 2012-05-14
Jordan pose la question :
How do I solve the equation:
3(x+4)(x-6)

Penny Nom lui répond.
A 25% sulfuric acid solution 2012-05-03
Scott pose la question :
In order to make a 25% solution with 96% Sulfuric Acid, how much of the Sulfuric do I add to DI water to make up 2000ml??
Penny Nom lui répond.
The area of a quadrilateral 2012-04-21
Rajat pose la question :
calculate the area ABCD in which AB is 48'1'',BC is 98'4'',CD is 61'4'',DA is 102'10'',AC is 110'3'',BD is 116'9''.
Penny Nom lui répond.
16 golfers 2012-04-20
steve pose la question :
i have 16 golfers, 4 in "A" flight , 4 in "B" flight , 4 in "C" and 4 in "D"

We are playing 4 rounds (4 teams of 4) and every group must have a player from the A B C an D flight, and in no round may 2 people have already played together!

Victoria West lui répond.
Golf for 12 2012-04-18
Brent pose la question :
I have a golf group that is set up as follows: 12 players, broken into 2 teams of 6. I am looking for a formula to have one player from team A play against one player for team B each round, not repeat the match, and ride with as little duplication as possible. I know that it is not statistically possible with these numbers and will have at least one round that has duplication.
Robert Dawson lui répond.
Two cars approach a right-angled intersection 2012-04-10
Michael pose la question :
Two cars approach a right-angled intersection, one traveling south a 40km/h and the other west at 70km/h. When the faster car is 4km from the intersection and the other case if 3km from the intersection, how fast is the distance between the car cars changing?
Penny Nom lui répond.
Golf for 28 2012-04-10
EARL pose la question :
HI, I HAVE 28 GOLFERS AND I NEED A SCHEDULE FOR 5 DIFFERENT DAYS THERE WILL BE 7 GROUPS OF 4 PLAYERS EACH DAY. ALL PLAYERS WOULD LIKE TO PLAY WITH ONE ANOTHER ONCE AND NOT PLAY WITH EACH MORE THAN ONCE IF POSSIBLE OR LIMIT THE TIMES A GOLFER PLAYS WITH ANOTHER TO A MINIMUM IF POSSIBLE. (PLEASE SEND AN EXAMPLE OF THE SCHEDULING OF THE GROUPS FOR THE TEE OFF ORDER)
THANKS FOR ANY SOLUTIONS YOU CAN PROVIDE.
EARL

Robert Dawson lui répond.
A semi-circular roof gutter 2012-04-09
Kyle pose la question :
A semi-circular roof guttering contains some water.
The cross-section of the guttering has a diameter of 10 centimetres. The surface of the water collected in the guttering is 8cm. What is the depth of water in the gutter?

Penny Nom lui répond.
A maximization problem 2012-04-09
Nancy pose la question :
After an injection, the concentration of drug in a muscle varies according to a function of time, f(t). Suppose that t is measured in hours and f(t)=e^-0.02t - e^-0.42t. Determine the time when the maximum concentration of drug occurs.
Penny Nom lui répond.
The spread of a rumor 2012-04-09
Roohi pose la question :
The function f(t) = a/(1+3e^(-bt)) has also been used to model the spread of a rumor. Suppose that a= 70 and b=3 0.2. Compute f(2), the percentage of the population that has heard the rumor after 2 hours. Compute f'(2) and describe what it represents. Compute lim t approaches infinity and describe what it represents.
Penny Nom lui répond.
A rectangular prism 2012-04-05
Tori pose la question :
How do you find the base height if the base width is 10ft. The height is 7ft. And the volume is105ft.
Penny Nom lui répond.
Golf for 17 2012-03-29
Richard pose la question :
Hi Guys,
I have a scheduling problem which I don't think you have covered before.
Apologies if you have!
I have 17 golfers due to play 4 rounds of golf.
Each round will consist of 3 threeballs and 2 fourballs (ie. 17 golfers in 5 groups)!
Is it possible to come up with a schedule where each golfer plays
with different partners in each round?

Robert Dawson lui répond.
The period T of a pendulum 2012-03-27
Ashley pose la question :
The period T of a pendulum is given in terms of its length, l, by T=2pi sqrt(l/g) where g is the acceleration due to gravity(a constant)
a. find dT/dl
b. what is the sign of dT/dl
c. what does the sign of dT/dl tell you about the period of the pendulums?

Penny Nom lui répond.
The area of an irregular hexagon 2012-03-20
Jacob pose la question :
I am having trouble finding the area of an irregular hexagon with 5 right angles. It has 5 given sides, with the lengths as follows:

8.2 ft.
6 ft.
4.5 ft.
2.2 ft.
6 and one third ft.

Chris Fisher lui répond.
The volume of an irregular tetrahedron 2012-02-22
Brittany pose la question :
If I am given an irregular tetrahedron with the coordinates of the 4 points how do I find the volume? For example I am asked to find the volume and only given the points A(-4,-3,5), B(2,-1,2), C(0,-5,0), and D(-2,0,0) can you shown me the working and formular to find the volume?
Robert Dawson lui répond.
Dt[sin t tan (t^2+1)] 2012-02-21
Ayu pose la question :
Ayu
Dt[sin t tan (t^2+1)]
derivatives

Harley Weston lui répond.
36 golfers 2012-02-15
Steve pose la question :
We will have 36 golfers playing 4 rounds of golf and would like to have different foursomes each round. In other words, no player will play with another more than once.
Harley Weston lui répond.
The derivative of x^-(1/2) 2012-01-14
Eric pose la question :
I have an problem figuring out the derivative of the negative square root of x i.e. x^-(1/2) using the first principle.
Could someone please show me?
Thanks in advance!

Harley Weston lui répond.
Lost in the woods 2012-01-12
Liz pose la question :
I am lost in the woods. I believe that I am in the woods 3 miles from a straight road. My car is located 6 miles down the road. I can walk 2miles/hour in the woods and 4 miles/hour along the road. To minimize the time needed to walk to my car, what point on the road should i walk to?
Harley Weston lui répond.
A volume of revolution 2012-01-11
john pose la question :
find volume of solid generated by revolving the region in the first quadrant bounded by the curve y squared=x cubed, the line x=4 and the x-axis about the line y=8. The answer in the back of the book is 704 pi divided by5
Penny Nom lui répond.
Triangular numbers that are square 2011-12-27
Luke pose la question :
The “TnS” is a positive integer which is both Triangular number and Square number. For example, 36 is a “TnS” number since 36=1+2+3+4+5+6+7+8 (Triangular number) and 36=6x6 (Square number). What is the next “TnS” which is greater than 36?
Penny Nom lui répond.
Three sides of a triangle 2011-12-24
saba pose la question :
the three sides of a triangular lot have lengths 10,11and 13cm,respectively. find the measure of its largest angle and the area of the lot?
Penny Nom lui répond.
Water is flowing into a cup 2011-12-19
Tim pose la question :
A cup has a radius of 2" at the bottom and 6" on the top. It is 10" high. 4 Minutes ago, water started pouring at 10 cubic " per minute. How fast was the water level rising 4 minutes ago? How fast is the water level rising now? What will the rate be when the glass is full?
Penny Nom lui répond.
A cube of ice is melting 2011-12-05
Emily pose la question :
a cube of ice (i.e.) each side is of the same length) is melting at a rate such that the length of each side is decreasing at a rate of 5cm per hour. how fast is the volume of the cube decreasing (in cubic cm per hour) at the instant the length of each side is 25cm?
Penny Nom lui répond.
The average age of 300 cars 2011-11-22
Jason pose la question :
I have 300 used cars built from 1970 to 1980, I would like to know the average age by total volume. If I provide the qty built per year, can I determine the average age of the lot (300) by total volume?
Total built 50 56 30 10 30 25 64 9 12 14
Year built 1971 1972 1973 1974 1975 1976 1977 1978 1979 1980

Thanks

Penny Nom lui répond.
Water pouring into a conical tank 2011-11-21
Patience pose la question :
Hi my name is patience and I'm having a problem with this question.
Water pours into a conical tank of semi vertical angle 30 degrees at the rate of 4 cm^3/s, where h is the depth of the water at time t. At what rate is the water rising in the tank when h = 10 cm?
Thank you

Penny Nom lui répond.
I started with Euler's identity and manipulated it 2011-11-14
anonymous pose la question :
I started with Euler's identity and manipulated it
e^i*pi=-1
e^-i*pi=(-1)^-1
e^-i*pi=-1
e^-i*i*pi=(-1)^i
e^--pi=(-1)^i
e^pi=(-1)^i
type it in in a calculator and you get e^pi=23.1406926... and (-1)^i=0.0432139183... What did I do wrong?

Robert Dawson lui répond.
Lines tangent to y^2=4x 2011-11-11
Reuchen pose la question :
Find equations of the lines tangent to y^2=4x and containing (-2,1).
Penny Nom lui répond.
Modular arithmetic 2011-10-30
Kim pose la question :
Hello,
I am editing a resource for students, and I think some of the answers may be incorrect. The text I was given and my questions are in the attachment. Any help you could give would be appreciated.
Thanks,

Kim

Harley Weston lui répond.
A mythical soccer ball 2011-10-27
Joel pose la question :
We've been working on this problem diligently and can't seem to come up with the answer book's answer. We think it may be wrong, yet want to check it with an expert. Here goes. The school's new soccer balls are covered with 64 regular hexagonal panels. Each hexagon measures 2 inches between opposite corners and 1.5 inches between opposite sides. What is the total surface area of the soccer ball?
Robert Dawson and Lorraine Dame lui répond.
A spherical ball in a conical wine glass 2011-10-26
Jules pose la question :
A heavy spherical ball is lowered carefully into a full conical wine glass whose depth is h and whose generating angle (between the axis and a generator) is w. Show that the greatest overflow occurs when the radius of the ball is (h*sin(w))/(sin(w)+cos(2w)).
Claude Tardif lui répond.
Implicit differentiation 2011-10-20
Monica pose la question :
Find dy/dx in terms of x and y, if sin(xy)=(x^2)-y.
Penny Nom lui répond.
A hemispherical bowl with a lead ball inside 2011-09-27
Jean pose la question :
"(a) Water is being poured into a hemispherical bowl of radius 3 inch at the rate of 1 inch^3/s. How fast is the water level rising when the water is 1 inch deep ?

(b) In (a), suppose that the bowl contains a lead ball 2 inch in diameter, and find how fast the water level is rising when the ball is half submerged."

Penny Nom lui répond.
The derivative of f(x) = (x+1)^1/2 2011-09-05
Carla pose la question :
Find the derivative using the limit process of
f(x) = (x+1)^1/2

Harley Weston lui répond.
4 x 2 2011-08-30
Lucjan pose la question :
how to sketch a diagram that shows 4x2 expression?
Penny Nom lui répond.
Romeo throws a pebble at Juliet's wondow 2011-08-22
Natalie pose la question :
There is a picture of Romeo trying to attract Juliet's attention without her nurse who is in a downstairs room, noticing. he stands 10m from the house and lobs a small pebble at her bedroom window (3.5m high). Romeo throws the pebble from a height of 1m with a speed of 11.5m/s at an angle of 60degrees to the horizontal. I have already found that it take 1.74seconds to reach the window and that it does in fact hit Juliet's window however i cannot work out the speed of the pebble when it hits the window! The answer is 9.12m/s but I cannot reach this answer. Hope you can help me :)
Robert Dawson lui répond.
3.8 x 2.2 2011-08-18
Isaac pose la question :
Question from isaac, a student: I am struggling in math & I need help on multiplying 3.8 x 2.2. Can you helpme
Melanie Tyrer and Harley Weston lui répond.
The height of a fluid in a horizontal tank 2011-07-24
jason pose la question :
Same set up as many others, cylindrical tank on its side, but I am interested in defining the change in volume and/or fluid level as a function of time at a constant volumetric outflow. I plan on hooking a pump to the tank so "gpms' will be constant. I have a couple different sized tanks and pumps so I want a general equation. Thanks for your help.
Harley Weston lui répond.
A regular hexagon inscribed in a circle 2011-07-18
Courtney pose la question :
If ABCDEF is a regular hexagon inscribed in a circle of radius r, prove that the length of each side of the hexagon equals r.
Penny Nom lui répond.
Golf for 14 2011-07-11
Doug pose la question :
We have 14 players playing over 4 days. can we play with different players every day and keep the 3 x foursomes and 1 x pair format
Victoria West lui répond.
Golf for 12 2011-07-07
Jim pose la question :
I have a golf trip scheduled with a total of 12 guys. We are playing three rounds of golf. For round 1 Players A B C D must play together. Is there a formula that would allow me to set up the other 8 foursomes with no repeats or minimal repeats? Thanks.
Victoria West lui répond.
A line tangent to f(x)=1/x 2011-06-05
Michael pose la question :
A line tangent to f(x)=1/x in the first quadrant creates a right triangle with legs the x-axis and the y-axis. Prove that this triangle is always 2 square units regardless of where the point of tangency is.
Penny Nom lui répond.
16 golfers and 8 rounds 2011-05-26
Gerry pose la question :
16 golfers and 8 rounds. To have each player play with as many others as possible.
Victoria West lui répond.
The length of a belt around three pulleys 2011-05-18
Grant pose la question :
I need to calculate the belt length around these pulleys, please can you help or refer me?

Known variables
D - Large Pulley Diameter
d - Small Pulley Diameter
c - Center Distance between D and d
T - Tension Pulley Diameter
x - Horizontal Distance between T and d' Centers
y - Vertical Distance between T and d's Centers
I need to calculate the belt length around these pulleys.


Kind Regards,
Grant

Harley Weston lui répond.
The volume of a triangular shaped aquarium 2011-05-07
Jeff pose la question :
HI, I searched the site but really couldn't find my answer, at least that I could understand! I have a triangular shaped aquarium ( think of a cube, except its an equal triangle shape). all 3 sides are 48 inches, and it is 34" tall. Can you please tell me what the volume is in gallons? Thanks!
Penny Nom lui répond.
Solve x(x-3) = 5 (x+4) 2011-04-19
Ashley pose la question :
I have to solve using the quadratic formula. And my answer should be in simplest exact form. And I can't approximate.
I really need help to prepare for a test ....
x(x-3) = 5 (x+4)

Penny Nom lui répond.
An antiderivative of the square root of (8t + 3) 2011-04-19
Caitlyn pose la question :
I know how to take an antiderivative. But this one's stumping me. I need it to finish a problem. What's the antiderivative of the square root of (8t + 3)

~Caitlyn=

Penny Nom lui répond.
The volume of a triangular pyramid 2011-04-15
Chelsea pose la question :
There is a triangular pyramid which is an equilateral on the base with all sides 16.5 and the height is 25. I have to find the volume. Please help!
Penny Nom lui répond.
2 cubic foot bags of mulch 2011-04-11
Ron pose la question :
if I buy 2 cubic foot bags of mulch, how bags would I need to equal 1 yard? (13 bags?)
Penny Nom lui répond.
Eliminate y 2011-04-07
Lynn pose la question :
2x + y = 8
y + 3z =5
z + 2w =1
5w + 3x = 9

Form three equations with y eliminated

Penny Nom lui répond.
Tennis schedule 2011-04-04
Toni pose la question :
I need to create a doubles tennis schedule for 6 players playing over a 32 week schedule where no player repeats with a partner.
Victoria West lui répond.
Designing a tin can 2011-03-31
Tina pose la question :
A tin can is to have a given capacity. Find the ratio of the height to diameter if the amount of tin ( total surface area) is a minimum.
Penny Nom lui répond.
[(90+36-4) ÷ 2] x 15 = 2011-03-30
ken pose la question :
[(90+36-4) ÷ 2] x 15 =
Penny Nom lui répond.
A stone is dropped into a lake 2011-03-24
AnneMarie pose la question :
A stone is dropped into a lake, creating a circular ripple that travels outward at a speed of 25 cm/s. Find the rate at which the area within the circle is increasing after 4s.
Penny Nom lui répond.
If 25 items are evenly spread over 7 days 2011-03-23
michael pose la question :
What is the formula to calculate:
If 25 items are evenly spread over 7 days, but one day has 3 items less than the other days, what is the number if items for each of the remaining 6 days?

Penny Nom lui répond.
What is x to the power of 0? 2011-03-23
Jason pose la question :
What is x to the power of 0?
Robert Dawson lui répond.
Regular and irregular polygons 2011-03-10
thaera pose la question :
the difference between regular shape and irregular shapes.
Penny Nom lui répond.
Vertices and sides of a polygon 2011-03-04
RINA pose la question :
My questions is : would a vertices and sides of a polygon be the same number.  For example, a octagon it has 8 points and 8 sides. Am i correct?
Robert Dawson lui répond.
At what rate is the grain pouring from the chute? 2011-02-26
MJ pose la question :
Suppose that grain pouring from a chute forms a conical heap in such a way that the height is always 2/3 the radius of the base. At the moment when the conical heap is 3 m high, its height is rising at the rate of 1/2 m/min. At what rate (in m^3/min) is the grain pouring from the chute?
Penny Nom lui répond.
Slicing a rectangular parallelepiped 2011-02-23
tom pose la question :
a) Pass a plane containing a vertex of a rectangular parallelepiped and a diagonal of a face not containing that vertex to cut a pyramid from the parallelepiped. What fraction of the volume of the parallelepiped is the volume of the pyramid thus cut off?

b)Pass a plane through a cube of edge 8 in so that the section formed will be a regular hexagon. Through each side of the hexagon pass two planes, one plane containing one of the two vertices of the cube which are farthest away from the plane of the hexagon, the other plane containing the diagonally opposite vertex of the cube. Find the volume of the solid bounded by these planes.

c)three of the edges of a rectangular parallelepiped that meet in a point are also the lateral edges of a pyramid. What fraction of the parallelepiped is this pyramid...?

you know the my only problem is how to illustrate this one...if only i could ...can you help me....all I need is the drawing and I can do the rest ..it's fine if you'll only answer any of these but I'll be thankful if you will do the three :)....

Penny Nom lui répond.
Prove sin x = sin (pi - x) 2011-02-15
Janet pose la question :
Prove sin x = sin (pi - x)
Penny Nom lui répond.
An irregular hexagon 2011-02-09
Emma pose la question :
Hello, I have math question about how to find the area of an irregular hexagon.
It is shaped like a backwards L and the side lengths are as follows
11cm
8cm
5cm
5cm
6 cm
3cm
Can you please help me find the area of this?
Emma

Penny Nom lui répond.
Multiplication and order 2011-01-21
Janet pose la question :
IN A MULTIPLICATION SENTENCE BASED ON A DRAWING..IS THERE ANY ORDER FOR THE MULTIPLICAND ANG MULTIPLIER FOR THE ANSWER TO RIGHT?
Robert Dawson lui répond.
Integrating ln^3x/x 2011-01-14
ken pose la question :
y=ln^3x/x from x=1 to x=11
Penny Nom lui répond.
Angular speed 2010-12-12
Jason pose la question :
7 in. pulley traveling @ 175ft/sec. What is the rpm?
Stephen La Rocque and Penny Nom lui répond.
The length is twice its breadth 2010-12-09
jenu pose la question :
The length of the rectangular field is twice its breadth. A man jogged around it 5 times and covered a distance of 3 km. what is the length of the field?
Penny Nom lui répond.
A rectangular solid 2010-12-08
ryan pose la question :
Good day! Have a question on rectangular solids. We were asked to find the dimensions of a rectangular solid, we were only given the volume of the rectangular solid, the total area, and the altitude. I tried using the volume of the rectangular solid and was able to get the area of the base, but I don't know how to get its length and its width. Hope you could help me again. Thanks!
Penny Nom lui répond.
Simultaneous equations 2010-12-05
ryan pose la question :

Question from ryan, a student:

3         4
--   -    --      =     1      (1)
x         y

7       2             11
--   -  --        =   --       (2)
x       y              12


Chris Fisher and Stephen La Rocque lui répond.
Angular speed 2010-12-03
Julie pose la question :
If the angular speed of a circle is 360 degrees divided by the time it took to rotate, how do I calculate ones that have greater rotations than the amount of time? I mean, you calculate the angular speed of a merry-go-around that spins once every 5 seconds by doing 360degrees/5 sec, but how do you calculate a merry-go-around that has 16 rotations every 4 seconds?
Penny Nom lui répond.
Can determine if it is scalene, isosceles, or equilateral 2010-12-01
Jessie pose la question :
find the measures of the sides of triangle KPL and classify each triangle by its sides. my first problem would be K(-3,2) P(2,1) L(-2,-3) ...The three points they give you are the vertices of the triangle and you need to match them up. Draw the triangle and write in the vertices and the related point with the vertex. You will then do the distance formula three times to find the distance of all three sides. Once you have the three sides you can determine if it is scalene, isosceles, or equilateral...using the distance formula how do i solve this?
Penny Nom lui répond.
A man made circular lake 2010-11-25
ailish pose la question :
A man made circular lake has a diameter of 338 m. A bridge is to be constructed across the lake in such a way that it is 119 m away from the center of the lake. How long is the bridge?
Penny Nom lui répond.
Two girls agree to mow a lawn 2010-11-23
Milorad pose la question :
Two girls agree to mow a lawn 60m by 80m so that the first girl cuts one half by cutting a uniform strip around the lawn. How wide a strip should the first girl cut?

I've been struggling with the question above, mainly with the equation and I have set it up as 2400=(60-2x)(80-2x), if this is incorrect I would like to know where I've gone wrong and also would like a full answer to see it properly done. I need an algebraic answer and not one with a diagram.

Penny Nom lui répond.
i^i 2010-11-21
trale pose la question :
Can we use e^ix=cosx+isinx for finding i^i like that: x= pi/2 => e^(ipi/2)=0+i then [e^(ipi/2)]^i=i^i.then we find i^i= 0,207879576.... is it true? can we give value for x for free?thank you.
Harley Weston lui répond.
Convert to multiplication 2010-10-23
angelina pose la question :
how to covert 5+5+5=15 to multiplication
Penny Nom lui répond.
An exponential equation 2010-10-19
Sandi pose la question :

Question from sandi, a student:

I have several questions similar to this one and was wondering if you could walk me through this one. I'm totally lost on how to do it.
Paramecia reproduce by splitting in two. In a laboratory flask, a colony of paramecia had an initial population of 500. Each day, the population of the paramecia was counted. The results are as listed.
Time (in days)------Population
0-----------------------500
1-----------------------600
2-----------------------720
3-----------------------864
4----------------------1037
5---------------------1244
6---------------------1493
7----------------------1792
8---------------------2150
1.)Using graphing calculator make a scatter plot of the data in table.
I think I did this part right I set my window at Xmin=0 Xmax=10 Xscl=1 Ymin=0 Ymax=2500 Yscl=100 Xres=1
2.) Determine an exponential equation to represent the population as a function of time without using a graphing calculator.I have no clue how to do this.
3.)Suppose the flask and food supply is large enough to support the trend of the population growth. Estimate the population of the colony when the time is 10 days.


Penny Nom lui répond.
Geocaching 2010-10-15
Clint pose la question :
I am stuck on a puzzle for Geocaching which is a GPS game. I know it is a number matrix but don't know how to start.

[1 -7 16 -3 -9; 1 -3 18 -20 -33] x [1 8 -9 3 5; 0 1 0 4 2; 0 0 1 2 1; 0 0 0 1 -1; 0 0 0 0 1] = ?

Stephen La Rocque lui répond.
What is the maximum weekly profit? 2010-10-10
Joe pose la question :
A local artist sells her portraits at the Eaton Mall. Each portrait sells for $20 and she sells an average of 30 per week. In order to increase her revenue, she wants to raise her price. But she will lose one sale for every dollar increase in price. If expenses are $10 per portrait, what price should be set to maximize the weekly profits? What is the maximum weekly profit?
Stephen La Rocque and Penny Nom lui répond.
x/200+x/400+x/600+x/800 2010-10-08
Ashishthombre pose la question :
step by step LCM of x/200+x/400+x/600+x/800
Penny Nom lui répond.
Rectangular arrays 2010-09-29
Kim pose la question :
My 4th grader has started working on rectangular array that shows 2 factors of a number.

The problem reads on centimeter grid paper draw as many arrays as you can for each of the #'s 2,3,4, 5,6,7,,11,12,15 and 16. That would be great except I have no idea what they are talking about and I cannot offer any help to my daughter.

Penny Nom lui répond.
A Taylor polynomial for (lnx)/x 2010-09-29
Dave pose la question :
I have a series problem that I cannot solve. The problem asks for you to compute a Taylor polynomial Tn(x) for f(x) = (lnx)/x. I calculated this poly out to T5(x) and attempted to use this to identify a pattern and create a series in order to calculate Tn(x). However, the coefficients on the numerator out to F5prime(x) are as follows: 1, -3, 11, -50, 274... Ok, so the negative is an easy fix -> (-1)^n-1. But the other coefficients are stumping me. I can't see any sort of pattern there and I've tried every trick I know. Is there another way to go about this? Thanks!
Chris Fisher lui répond.
limit as x approaches a of ((x^(1/2))-(a^(1/2)))/(x-a)? 2010-09-29
emily pose la question :
limit as x approaches a of ((x^(1/2))-(a^(1/2)))/(x-a)?
Penny Nom lui répond.
May Lee's cake 2010-09-18
Wafa pose la question :
May Lee bought a cake which is circular in shape. Her sister ate a quarter of the cake. Given that the area of the top surface of the remaining portion is 520 square cm, find the diameter of the cake.
Penny Nom lui répond.
Continuity 2010-09-18
Carina pose la question :
Hi. My name's Carina and I'm currently a sophomore in high school. I'm having a lot of difficulties in AP Calculus with continuity, one-sided limits, and removable discontinuities. Basically, I have no idea how to do them or even what they are. I read the lesson but I still don't get it. Can someone put it in simpler terms so I can understand how to complete my questions? Thank you!
Robert Dawson lui répond.
Two perpendicular chords 2010-09-11
edwin pose la question :
two perpendicular chords AB and CD intersect at P. if X,Y are their midpoints and M the centre of the circle, prove that MP=XY. I do not a have clue on how to do it so can you please help me with it
Robert Dawson lui répond.
Maximizing the volume of a cylinder 2010-08-31
Haris pose la question :
question: the cylinder below is to be made with 3000cm^2 of sheet metal. the aim of this assignment is to determine the dimensions (r and h) that would give the maximum volume. how do i do this? i have no idea. can you please send me a step-to-step guide on how t do this? thank you very much.
Penny Nom lui répond.
Three cars 2010-08-30
Anil pose la question :
3 cars are moving at speed of 4 kmph,5.5 kmph and 8 kmph in a circular track.The circular track has a distance of 11 km. What is the time taken for all the 3 cars to meet at the starting point ?
Stephen La Rocque lui répond.
A max min problem 2010-08-19
Mark pose la question :
a rectangular field is to be enclosed and divided into four equal lots by fences parallel to one of the side. A total of 10000 meters of fence are available .Find the area of the largest field that can be enclosed.
Penny Nom lui répond.
Two cars on a circular track 2010-08-12
david pose la question :
If two cars starts to drive at point 1 and go in a circle when will they meet? (one cars speed is faster the other ) all I need is a formula
Tyler Wood lui répond.
70 cubic meters of mulch 2010-07-30
zsuzsi pose la question :
i have 70 cubic meters of mulch to move and an 8 tonne truck to use how much tonnage is to 1 square meter so that i can work out how many trips i need to make
Penny Nom lui répond.
The area of a hexagon 2010-07-28
Katie pose la question :
Hi i have to find the area of a hexagon with only the side length 2cm given would you be able t help?
Walter Whiteley lui répond.
An 18ft round pool 2010-07-19
christine pose la question :
will an 18ft round pool fit on a 12x16 ft rectangular cement slab?
Tyler Wood lui répond.
777777 times 111111 2010-07-07
Chew pose la question :
What is 777777 times 111111 without using a calculator?
Chris Fisher lui répond.
Maximize the floor area 2010-07-07
shirlyn pose la question :
A rectangular building will be constructed on a lot in the form of a right triangle with legs of 60 ft. and 80 ft. If the building has one side along the hypotenuse, find its dimensions for maximum floor area.
Penny Nom lui répond.
(x^3 + 11x) is divisible by 6 2010-06-24
PT pose la question :
Given that x is a non-zero integer, how do you show that for all values of x, (x3 + 11x) is divisible by 6?

I know it works but how do I answer the "all values of x" part?

Thanks in advance!

Robert Dawson lui répond.
A max/min problem 2010-06-12
valentin pose la question :
What is the maximum area of an isosceles triangle with two side lengths equal to 5 and one side length equal to 2x, where 0 ≤ x ≤ 5?
Harley Weston lui répond.
Compound interest 2010-06-01
Kenneth pose la question :
Hello: I have a question regarding the following compound interest and future value calculation.

Year 1 P + rP equals balance after the first year.
Year 2 (P + rP) + r(P + rP) equals balance after the second year.
Year 3 ? equals balance after the third year.

This question is in two parts.

1. What would follow for year three?

2. I know that a pattern will develop. What will it be so that I can determine the extended pattern for following years ?

Penny Nom lui répond.
Integration of sin^3 (2x) 2010-05-29
ascher pose la question :
how do you integrate this equation ∫ sin^3 (2x) dx
Robert Dawson and Penny Nom lui répond.
Weighing a 1000 lb cylinder on a 500 lb scale 2010-05-28
Jerry pose la question :
I am trying to weigh a large possibly 1000 lb cylinder on a 500 lb scale, is this possible? If so what is the actual formula to figure out the correct weight? I am told it is possible with the fulcrum method but have not been able to find a formula. Thanks Jerry
Robert Dawson lui répond.
Two problems 2010-05-27
debbie pose la question :

Question from debbie, a parent:

hi, i have a daughter and she asked me a maths question I cannot solve. I was just wondering if you can give me the answers plus the working out so I could explain to my daughter,

1. The leftmost digit of a six-digit number N is 1. If this digit is removed and then written as a rightmost digit, the number thus obtained is three times N .Find N.

2. Four friends are racing together down a flight of stairs. A goes 2 steps at a time, B 3 steps at a time. C 4 steps at a time and D 5 steps at a time. The only steps which all four tread on are the top one and the bottom one. How many stairs in the flight were stepped on exactly once?


Penny Nom lui répond.
An optimization problem 2010-05-23
Marina pose la question :
Hello, I have an optimization homework assignment and this question has me stumped..I don't even know

A hiker finds herself in a forest 2 km from a long straight road. She wants to walk to her cabin 10 km away and also 2 km from the road. She can walk 8km/hr on the road but only 3km/hr in the forest. She decides to walk thru the forest to the road, along the road, and again thru the forest to her cabin. What angle theta would minimize the total time required for her to reach her cabin?
I'll do my best to copy the diagram here:

                             10km
Hiker_ _ _ _ _ _ _ _ _ _ _ _ _ _ _ _ _ _ _ _ _Cabin
      \                           |                              /
       \                          |                             /
     f  \                      2km                          /
         \                        |                           /
theta   \___________________________ /
                            Road


Penny Nom lui répond.
The rejection region 2010-05-01
Mong pose la question :
In Tests of Hypothesis, what is the decision when the test statistic we compute from the sample information is exactly equal to the critical value? that is, we reject null hypothesis or do not reject it? Thanks!
Chris Fisher lui répond.
The rate of change of y with respect to x 2010-04-29
Tom pose la question :
I just had a quick calc question about wording that wasn't ever addressed in class. When the book says "the rate of change of y with respect to x", should it be considered how fast y is changing in comparison to x?

I ask because the textbook says that "y is changing 3 times faster than x, so the rate of change of y with respect to x is 3." I'm use to rate being like velocity, as in units of distance per units of time. All we're told in class is that it's the slope of the tangent line, I was hoping you could clarify for me what exactly is meant by the wording of a "rate of change of something with respect to something else". More specifically, what "rate" and "with respect to" mean within this context?

Thanks for your time

Harley Weston lui répond.
A trig equation 2010-04-28
Steve pose la question :
2cos^2(X)-2sin^2(x)+1=0
Harley Weston lui répond.
cos(x) = sin(x - 1) 2010-04-28
alex pose la question :
In the equation cos x = sin x-1 for -pi/2 A: solve for x graphically
B: solve algebraically and prove the solution is correct.

Alex

Penny Nom lui répond.
y = - log(x) 2010-04-28
Alex pose la question :
y= - log(x), where y = 4.3
solve for x.

Harley Weston lui répond.
A rectangular prism 2010-04-27
Savannah pose la question :
A rectangular prism is made of exactly 8 cubes. Find the length, width, and height.
Penny Nom lui répond.
A rectangular garden 2010-04-25
Billy pose la question :
Tanisha wants to make a rectangular garden with a perimeter of 38 feet. What is the greatest area possible that tanisha can make the garden?
Penny Nom lui répond.
If (x, 4) is equidistant from (5, -2) and (3, 4), find x. 2010-04-21
abeth pose la question :
If (x, 4) is equidistant from (5, -2) and (3, 4), find x.
Find the point on the y - axis that is equidistant from (-4, -2) and (3, 1).

Penny Nom lui répond.
What are the dimensions of the box? 2010-04-13
Steven pose la question :
The longest rod that will just fit inside a rectangular box, if placed diagonally top to bottom, is 17 inches. The box is 1 inch shorter and 3 inches longer than it is wide. How much must you cut off the rod so that it will lie flat in the bottom of the container? What are the dimensions of the box?
Robert Dawson lui répond.
Integrate the ((4th root of x^3)+1) dx 2010-04-12
Bridget pose la question :
integrate the ((4th root of x^3)+1) dx
Tyler Wood lui répond.
The derivative of y=x^x 2010-04-09
David pose la question :
So, its David, and I was wondering about the derivative of y=x^x. I have often seen it be shown as x^x(ln(x)+1), but when I did it through limits it turned out differently. Here's what I did:
It is commonly know that df(x)/dx of a function is also the limit as h->0 of f(x+h)-f(x)/h. To do this for x^x you have to start with lim h->0 ((x+h)^(x+h)-x^x)/h. The binomial theorem then shows us that this is equal to lim h->0 (x^(x+h)+(x+h)x^(x+h-1)h+...-x^x)/h
This is also equal to lim a->0 lim h->0 (x^(x+a)+(x+h)x^(x+h-1)h...-x^x)/h.
Evaluating for a=0 you get lim h->0 (x^x+(x+h)x^(x+h-1)h...x^x)/h
Seeing as the last 2 terms on the numerator cancel out you can simplify to a numerator with h's is each of the terms, which you can then divide by h to get:
lim h->0 (x+h)x^(x+h-1)... which when evaluated for h=0 gives us: x(x^(x-1)). This statement is also equal to x^x.
This contradicts the definition of the derivative of x^x that is commonly shown. So, my question is: can you find any flaws in the logic of that procedure? I do not want to be shown how to differentiate x^x implicitly because I already know how to do that.

Robert Dawson lui répond.
A model for the population of a Canadian city 2010-04-07
jess pose la question :
Let t^2 represent t-squared. The population of a Canadian City is modelled by: f(t) = 12t^2 + 800t + 40,000, where t is the time in years. When t= 0, the year is 2007.
a) According to the model, what will the population be in 2010?
B) In what year is the population predicted to be 300,000?

Penny Nom lui répond.
A max min problem 2010-04-06
Terry pose la question :
The vertex of a right circular cone and the circular edge of its base lie on the surface of a sphere with a radius of 2m. Find the dimensions of the cone of maximum volume that can be inscribed in the sphere.
Harley Weston lui répond.
The derivative of cos^3x 2010-04-06
Erson pose la question :
Find y' of the given function: y = cos^3x.
Harley Weston lui répond.
Sand falling off a conveyer 2010-04-02
Katherine pose la question :
sand is falling off a conveyer onto a pile at the rate of 1.5 cubic feet per minute. The diameter of the base is approximately twice the altitude. At what rate is the height of the pile changing when it is 10 feet high?
Penny Nom lui répond.
Pyramids and parallelograms 2010-04-01
Vivi pose la question :
Hi, my name is Vivi, and math teacher is giving a geometry test. I tend to forget how to find the surface area and volume of triangular pyramids and parallelograms. Do you have a way of remembering how to find the surface area?
Harley Weston lui répond.
A champagne pyramid 2010-03-22
Kathy pose la question :
I have 680 champagne glasses and my pyramid can only be 15 levels high. How many glasses need to be on each level? Is there a formula I can use?
Robert Dawson lui répond.
Water in a culvert 2010-03-18
Chip pose la question :
Hello, I have a problem to which I know there must be an analytical solution - but as it has been 50 years since I studied math I can't remember quite how to do it!! I have a 12" round culvert in my yard that runs water all year. I would like to be able to calculate the flow through the culvert by measuring the depth of water flowing through and measuring the speed of the current. I can measure the depth and the speed, but I forget how to calculate the cross section of the flow. If you could help I would greatly appreciate it. Thank you
Penny Nom lui répond.
Angular velocity 2010-03-17
Lindsay pose la question :
Determine the angular velocity, in radians per second, of a 14-inch car wheel, if the car is traveling at 60 miles per hour
Harley Weston lui répond.
An irregular octagon 2010-03-09
Gayle pose la question :
Question from Gayle:

I am building an irregular shaped octagon wooden box.
The measurements are 291/2 inches by 211/2 inches.
Sides are 12 inches.
It will be 36 inches high.

What would the cutting angles degrees be?

Harley Weston lui répond.
0.999 ^ (500) 2010-03-07
debra pose la question :
I just need to know how to solve the following problem without using a calculator: .999 ^ (500). I know the answer is .606, I just want to do it by hand since I can't use a calculator on my test.
Penny Nom and Claude Tardif lui répond.
The integral of X^3/the square root of 1-x^2 dx 2010-03-07
William pose la question :
The integral of X^3/the square root of 1-x^2 dx.
Harley Weston lui répond.
The base area of rectangular prism 2010-03-05
Desiree pose la question :
How do you find the base area of rectangular prism that has 4 cm by 4cm by 6cm ?
Penny Nom lui répond.
1/6 x 5 x 12 = what 2010-02-19
dana pose la question :
1/6 x 5 x 12 = what
Lorraine Dame lui répond.
The center of a rectangular room 2010-02-16
Diana pose la question :
Consider a rectangular room, 15 feet wide, 30 feet long and 12 feet high. What is the exact distance from any of the 8 corners of the room and its geometric center? Can you write a generic formula for such a distance? And if we keep the same proportions on all dimensions, can you write an expression for the same distance as a function of the floor perimeter?
Penny Nom lui répond.
Two bus routes 2010-02-03
kiyah pose la question :
from 4:30 pm to 6:30 pm the route 1 bus stops every 12 min at the gym's bus stop. the route 2 bus stops there every 15 min. if both buses are now at the stop and schedule is kept, how long will it be before both buses will be at the stop again?
Penny Nom lui répond.
Everything in a cube 2 miles long? 2010-01-31
Naresh pose la question :
in a book, i got to read this :

Is it possible to pack the entire population of earth and everything that was created by humankind in a cube whose edges are 2 miles long ?

Is it possible? Thanks.

Stephen La Rocque lui répond.
The inradius of an icosahedron 2010-01-21
fnavroth pose la question :
Suppose you have an equilateral triangle. The area of the triangle is exactly 1200 square centimetres. Now suppose you have twenty of those triangles. It's possible to assemble those twenty triangles into a closed three-dimensional shape, a regular polyhedron. What would be the volume, in cubic centimetres, of the largest sphere that could fit inside the shape?
Chris Fisher lui répond.
Loading a headboard in a uhaul 2010-01-20
gina pose la question :
I have a uhaul 9'10" long 4"9"wide 4'7"height I have a headboard 74 inches in height would it fit the box diagonally
Penny Nom lui répond.
A cone circumscribed about a given hemisphere 2010-01-19
Neven pose la question :
The cone of smallest possible volume is circumscribed about a given hemisphere. What is the ratio of its height to the diameter of its base?
(G.F.Simmons, Calculus with Analytic Geometry, CH4 Applications of Derivatives)

Chris Fisher lui répond.
A schedule for a pool tournament 2010-01-14
john pose la question :
I need to know how to set up a pool tournament 12-14 players playing once a week, either 10-12 weeks whichever works out the best, what players would play what players week one through week ten or twelve? I hope you can help, I cant figure out the formula.

thanks

John

Victoria West lui répond.
A triangular prism 2010-01-13
Breon pose la question :
what is the formula for a triangular prism?
Robert Dawson lui répond.
Trigonometry and picture hanging 2010-01-13
george pose la question :
The top of a picture 1m high 0.8m from the ceiling. At a point on the ceiling directly in front of the picture, we wish to install a light so that the angle subtended by the picture equals to the angle of depression of the top of the picture. How far out from the wall should the light be installed?
Penny Nom lui répond.
Related Rates Problem 2010-01-12
Neven pose la question :
A woman raises a bucket of cement to a platform 40 ft above her head by means of a rope 80 ft long that passes over a pulley on the platform. If she holds her end of the rope firmly at head level and walks away at 5ft/s, how fast is the bucket rising when she is 30 ft away from the spot directly below the pulley? (G. F. Simmons, Calculus with Analytic Geometry, pg.142)
Penny Nom lui répond.
A pair of simultaneous equations 2010-01-09
Yumiko pose la question :
Solve the following pair of simultaneous equations.

x^2 -4x = y^2-4
3y=2x - 3

Penny Nom lui répond.
A sports schedule for the ymca 2010-01-08
Erich pose la question :
i have to make a sports schedule for the ymca. i have 7 teams. they need to play each other 2 times. we have 3 time slots per night. i would like them to have an even number of time slots thru the season. one team must have abye each week. can you chow mw the schedul. example time 1 team 1 vs. team 2. time 2 team 3 vs. team 4 time 3 team 5 vs. team 6. team 7 bye week 1.
Victoria West lui répond.
A networking event 2010-01-07
Tricia pose la question :
I have 70 people to go around to 10 tables with 7 at each table. only 6 from each table will rotate each time. There is 1 table monitor that always stays at same table. How do I make the rotation so everyone moves and gets to see all 70 people at the networking event?
Robert Dawson lui répond.
Dividing seashells among girls 2010-01-06
Eileen pose la question :
A group of girls collected some seashells from the beach. They tried to divided these seashells equally among them. If each girl received 8 seashells, they would need 5 more seashells. If each girl received 7 seashells, they would have 3 seashells extra. How many seashells did they collect from the beach altogether?
Robert Dawson and Penny Nom lui répond.
A triangular pyramid 2010-01-05
ally pose la question :
how many vertices, edges, faces in a triangular pyramid?
Robert Dawson lui répond.
Figure out the square root of something 2009-12-20
eric pose la question :
is there an ez equation to figure out the square root of something?
Harley Weston lui répond.
A pile of sand 2009-12-16
Malik pose la question :
Sand is leaking out of a hole at the bottom of a container at a rate of 90cm3/min. As it leaks out, it forms a pile in the shape of a right circular cone whose base is 30cm below the bottom of the container. The base radius is increasing at a rate of 6mm/min. If, at the instant that 600cm3 have leaked out, the radius is 12cm, find the amount of leakage when the pile touches the bottom of the container.
Harley Weston lui répond.
Energy in calories 2009-12-15
Josephine pose la question :
A soft drink manufacturer claims that a new diet soft drink is now "low Joule". The label indicates that the available energy per serving is 6300 J. What is the equivalent of this energy in calories? (1 Calorie=1000 cal)
Robert Dawson lui répond.
f(x)=x+2sinx 2009-12-12
amroziz pose la question :
for which values of x does the graph of f(x)=x+2sinx have horizontal tangent
Harley Weston lui répond.
How fast is the distance between the two cars decreasing? 2009-12-08
Jenny pose la question :
Two cares are on a collision course toward point P. The paths of the two cars make a 30 degree angle with each other. The first car is 40 km from P, and traveling toward P at 16 km/hour. The second car is 50 km from P, traveling at 20 km/hour. How fast is the (straight line) distance between the two cars decreasing. (Hint: Law of Cosines)
Harley Weston lui répond.
Solving two equations, one with a square root 2009-11-23
kacie pose la question :
y = square root of x+3
x-4y = -7

im having trouble with this problem...i have to find where they intersect.

Harley Weston lui répond.
The triangle formed by the tangent and the coordinates axes 2009-11-23
Nirmala pose la question :
Given that y=1/x, x is not equal to zero. Prove that the area of the triangle formed by the tangent and the coordinates axes is 2.
Harley Weston lui répond.
Angular speed 2009-11-19
Atalie pose la question :
The engine of a sports car rotates at 5000 revolution per minute. Calculate the angular speed of the engine in radians per second, given that 2pi radians = 1 revolution
Robert Dawson lui répond.
A triangular shaped ditch 2009-11-12
Tim pose la question :
I am planning on constructing a triangular shaped ditch the measures 550 meters in length, 6 meters wide at the surface, .3048 meters at the invert of the ditch and 2 meters deep.

How many cubic meters of material will have to be excavated to construct this ditch

Robert Dawson lui répond.
f(x)= (e^x) / [(e^x)+(ex^2)] 2009-11-10
natalie pose la question :
I'm trying to graph the function, f(x)= (e^x) / (e^x)+(ex^2) [e to the x divided by e to the x plus e times x squared] I know that there aren't any vertical asymptotes, but is there a horizontal asymptote? and also, I'm stuck on finding the concavity for this graph. I tried to find f "(x), but it came out to be really long and I am not sure how to find the x values for f "(x) without using a graphic calculator. thanks, natalie
Chris Fisher and Harley Weston lui répond.
At what rate are the people moving apart? 2009-11-01
saira pose la question :
A man starts walking north at 4 ft/s from a point P. 5 minutes later a woman starts walking south at 5 ft/s from a point 500 ft due east of P. At what rate are the people moving apart 15 minute after the woman starts walking ?
Harley Weston lui répond.
A path around a pond 2009-10-31
adeniji pose la question :
find the area of a concrete path 2m wide surrounding a circular pond 12m in diameter
Penny Nom lui répond.
Painting a dome 2009-10-30
Jessica pose la question :
A hemispherical dome with a radius of 50 ft will be given a coat of paint .01 inch thick. The Contractor for the job wants to estimate the number of gallons of paint needed. Use a differential to obtain an estimate (231 cubic inches/gallon) HINT: Approximate the change in volume of hemisphere corresponding to increase of .01 inch in the radius.
Robert Dawson lui répond.
Acreage calculation 2009-10-29
Angela pose la question :
I am appraising a small portion of property for a right of way for a new sewer line. I have appraised many similar projects in the past; however the engineer always calculated the exact acreage and/or dimensions of the parcel being taken. My survey states the following: "141 linear ft. of 25' permanent R/W. I divided the linear feet x 12 and mult. by 25 to get the square footage, but it just doesn't seem right. Any suggestions? Am I missing something?
Robert Dawson lui répond.
An irregular quadrilateral 2009-10-29
riya pose la question :
what is irregular quadrilateral?
Penny Nom lui répond.
Is it a rectangle? 2009-10-29
sefora pose la question :
The coordinates of the vertices of quadrilaterals are given. Draw each quadrilateral on a grid. Determine whether it is a rectangle (show your work)

P(5,1), Q(-4,4), R(-6,-2), S(3,-5)

Penny Nom lui répond.
Graphing y=(4-x^2)^5 2009-10-25
natalie pose la question :
I want to graph the curve of y=(4-x^2)^5 without using a graphing calculator. To do this, I'm suppose to find: domain, y and x intercepts, asymptotes, intervals of increase/decrease, local max/min, concavity and points of inflection. I got all the way to the step where I'm solving the concavity and I'm stuck. I found the f"(x) and it came out to be really large polynomial. I want to know how I can solve for the x of f"(x) without the use of a graphing calculator, when the polynomial has x^6 and x^8. Thank you so much, natalie
Harley Weston lui répond.
A max/min problem 2009-10-12
avien pose la question :
a rectangle has a line of fixed length Lreaching from the vertex to the midpoint of one of the far sides. what is the maximum possible area of such a rectangle? SHOW SOLUTION USING CALCULUS
Penny Nom lui répond.
Golf for 8 2009-10-10
Chris pose la question :
I have a group of 8 golfers who will be playing three rounds, What is the best formula so we can all try to play with different people during the 3 rounds .
Victoria West lui répond.
24 golfers, 6 days 2009-10-03
patrick pose la question :
i need a pairing schedule for 24 golfers for six days playing in foursomes. Please
Victoria West lui répond.
A line tangent to a parabola 2009-10-01
kanchan pose la question :
for what value of c a line y=mx+c touches a parabola y^2=4a(x-a)
Penny Nom lui répond.
24 golfers 2009-10-01
Peter pose la question :
I am working on a 19 week golf schedule with 24 golfers, and I would like to know how to make up six foursomes each week without having a player paired with the same golfers.
Victoria West lui répond.
5 x 8 + 6 divided 6 - 12 x 2 2009-09-24
Susan pose la question :
5 x 8 + 6 divided 6 - 12 x 2. I am not sure of the rules of operation for this type of question
Penny Nom lui répond.
24 golfers: 4 days 2009-09-21
Steven pose la question :
We have 24 golfers (6 A's, 6 B's, 6 C's & 6 D's) playing 4 days. Each day a foursome consist of an A, a B, a C & a D player. Can you give me a pairing list so that no two golfers will play on the same foursome for the 4 days?
Chris Fisher and Victoria West lui répond.
Dart teams 2009-09-18
Sally pose la question :
I have 14 dart teams. Each team has a home bar.Usually it is one week home, the next week away. I have 3 locations that have one board only, yet have 2 teams at each location. I have one location that has 2 dart boards, and 3 teams. The last location have 3 dart boards and 4 teams.Can you make me a schedule that will be home and away, and every teams plays each other twice. In case you are wondering, the bars sponsor the teams.Thank you. Sally
Robert Dawson lui répond.
solve integral of ( x^2+x+1)^5 2009-09-18
jaka pose la question :
solve integral of ( x^2+x+1)^5
Robert Dawson lui répond.
Common multiples of 36 and 48 2009-09-15
Kamaldeep pose la question :
Find the first 2 common multiples of 36 and 48.
Penny Nom lui répond.
Jogging and swimming 2009-09-11
Brandon pose la question :
Trevor swims jogs 3 days and swims every 4 days. How often does he jog and swim on the same day
Leeanne Boehm lui répond.
A circular border around a pool 2009-09-08
Calvin pose la question :
A pool in the shape of a circle measures 10 feet across. One cubic yard of concrete is to be used to create a circular border of uniform width around the pool. If the border is to have a depth of 3 inches, how wide will the border be? ( 1 cubic yard=27 cubic feet )
Stephen La Rocque lui répond.
4 couples seating themselves at a round table 2009-09-04
Hui pose la question :
Q: 4 couples seating themselves at a round table. Men must seat together and women seat together. How many ways are there? Answer in the printed text is 576. I got 144. Pls advice.
Claude Tardif lui répond.
An area problem 2009-08-30
Amber pose la question :
Area of hexagon ABCDEF with the vertices (3,0) (7,0) (9,2) (7,4) (3,4) (1,0)
Harley Weston lui répond.
Simultaneous equations 2009-08-28
onias pose la question :
solve 3/a - 2/b = 1/2 , 5/a + 3/b = 29/12
Robert Dawson lui répond.
A football schedule 2009-08-14
Bill pose la question :
I am having trouble with a football schedule that has two divisions of five teams each, 1-5 and 6-10. 1-5 will play against each other in the first 4 weeks and also the the last 4 weeks with teams 6-10 completing the middle of the schedule. The schedule is 13 weeks. Thank you for your time
Chris Fisher and Victoria West lui répond.
An antiderivative problem 2009-08-13
Indrajit pose la question :
∫4e^x + 6e^-x/(9e^x + 4e^-x)dx = Ax + Bloge(9e2x - 4) + C

then A=?......B=?.....C=?

plz solve it...."^" stands for "to the power of"....

Harley Weston lui répond.
A rectangular pen 2009-08-13
Kari pose la question :
A rectangular pen is to be built using a total of 800 ft of fencing. Part of this fencing will be used to build a fence across the middle of the rectangle (the rectangle is 2 squares fused together so if you can please picture it). Find the length and width that will give a rectangle with maximum total area.
Stephen La Rocque lui répond.
Angular speed 2009-08-04
Mary pose la question :
The engine of a sports car rotates at 5000 revolutions per minute (rpm). Calculate the angular speed of the engine in radians per second. Use 2 radians = 1 revolution.
Penny Nom lui répond.
Torricelli's trumpet 2009-07-29
Gary pose la question :
I was reading about torricelli's trumpet which is described by the equation1/x which is then rotated around the x axis which results in a figure which looks like a trumpet. Now in order to find the volume the integral 1/x^2 dx is used which diverges when integrated so the volume is finite.However if you integrate 1/x dx which is the formula on the plane the answer diverges. Now if you took an infinite area then rotated it around the x axis shouldn't you get an infinite volume? Notice the area I am talking about is under the line 1/x not the surface area of the trumpet which is what the painters paradox is about What am I missing? Thanks
Robert Dawson lui répond.
Golfing with an unpopular golfer 2009-07-23
Ian pose la question :
"We are a group of 8 golfers, one of whom is unpopular. How can we construct a schedule, of two foursomes, so that each person is scheduled to play with him the same number of times. What does the week by week schedule look like?"
Robert Dawson and Victoria West lui répond.
Annular sector 2009-07-20
Ed pose la question :
What is the name for the section of a flat ring (annulus)? Similar to a section on a radar screen, a quadrilateral but the top and bottom are curved.
Robert Dawson lui répond.
A golf league with 12 players 2009-07-17
Jane pose la question :
My weekly golf league has 12 players in 4 threesomes. How many weeks would it take to play everyone once and not have too many duplications? What would be the schedule each week? Thanks
Victoria West lui répond.
Finding the Line Joining Perpendicular Feet 2009-07-16
mukulu pose la question :
Please help I've been tryng to search in your data but i failed to get the solution Find the equation of a straight line joining the feet of the perpendiculars drawn from the point A(1,1) to the line 3x-3y-4= 0 and 3x+y-6=0.
Janice Cotcher lui répond.
The surface area of a rectangular pyramid 2009-07-13
Shivani pose la question :
What is the surface area of a rectangular pyramid with a length of 6 inches, a width of 7 inches and a SLANT height of 4 inches.
Penny Nom lui répond.
Multiplying Large Numbers 2009-07-09
Jessica pose la question :
What is 12 billion X 6 trillion?
Janice Cotcher lui répond.
Multiplying Large Numbers 2009-07-09
Jessica pose la question :
What is 12 billion X 6 trillion?
Janice Cotcher lui répond.
Simultaneous Equations 2009-07-06
Mukulu pose la question :
Solve the equation simultaneously X/5=(Y+2)/2= (Z-1)/4 ……………….eqt 1 3X+4Y+2Z-25=0 ………………eqt 2
Janice Cotcher lui répond.
Investing in multiple accounts 2009-06-26
Kenneth pose la question :
Hello:

If an investor has $1000.00 to invest in multiple accounts, and he wants a total return of 4%, is there one calculation that can be used to determine what these amounts could be even though there may be numerous amounts used as answers for most of the following examples?

For example,
Invest $1000.00 @ 2% and 5% for total return of 4%.
Invest $1000.00 @ 2%, 3% and 5% for total return of 4%.
Invest $1000.00 @ 2%, 3%, and 5% for total return of 4%.
Invest $1000.00 @ 2%, 3%, 4% and 5% for total return of 4%.
etc.

Robert Dawson lui répond.
The surface area of a tank 2009-06-25
Charles pose la question :
I need to calculate the external wall and top sqft surface area of a tank 54ft od x 15ft high with a dome top 54ft od x 8ft high.
Penny Nom lui répond.
(570 divided by 15) multiplied by (5 plus 26) 2009-06-24
Sherrie pose la question :
What is the answer to this math question:
(570 divided by 15) multiplied by (5 plus 26)..........
Thank you,
Sherrie:-)

Penny Nom lui répond.
The Pythagorean theorem 2009-06-24
supreet pose la question :
What are some real-world applications of the Pythagorean theorem?
and
How are the Pythagorean theorem and the distance formula related?

Harley Weston lui répond.
Two questions from math class 2009-06-18
Con pose la question :
Hello,

My name is Con and my son is required to answer the following questions for his maths class.

He has attempted Q1 through trial and error and has found the answer to 72453. Is this correct?

He has attempted to draw the triangles described in Q2 in a number of ways and has found that BE can not equal ED and is dependent of angle BAC. Therefore, he claims that the triangle can not be drawn/practical. Is this correct or is there a slolution?

Q1.
Digits 2, 3, 4, 5 and 7 are each used once to compose a 5-digit number abcde such that 4 divides a 3-digit number abc, 5 divides a 3-digit number bcd and 3 divides a 3-digit number cde. Find the 5-digit number abcde.

Q2.
Let ABC be a triangle with AB=AC. D is a point on AC such that BC=BD. E is a point on AB such that BE = ED = AD. Find the size of the angle EAD. Con

Chris Fisher lui répond.
The integral of x^x 2009-06-18
ANGIKAR pose la question :
what would be the integration of (X^Xdx)?

give answer in details.

Robert Dawson and Harley Weston lui répond.
Triangular Numbers 2009-06-16
Chinonyerem pose la question :
Question from Chinonyerem, a student:

Each of the numbers
1 = 1, 3 = 1+2, 6 = 1+2+3, 10 = 1+2+3+4 ,...
represents the number of dots that can be arranged evenly in an equilateral
triangle:
                            .
                  .        . .
        .         .       . . .             ...
.      . .      . . .    . . . .
This led the ancient Greeks to call a number TRIANGULAR if it is the
sum of consecutive integers, beginning with 1. Prove the following facts
concerning triangular numbers:
(a) A number is triangular if and only if it is of the form n(n+1)/2 for some n >= 1
(b) The integer n is a triangular number if and only if 8n+1 is a perfect square
(c) The sum of any two consecutive triangular numbers is a perfect square
(d) If n is a triangular number, then so are 9n+1, 25n+3, and 49n+6
Penny Nom lui répond.

11 golfers playing 4 rounds 2009-06-14
Brian pose la question :
I have 11 golfers playing 4 rounds of golf. It would be great if we could play at least once with everybody. I realize we will have 2 foursomes and 1 threesome each round...can you help?
Victoria West lui répond.
The product of gradients between 2 perpendiculars lines 2009-06-11
Alister pose la question :
how do i prove that the product of gradients between 2 perpendiculars lines equal to -1....
Penny Nom lui répond.
A rotating schedule 2009-06-10
Doreen pose la question :
We have seven people - we want to create a rotating schedule for two people at a time to attend one day a week with each person working the same amount of days in the year.
Victoria West lui répond.
Cubic yards in an Irregular Shape 2009-06-08
ron pose la question :
hi i have a berm 100'w x200'L at the base and 9'tall the top is 85'L x 20'w i would to find out how many cubic yards are in the pile and the formula
Janice Cotcher lui répond.
An aeroplane flying in a wind 2009-06-02
Maria pose la question :
The aeroplane flies in a wind of speed 50 kph from the direction S80◦W. However, the direction in which it is pointed, and its speed in still air, are such that its resultant speed and direction are 800 kph, and direction N71.6◦E

I need to find the size of the angle between v and vw, in degrees to one decimal place, and find the speed at which the aeroplane would fly if the air were still, to the nearest kph.

It would help me greatly if I could see a diagram illustrating the velocity va that the aeroplane would have if the air were still, the velocity vw of the wind, and the resultant velocity v of the aeroplane (diagram should be in the form of a triangle illustrating how one velocity vector is the sum of the other two)

Thanks

Harley Weston lui répond.
The position of the fulcrum 2009-05-23
jim pose la question :
I think I need a formula. I need to know how far an object will be lifted. A beam is 246 inches long on one side of the fulcrum, and 41 inches on the other side, if I push down 36 inches on the long side of the beam, how much will the short side move up?
Stephen La Rocque lui répond.
28 teams 14 lanes , 9 weeks no repeats 2009-05-19
paul pose la question :
28 teams 14 lanes , 9 weeks no repeats
Karen Meagher lui répond.
Fraction Word Problem 2009-05-13
Sonya pose la question :
Debbie wants to eat 1/6 of her 12 pieces of candy. How many pieces did she eat ?
Janice Cotcher lui répond.
16 teams, 15 weeks, would like to pair up with no repeats. 2009-05-12
Kimberly pose la question :
16 teams, 15 weeks, would like to pair up with no repeats. Thank you so very much.
Robert Dawson lui répond.
differentiate y sin[x^2]=x sin[y^2] 2009-05-11
mamiriri pose la question :
derivate y sin[x^2]=x sin[y^2]
Harley Weston lui répond.
Six teams, two sites, and four days 2009-05-08
John pose la question :
I have six teams, two sites, and four days. We have decided that we want to have tri-meets, where there would be three teams competing against each other at the same site each day. Here is the problem, I need to make sure that each team sees each other team at least once and each team goes to each site at least once.
Victoria West lui répond.
The dimensions of a toy chest 2009-05-07
charlotta pose la question :
My son needs to find the 3 dimensions of a toy chest that has to have a volume of 24 cubic feet can you show how to get the answer
Penny Nom lui répond.
Perpendicular lines 2009-05-07
shabnam pose la question :
the line presented by y= 3x-2 and a line perpendicular to it intersect at R(1,1). Determine the equation of the perpendicular line
Stephen La Rocque lui répond.
A 6 team social softball league 2009-05-02
Don pose la question :
Hi We have a 6 team social softball league with only two fields. Each team plays two games each Saturday. We have three time slots 10:00, 12:00 and 2:00 We play for 11 Saturdays and than have a 2 weekend playoff. All teams prefer double headers vs the split at 10:00and 2:00. How do we schedule as balanced a schedule as possible and minmize the splits ? Thanks Don
Laura Morrison and Victoria West lui répond.
Rectangular prisms 2009-05-01
deborah pose la question :
Could you please tell me some examples of different objects in the real world of rectangular prisms?
Harley Weston lui répond.
The integral of a to power x squared 2009-04-28
JIM pose la question :
WHEN I ATTENDED U.OF T. (TORONTO ) MANY YEARS AGO WE WERE TOLD THE FOLLOWING INTEGRAL COULD NOT BE SOLVED : a to power x squared . is this still true ?

CURIOUS , JIM

Robert Dawson lui répond.
Completing the square 2009-04-27
Daniel pose la question :
I tried following an example of yours but I still couldn't figure it out. Here is my question,

2x^2 - 6x + 1 = 0.

Any help would be greatly appreciated.

Stephen La Rocque lui répond.
Common multiples of 2 and 5 2009-04-23
pat pose la question :
what are the common multiples of 2 and 5, through 30, because i been working on it for hours
Robert Dawson and Stephen La Rocque lui répond.
Two regular hexagons 2009-04-21
niko pose la question :
Two regular hexagons are ____________ similar.
Stephen La Rocque lui répond.
Volume of a prism 2009-04-21
David pose la question :
The base of a rectangular prism has an area of 15.3 square inches and a volume of 185.13 cubic inches. Write an equation that can be used to find the height "h" of the prism
Robert Dawson and Stephen La Rocque lui répond.
A perpendicular line in standard form 2009-04-21
Kristy pose la question :
Can you help me with this equation? Find the equation, in standard form of the line perpendicular to 2x-3y=-5 and passing through (3,-2) With the equation in standard form with all integer coefficient.
Stephen La Rocque lui répond.
A max-min problem 2009-04-20
Charlene pose la question :
A fixed circle lies in the plane. A triangle is drawn inside the circle with all three vertices on the circle and two of the vertices at the ends of a diameter. Where should the third vertex lie to maximize the perimeter of the triangle?
Penny Nom lui répond.
28 golfers 2009-04-18
DON pose la question :
HI, I HAVE 28 GOLFERS AND I NEED A SCHEDULE FOR 24 DIFFERENT DAYS THERE WILL BE 7 GROUPS OF 4 PLAYERS EACH DAY. ALL PLAYERS WOULD LIKE TO PLAY WITH ONE ANOTHER AS EVEN AS POSSIBLE (SAY 2 TO 4 TIMES EACH OVER THE 24 DAYS)...THANKS DON
Victoria West lui répond.
Exponential form 2009-04-16
Pete pose la question :
Hi, How do you express ³√h^-4 in exponential form. I am having a lot of trouble with this one.
thanks
Pete

Stephen La Rocque lui répond.
10 yards of rubber mulch 2009-04-15
Donna pose la question :
If I want to buy 10 yards of rubber mulch (bulk), but it only comes in 20 pound bags, how many bags do I need to buy?
Harley Weston lui répond.
Infinity and Aleph-Null 2009-04-14
Justin pose la question :
Yes, I am reading the Paul Halmos book on Set theory, thanks for telling me how to get it! I was just wondering from your last answer though if the positive real infinity of calculus then corresponds to Aleph-null? I am sorry if this is a similar question to the one I asked before but I was just wondering about this!

All the Best,

Justin

Robert Dawson lui répond.
Choices at a restaurant 2009-04-13
Rob pose la question :
There is a restaurant you get:

Rice/Noodles (1) Main Ingredient (any) Sauce (1)
1 1 1
2 2 2
3 3 3
4 4 4
5 5 5
6 6 6
7 7  
  8  
  9  
  10  
  11  
  12  

So the question is how many different combinations are there. You can only have 1 rice/noodles in a selection and only 1 sauce in a selection but you can have between 1 and all twelve mains in a selection. there are 7 rice/noodles , 12 mains and 6 sauces. How many possibilies. I did it mentally in the restuarant, no pen, paper or calculater and i got 3276..i think thats wrong. please help

Rob
Robert Dawson, Stephen La Rocque and Claude Tardif lui répond.

2sinB=3tanA 2009-04-10
Xanathax pose la question :
ABC is a right-angled triangle. 2sinB=3tanA. Calculate the measure of angle A.
Penny Nom lui répond.
Winding paper after a break 2009-04-10
Olen pose la question :
Question from Olen:

I work in a paper mill and have been handed the task to search for a formula to determine how much paper needs to be added to a parent roll to make up the difference at the winder. (Ex. The spool diameter at the reel is 18.25" we measure roughly 33.5" to make two 58" rolls in the winder. If the is a paper break and the roll diameter in the winder is 30" how much do I add to a single parent roll (22" roughly) to make one 58 " and the 28" needed at the winder. I would appreciate any help to complete this task. I would like to be able to build a chart that operators can refer to based on what is needed. Thank you.

Harley Weston lui répond.
Positive real infinity and Aleph-null 2009-04-09
Justin pose la question :
Hello, I was just wondering why does the positive real infinity correspond to Aleph-null? Thanks a lot for answering my question!

Justin

Ami and Robert Dawson lui répond.
Parallel and perpendicular lines 2009-04-08
Blake pose la question :
I need to know what the formula for working a problem. I need to know how to find if i line is parallel or perpendicular by only looking at the equation. i know you must know the slope but how do i find slope?
Stephen La Rocque lui répond.
Multiplication of polynomials 2009-04-07
Carla pose la question :
I am struggling to understand Multiplication of Polynomials. No matter how hard I try to understand Multiplication of Polynomials, I just can't get it!

The problem that I am trying to solve is this :

-3x^3y(-y + 2 -x^2 + x)

Robert Dawson lui répond.
Sand falls from a conveyor belt 2009-04-01
Tracy pose la question :
Sand falls from a conveyor belt at the rate of 10 cubic feet per minute onto a conical pile. The radius of the base is always equal to half the pile's height. How fast is the height growing when the pile is 5ft high?
Stephen La Rocque lui répond.
A spherical Tootsie Roll Pop 2009-04-01
Tracy pose la question :
A spherical Tootsie Roll Pop you are sucking on is giving up volume at a steady rate of .8 ml/min. How fast will the radius be decreasing when the Tootsie Roll Pop is 20 mm across?
Harley Weston lui répond.
An isosceles triangle 2009-03-26
sela pose la question :
An isosceles triangle has two equal sides of length 10 cm. Theta is the angle between two equal sides.
a) Express area of a triangle as a function of theta
b) If theta is increasing at a rate of 10 degrees/minute, how fast is area changing at the instant theta=pi/3?
c) at what value of theta will the triangle have the maximum area?

Penny Nom lui répond.
A max-min problem 2009-03-24
Jay pose la question :
Determine the area of the largest rectangle that can be inscribed between the x-axis and the curve defined by y = 26 - x^2.
Harley Weston lui répond.
The diameter of a roll of plastic 2009-03-24
truong pose la question :
hi. i have trouble to calculate the diameter of the plastic roll. the sheet is 765 m long and 0.8 mm to wrap around the core 400 mm in dia. please help me with the formula to calculate the dia of plastic roll, thanks in advance
Harley Weston lui répond.
A rectangular open box 2009-03-21
tina pose la question :
An open box is to be made from a rectangular piece of tin by cutting two inches squares out of the corners and folding up the sides. The volume of the box will be 100 cubic inches. Find the dimensions of the rectangular piece of tin.
Stephen La Rocque lui répond.
A dead fly is stuck to a belt that passes over two pulleys 2009-03-21
Jules pose la question :
A dead fly is stuck to a belt that passes over two pulleys 6 inches and 8 inches in radius. Assuming no slippage,how fast is the fly moving when the large pulley (8 inches) makes 21 revolutions per second ? How many revolutions per second does the small pulley make ? How long will it take the dead fly to travel 1 mile ?
Penny Nom lui répond.
16 golfers 2009-03-18
Bill pose la question :
We have 16 golfers. We golf for 5 days, in foursomes. It should work out that each player plays with each other player 1 time exactly.
I just can't figure it out.

Thank you in advance!

Bill

Claude Tardif and Victoria West lui répond.
A 14 team league that is divided into 2 divisions 2009-03-07
Ronnie pose la question :
I have a 14 team league that is divided into 2 divisions. We have a total of 18 weeks to play. I would like for each team to play the team in their division twice. The remaining 6 weeks would be played with the other division. Any help would be appreciated.
Chris Fisher and Victoria West lui répond.
A common tangent to two curves 2009-03-02
Jay pose la question :
For what values of a and b will the parabola y = x^2 + ax + b be tangent to the curve y = x^3 at (1,1)?
Penny Nom lui répond.
Implicit differentiation 2009-03-01
Emily pose la question :
determine the derivative y' at the point (1,0)
y= ln(x^2+y^2)

y'(1)= ??

Stephen La Rocque lui répond.
36cm to the second power divide 10cm 2009-02-26
val pose la question :
36cm to the second power divide 10cn
Penny Nom lui répond.
A triangular pyramid 2009-02-26
Craig pose la question :
Can you help with this question:
Write the names of the faces and the number of each kind of face on a triangular pyramid?

Penny Nom lui répond.
Multiplying in different bases 2009-02-25
Susan pose la question :
11 base 2 X 22 base 3 + 33 base 4 = _________ base 5
Robert Dawson lui répond.
A rectangular prism 2009-02-25
Christina pose la question :
I am trying to find the area of the base of a rectangular prism where one cube=_1 cu in. The Answer is given in square inches. I also need the volume of the prism in cu in. For example length=3, w=2, h=5
Penny Nom lui répond.
mutiplying decimals 2009-02-24
telly pose la question :
how to do this sum0.24*0.398
Robert Dawson and Penny Nom lui répond.
Time in a Swing 2009-02-22
Barb pose la question :
I looked at the questions concerning a pendulum as I know I need to use this formula but I am stiil not able to figure this problem out. Can you help? If a child is on a swing with a 10 foot chain, then how long des it take to complete one cycle of the swing? I know I am suppose to use the formula 8T^2 = pi^2 L but I do not understand how to do this. Thanks
Janice Cotcher lui répond.
16 golfers playing one round of golf on each of four days 2009-02-22
Jim pose la question :
I have seen solutions for other combinations of setting up a golf trip but I haven't seen one for my group. We have 16 golfers playing one round of golf on each of four days. Is there a way for all of them to play in foursomes so that everyone gets to play with everyone else? Thanks for your consideration.
Chris Fisher lui répond.
6 golfers playing 8 rounds 2009-02-21
evan pose la question :
I have a group of 6 golfers playing 8 rounds. we would like to rotate the 3 somes so each person golfs with different people as many times as possible
Chris Fisher lui répond.
A ten player schedule 2009-02-19
john pose la question :
I HAVE 10 PLAYERS ONE GAME PER NITE FOR 10 WEEKS WHO PLAYS WHO FOR WEEK 1 THRU 10 THANK YOU
Victoria West lui répond.
Implicit differentiation 2009-02-18
Sunny pose la question :
Find slope of the tangent line to the curve 2(x^2+y^2)2=25(x^2–y^2) at (3,-1)
Robert Dawson and Harley Weston lui répond.
The area between the x-axis and a curve 2009-02-18
Lauren pose la question :
This is from a homework question I can't figure out.

Let R be the region in the fourth quadrant enclosed by the x-axis and the curve y= x^2 - 2kx, where k > 0. If the area of the region R is 36 then what is the value of k?

Robert Dawson lui répond.
The second derivative of h(x)=f(g(x)) 2009-02-16
Kristina pose la question :
If h(x)=f(g(x)), and is differentiable, then find h"(x).
Robert Dawson lui répond.
The surface area of a cube and rectangular prism 2009-02-16
Mhiko pose la question :
how to derived the total surface area of a cube and rectangular prism using geoboard??
Robert Dawson lui répond.
Nullity 2009-02-16
Justin pose la question :
What exactly does the term nullity mean in the context of transreal numbers invented by Dr. James Anderson?

Thanks for your help in answering this question!

All the Best,

Justin

Robert Dawson lui répond.
0/0 2009-02-15
Justin pose la question :
Hello, I was just wondering, what is the difference between 0/0 being represented as nullity or as an indeterminate form?

Justin

Harley Weston lui répond.
The height of a triangle 2009-02-14
GARIMA pose la question :
IF THE SIDES OF TRIANGLE ARE GIVEN . HOW WE WILL CALCULATE THE HEIGHT OF TRIANGLE.
Penny Nom lui répond.
8 golfers, 5 rounds 2009-02-12
John pose la question :
8 of us are going on a golf trip next month. We are golfing 5 rounds. Is there any way that everyone can play with each other at least twice? I have been trying to figure it out but it always seems that someone plays with another golfer only once.
Chris Fisher lui répond.
9 golfers 2009-02-12
Beth pose la question :
I am organizing a golf trip for 9 women. We are playing in threesomes and are playing four days. I would love to mix it up so that everyone gets to play with each other at least once. Any mathematical suggestions as to how I can make this happen??
Robert Dawson lui répond.
A semi circular room 2009-02-11
Taylor pose la question :
being 450pi (the area of a semi circle)

how much money would it cost to lay carpet at $2.57 per foot???

if the area of a semi circular room is 450 pi ft then what is the volume being that the diameter is 60 feet and the ceiling is 16 feet tall

Robert Dawson and Penny Nom lui répond.
Solid figures 2009-02-10
Rebecca pose la question :
I need to know the names of solid figures. I have several pictures that I need to answer what type they are. Thanks, Rebecca 3rd grade
Penny Nom lui répond.
A definite integral 2009-02-09
Mathata pose la question :
Evaluate: integral from 0 to 1, x^2 e^x^3dx
Harley Weston lui répond.
Seating around a circular table 2009-02-09
Jenny pose la question :
A committee of 15 -- 9 women and 6 men -- is to be seated at a circular table (with 15 seats). In how many ways can the seats be assigned so that no two men are seated next to each other?
Victoria West lui répond.
A triangular pyramid 2009-02-06
kamran pose la question :
what would be an example of a triangular pyramid
Stephen La Rocque lui répond.
A trig limit 2009-02-05
Samantha pose la question :
lim x-> 0 ( ( r*cos(wt +h) + r*cos(wt) )/ h )

Where r & w are constants.

Harley Weston lui répond.
Faces, edges and vertices 2009-02-04
sheila pose la question :
what is the relationship between the number of faces and the number of edges of a triangular pyramid ?
Penny Nom lui répond.
A point on -8x^2+5xy+y^3=-149 2009-02-04
Vivian pose la question :
Consider the curve defined by -8x2+5xy+y3=-149
a) find dy/dx
b) Write an equation for the line tangent to the curve at the point (4,-1)
c) There is a number k so that the point (4.2,k) is on the curve. Using the tangent line found in part b), approximate the value of k.
d) write an equation that can be solved to find the actual value of k so that the point (4.2,k) is on the curve
e) Solve the equation found in part d) for the value of k

Harley Weston lui répond.
12 golfers 2009-02-04
david pose la question :
12 golfers playing in 3 four balls over five rounds. can everyone play with everyone else at least once.
Victoria West lui répond.
Simultaneous equations with an xy term 2009-02-01
angelee pose la question :
xy+5x-2y-10=0 2x+y=1
Penny Nom lui répond.
7 golfers playing 4 days of golf 2009-01-30
Trish pose la question :
I have 7 golfers and we are playing 4 days of golf. I would like to do pairings so that everyone gets to play with each other at least one time. Can you help with this? Not a math whiz here and looking for help. Thank you.
Victoria West lui répond.
limit sinx/x 2009-01-30
Jackie pose la question :
how to evaluate limit sinx/x as x tends to zero if x is in degrees
Stephen La Rocque and Harley Weston lui répond.
A tennis schedule 2009-01-26
Lorraine pose la question :
I need to create a tennis schedule. We have 8 players, but only 4 play at one time. I need to create a schedule for 34 matches, but of the 34 each player plays 17 times. Each player must also play with each of the other 7 the same number of times. For instance player 1 will play 17 times, and will play with the other players the same number of times as player 2 does. Is this even possible?
Robert Dawson lui répond.
An integral from 1 to infinity 2009-01-24
Ray pose la question :
Determine the area bounded by the x-axis and the curve y=1/(x^2) from x=1 to x=infinity.
A. 1.00
B. infinity
C. indeterminate
D. 2.00

Harley Weston lui répond.
20 of us golf together in groups of 4 2009-01-24
D. pose la question :
Every Sunday, 20 of us golf together in groups of 4. I am looking for a way that each of us play with 3 other people each week and ultimately get to play in groups that are unique. For instance if week 1, I play with 2, 3, 4 and then the next week I play with 5, 6, 7, and the 3rd week I play with 8, 9, 10 and so forth until I have played with everyone. Everbody else should be doing the same thing. Can you give me a schedule for this and how many weeks would it take for all of us to accomplish this where we all play with different combinations of people. (We should not play with the same person very often or even the same pairs of people but everyone should play with everybody else) I hope this makes sense........whew and thanks!
Victoria West lui répond.
Archimedes' formula for parabolic arches 2009-01-23
La pose la question :
Use calculus to verify Archimedes' formula for y=9-x^2. Prove Archimedes' formula for a general parabolic arch.
Harley Weston lui répond.
In the shadow of a flagpole 2009-01-22
La pose la question :
How fast is the length of the shadow of an 18 foot flagpole growing when the angle of elevation of the sun is 45 degrees and is decreasing at a rate of 10 degrees per hour?
Harley Weston lui répond.
The volume of a prism 2009-01-19
Mady pose la question :
What is the general equation used to find the volume of a triangular prism?
Robert Dawson and Harley Weston lui répond.
Partial derivatives 2009-01-17
Meghan pose la question :
I have a question I've been working at for a while with maxima/minima of partial derivatives.

"Postal rules require that the length + girth of a package (dimensions x, y, l) cannot exceed 84 inches in order to be mailed. Find the dimensions of the rectangular package of greatest volume that can be mailed. (84 = length + girth = l + 2x + 2y)"

Harley Weston lui répond.
A 28 week golf schedule 2009-01-17
Don pose la question :
You have a 12 man golf schedule playing 8 weeks. Can you go further and make it a 28 week schedule. 3 foursomes. any help would be greatly appreciated.
Victoria West lui répond.
A golf trip with 12 golfers 2009-01-16
Brad pose la question :
I am organizing a golf trip with 12 golfers. We will play 5 rounds (3 foursomes each round) total and play two 9 holes matches per round (18holes). So two matches are created within each foursome. What are the possible combinations of foursomes so that everyone plays each other at least once with the least number of repeat matches? For example: Here are the foursomes for round 1:(1,2,3,4) (5,6,7,8) (9,10,11,12) match 1 round 1: 1vs2, 3vs4, 5vs6, 7vs8, 9vs10,11vs12 we can not split foursomes up for the second match (9 hole matches): So match 2 round 1: 1vs3, 2vs4,5vs7,6vs8,9vs11,10vs12
Victoria West lui répond.
A line is perpendicular to another line 2009-01-16
Greg pose la question :
A line is perpendicular to the line y=2x+3 and has the same x-intercept as x+3y+10. Find the equation of this line. Express your answer in the form of y=mx+b. Justify your answer.
Robert Dawson lui répond.
Multiplying negative numbers 2009-01-15
stephanie pose la question :
hi, I'm currently working in a grade 8 class where the students are learning about integers. the students were asked to find examples of how integers are applied in real life. they were able to find aplications for: adding and subtracting both positive and negative integers. however, we could not find an example of where two negative integers would be either divided or multiplied together in real life. For example: (-2)(-4)= +8. please help.
Harley Weston lui répond.
A rectangular prism 2009-01-13
nika pose la question :
How many bases does a rectangular prism have? What are they?
Harley Weston lui répond.
Negative rate of change 2009-01-12
hemanshu pose la question :
when i have to find rate of change of decrease in any value my ans comes in negative why??????????
Penny Nom lui répond.
16 golfers 2009-01-11
Bill pose la question :
I have read many of your answers to similar questions but still need your assistance. We have 16 golfers that will be playing in 4 groups of 4 each day on an upcoming golf vacation. Can you suggest the best way to scheudle the individuals assigned to each daily foursome so that we have the most diversity in foursomes throughout the vacation. I am trying to have evryone play with as many members of the group over the week with the least amount of repetition. If you would please provide it for both 6 and 7 days, since we are not sure yet how many days we will be playing.
16 golfers-playing in groups of 4 each day-6 days
16 golfers-playing in groups of 4 each day-7 days

Victoria West lui répond.
Triangular prisms and pyramids 2009-01-10
KJ pose la question :
What are some items found in a home or school that could be considered a triangular prism or triangular pyramid?
Robert Dawson and Penny Nom lui répond.
18 golfers and 5 rounds 2009-01-10
steve pose la question :
We have 18 golfers 5 rounds . Would like 3 foursomes and 2threesomes each day Different groups each day.
Victoria West lui répond.
What is the maximum revenue? 2009-01-09
Kristy pose la question :
A skating rink manager finds that revenue R based on an hourly fee x for skating is represented by the function R(x) = -200x^2 + 1500x

What is the maximum revenue and what hourly fee will produce maximum revenues?

Harley Weston lui répond.
A max/min problem 2009-01-09
Angelica pose la question :
have 400 feet of fence. Want to make a rectangular play area. What dimensions should I use to enclose the maximum possible area?
Robert Dawson lui répond.
The area of a region bounded by two curves 2009-01-07
Rogerson pose la question :
Find the area, S, enclosed by the given curve(s) and the given line.
y = x^2 - x - 1, y = x+2

Harley Weston lui répond.
A kennel with 3 individual pens 2009-01-06
Jean pose la question :
An animal clinic wants to construct a kennel with 3 individual pens, each with a gate 4 feet wide and an area of 90 square feet. The fencing does not include the gates. Write a function to express the fencing as a function of x. Find the dimensions for each pen, to the nearest tenth of a foot that would produce the required area of 90 square feet but would use the least fencing. What is the minimum fencing to the nearest tenth?
Harley Weston lui répond.
The area enclosed by a curve and the x-axis 2009-01-04
Rogerson pose la question :
Find the area, S, enclosed by the curve y = -x^2 + 6x - 5 and the x-axis in the interval 0≤x≤4.
Harley Weston lui répond.
Determine y'' by implicitly differentiating twice 2009-01-04
Walter pose la question :
Given x^3 - 3xy + y^3 = 1 , determine y'' by implicitly differentiating twice. I cannot solve this. Would you be kind enough to perform the mathematics and show the steps involved in obtaining the solution?
Harley Weston lui répond.
The area of a region in the plane 2009-01-03
Rogerson pose la question :
Find the area, S, of the shaded region enclosed by the given cureve, the given line and the x-axis.

y = -x^2 + 1
line x = 2

Harley Weston lui répond.
24 golfers playing golf for 9 rounds 2008-12-23
Duane pose la question :
Got 24 golfers playing golf for 9 rounds. Any formula where everyone can play with everyone else at least once. We are playing 4somes only. Thanks, Duane
Victoria West lui répond.
Pouring angles for a crucible 2008-12-20
Richard pose la question :
I am trying to work at pouring angles and volume left in during pouring a crucible, The crucible is cylindrical and flat bottomed.

I know the diameter, radius and volume of the crucibles. and the volume of liquid going into it.

So lets say the crucible is only half full firstly I need to work out the angle just before its going to pour. ( I can work this out as long as there is a certain volume of liquid if its not enough I cant do it)

Now the problem I also need to work out how much I should tilt the crucible to allow a certain amount out and be able to do this untill the volume reaches 0 at 90' turn. This is where I am stuck.

The reason for needing to be able to work this out is so i can develop a constant flow for example 10Kg of metal per second. Thank you very much for you time

Harley Weston lui répond.
Three circular rings 2008-12-18
seema pose la question :
three circular rings of equal radii of 1cm each are touching each other. a string runs all around the set of rings very tightly. what is the minimum length of string required to bind all the three rings ?
Robert Dawson lui répond.
Integral of cos^2 X between pi/2 and 0 2008-12-18
Wanda pose la question :
Integral or Area of cos^2 X between pi/2 and 0.

The answer that I got is -pi/4. Is this correct? If not, how did you come up with your answer?

Robert Dawson lui répond.
The 2 rightmost digits 2008-12-18
Peter pose la question :
Is there a pattern for the 2 rightmost digits of a power? For example, one problem for a math competition was what are the 2 rightmost digits of 3^1993?
Robert Dawson and Victoria West lui répond.
Surface area of an irregular shape 2008-12-15
Patrick pose la question :
An irregular shaped object (lets say a gold nugget, not smooth with pockets) can have its volume determined by comparing its mass in water.

Is there any method or means or anything that could be used to determine the surface area of this shape? Whether that be theoretical mathematical formula to using a special infrared technique,etc...

The problem I foresee is that the component parts cannot be divided into smaller geometric shapes. I would propose an answer although I don't know if it is a good one: A liquid material that dries super-thin, but has a very specific and easily determined volume/mass is coated over the object. Measure the mass difference between the beginning sample of fluid and the mass after the object has been coated. Then determine the surface area of the same mass of fluid in a geometric shape. Is this feasible?

Robert Dawson lui répond.
A sphere in a can of water 2008-12-12
Meghan pose la question :
A cylindrical can open at the top has (inside) base radius equal to 1. The height of the can is greater than 2. Imagine placing a steel sphere of radius less than 1 into the can, then pouring water into the can until the top of the sphere is just covered.

What should be the radius of the sphere so the volume of water used is as large as possible?

Harley Weston lui répond.
Golf - 24 guys 4-somes 6 rounds 2008-12-07
Duane pose la question :
Thank you. What about 24 guys 4-somes 6 rounds everyone playing with each other at least once.

Thanks again.
Duane

Victoria West lui répond.
Gravity's Effect at a Distance 2008-12-06
ROHAN pose la question :
AT WHAT HEIGHT FROM THE SURFACE OF EARTH,THE WEIGHT OF THE BODY BECOMES HALF? (RADIUS OF THE EARTH = 6400)
Stephen La Rocque lui répond.
6 golfers play threeball 2008-12-04
Ian pose la question :
I have a group of 6 golfers wanting to play 3 rounds as 2x threeballs, but with different players each day. Is this possible? Can you provide some threeball combinations for this please?
Victoria West lui répond.
Six golfers 2008-11-25
Joe pose la question :
I have a group of 6 golfers wanting to play 3 or 4 rounds but with different players each day if possible. We know we cannot make it happen without some playing with the same person. Can you provide some pairings for this?
Victoria West lui répond.
Jogging around a circular track 2008-11-19
kim pose la question :
Joan wants to jog 10 miles on a circular track 1/4 miles in diameter. How many times must she circle the track?
Penny Nom lui répond.
Insulation in an attic 2008-11-18
Scott pose la question :
A homeowner wishes to insulate her attic with fiberglass insulation to conserve energy. The insulation comes in 40-cm-wide rolls that are cut to fit between the rafters in the attic. If the roof is 6 m from peak to eave and the attic space is 2 m high at the peak, how long does each of the pieces of insulation need to be? Round to the nearest tenth.
Harley Weston lui répond.
A word problem about a regular pentagon 2008-11-16
Sandra pose la question :
If the area of a regular pentagon is A=1.720a^2, in which a is one of the sides. Find the area of a regular pentagon with a side that measures 50cm.
Chris Fisher and Penny Nom lui répond.
Multiplication 2008-11-14
natalie pose la question :
what is 8 times 30
Penny Nom lui répond.
Confusion in a multiple choice question 2008-11-14
BJ pose la question :
My son got this math problem which he could mostly solve. Here it is:

The highest location in a certain country is 4525 m above sea level. The lowest point in the same country is 192 m below sea level.

a) Find the difference of the two elevations. His answer: a-(-b)= a+b or 4525-(-192)= 4525+192=4717 m. No problem.

b) A city is 2221 m above sea level. Is this elevation closer to the highest point or the lowest point? His answer: highest point because: 4525-2221= 2304 m (closest to the highest point) and 2221-(-192)=2221+192= 2413 m. (farthest to the lowest point). OK so far.

But then he was given 4 choices for this question with no other information:

a) 4717 m; lowest b) 4333 m; lowest c) 4333 m; highest d) 4717 m; highest

What does it mean? What's the connection with the rest of the problem?

Harley Weston lui répond.
How fast is the distance between the airplanes decreasing? 2008-11-10
Crystal pose la question :
At a certain instant, airplane A is flying a level course at 500 mph. At the same time, airplane B is straight above airplane A and flying at the rate of 700 mph. On a course that intercepts A's course at a point C that is 4 miles from B and 2 miles from A. At the instant in question, how fast is the distance between the airplanes decreasing?
Harley Weston lui répond.
The height of an equilateral triangle 2008-11-06
touqeer pose la question :
My question is that how can we find the height of an equilateral triangle without using pythagoras theorem?
Penny Nom lui répond.
A trig limit 2008-11-04
Teri pose la question :
Although I have this problem completely worked out in front of me I still cannot understand how it was done. The problem is:
Find the limit.
lim x->0 sin2x/tan7x.

Harley Weston lui répond.
Separating variables 2008-11-04
Terry pose la question :
by separating variables solve the initial value problem

(x+1)y' + y = 0 y(0) = 1

Harley Weston lui répond.
A belt drives a pulley 2008-11-03
Rebecca pose la question :
A point on a belt is moving at the rate of 30 ft/sec and the belt drives a pulley at the rate of 200 rpm. What is the radius of the pulley?
Penny Nom lui répond.
An irregular, truncated pyramid 2008-10-24
phillip pose la question :
Hi, I have a rectangular base to square top, pyramid shape to fabricate out of steel plate (4 sections). The base is 3000mm by 800mm, the height is 1100mm and the top is 350mm square. What I need to know is the lengths of the sides of the plate(were they meet vertically). Hope ive made it clear enough. Thanks Phil.
Stephen La Rocque lui répond.
Taxes in Taxylvania 2008-10-22
April pose la question :
Taxylvania has a tax code that rewards charitable giving. If a person gives p% of his income to charity, that person pays (35-1.8p)% tax on the remaining money. For example, if a person gives 10% of his income to charity, he pays 17 % tax on the remaining money. If a person gives 19.44% of his income to charity, he pays no tax on the remaining money. A person does not receive a tax refund if he gives more than 19.44% of his income to charity. Count Taxula earns $27,000. What percentage of his income should he give to charity to maximize the money he has after taxes and charitable giving?
Harley Weston lui répond.
Antiderivative of 1/(x(1 - x)) 2008-10-22
Matt pose la question :
derivative of dx/(x(1-x))

From what I've seen I should break apart the equation as such derivative of dx/x - dx/(1-x) and then get the 2 corresponding log functions.

If that is correct why does this factoring work, if that is incorrect what is the proper way to find the derivative.

Harley Weston lui répond.
I have 6 vertices and 10 edges 2008-10-20
Jacquelin pose la question :
I have 6 vertices and 10 edges. One of my faces is a regular polygon.
Chris Fisher lui répond.
The slope of a tangent line 2008-10-18
Amanda pose la question :
If f(x)=square root of (x+4), and the slope of the tangent line at x=21 was 1/n for some integer n, then what would you expect n to be?
Stephen La Rocque lui répond.
Two equations in two unknowns 2008-10-17
Dushayne pose la question :
Please help me in solving this problem:
a. 3x-4y=32
5x+2y=10

b. 2x+3y=11
4x+3y=10

Penny Nom lui répond.
Perpendicular lines 2008-10-16
Hailey pose la question :
In the xy-plane, the line with equation 2x + y = 3 is perpendicular to the line with equation y = mx + b, where m and b are constants. What is the value of m?
Penny Nom lui répond.
Concavity and the second derivative 2008-10-15
Christina pose la question :
I'm having trouble solving for a second derivative for the following graphing question.

f(x) = (X^2+2x+4)/2x

using the quotient rule, I found:
f'(x) = (x^2-4)/(2x^2)

however, using the quotient rule again I can't seem to solve it (concavity):
f'''(x)=[(2x)(2x^2)-(x^2-4)(4x)]/[(2x^2)^2]
f''(x)=[(4x^3-(4x^3 -16x)]/4x^4
f''(x)=16x/4x^4
f''(x)=4/x^3

and making the equation equal to zero result in 0=4 which doesn't seem to make sense...

Penny Nom lui répond.
The volume of fluid in a semi circular trough 2008-10-10
Kerry pose la question :
How do I find the volume of fluid in a semi circular trough?
Penny Nom lui répond.
Linear and Angular Velocity 2008-10-10
Matthew pose la question :
The pedal and gear relationship of a bicycle is shown. The radii of the gears are r(sub1) = 5 cm and r(sub2)= 12 cm. The radius of the wheel is r(sub3) = 30 cm. How many rotations per minute of the pedal gear will produce a racing cyclist's speed of 60 km per hr. For a diagram: http://bikepedalsgears.weebly.com/
Stephen La Rocque lui répond.
Two modular equations 2008-10-08
Mhiko pose la question :
please solve this Chinese remainder problem..and give me a solution or rule in order to solve this problem/

x=2mod15
x=1mod25

Stephen La Rocque lui répond.
3,6,10,15,and 21 2008-09-28
jarred pose la question :
i am currently stumped on a math project that requires me to find out the recursive formula for a sequence of numbers. the numbers in the sequence are 3,6,10,15,and 21. Thank you for your time.
Walter Whiteley lui répond.
The region between two circles 2008-09-24
Carol pose la question :
Good day! Here is a picture of the problem that we need to solve. (I send the picture through e-mail.) A small circle is inside a larger circle, the only given in the problem is the chord of the larger circle tangent to the smaller circle which measures 16cm. The question is, what is the area of the shaded region? Can you answer this question? Thanks! :)
Harley Weston lui répond.
An Euler diagram and a logic argument 2008-09-18
Regina pose la question :
Use a Euler diagram to determine whether the following argument is valid or invalid.

No wizard can yodel
All lizards can yodel

No wizard is a lizard

Penny Nom lui répond.
A multiple choice exam 2008-09-13
Phalange pose la question :
A multiple choice exam consists of 12 questions, each having 5 possible answers. To pass, you must answer at least 9 out of 12 questions correctly. What is the probability of passing if:
a. You go into the exam without knowing a thing, and have to resort to pure guessing?
b. You have studied enough so that on each question, 3 choices can be eliminated. But then you have to make a pure guess between the remaining 2 choices.
c. You have studied enough so that you know for sure the correct answer on 2 questions. For the remaining 10 questions you have to resort to pure guessing.

Harley Weston lui répond.
Perpendicular 2008-09-08
amanda pose la question :
hi my name is amanda medjuck i am in grade 7. we are learning basic geometry terms and have to define words BUT I DONT KNOW WHAT A PERPENDICULAR LINE IS PLEASE HELP ME!!!
Penny Nom lui répond.
The biggest right circular cone that can be inscribed in a sphere 2008-09-08
astrogirl pose la question :
find the volume of the biggest right circular cone that can be inscribed in a sphere of radius a=3
Harley Weston lui répond.
An array of tulip bulbs 2008-09-06
Mary pose la question :
My daughter had this problem on one of her tests. She forgot to ask her teacher what she did wrong. I understand she did not answer the question in full, but what she did answer I feel is right.

Please explain to me the correct answer. Thank you. Mary

Harley Weston lui répond.
Parallel and perpendicular lines 2008-09-06
C pose la question :
Can you help me determine whether these lines are parallel, perpendicular or neither and why?

Problem 1: y=3x+2
y=1/3x - 4

Problem 2: y=1/3x + 1/2
y=1/3 x -2

Penny Nom lui répond.
How accurate is the following calculation? 2008-09-03
Craig pose la question :
Given that these numbers are only accurate to 2 decimal places, how accurate is the following calculation?

1.73 - 2.16 + 0.08 + 1.00 - 2.23 - 0.97 + 3.02 = 0.47

How do I work out the accuracy?

Penny Nom lui répond.
Irregular polygon and Circle that Intersects All Sides 2008-08-20
Xetro pose la question :
Hi, Suppose you have an irregular polygon(convex or concave) with n > 3 sides. The question is - Find some circle that will cut(in limiting case - touch) all the sides of that polygon. It doesnt matter how many times it cuts the side(1 or 2), it just have to cut or touch it. How to find such a circle? or how to decide if such circle even exists? What if those segments do not form a polygon but are some arbitrary segments ? Really want to know how to do it................ Thanks a lot.. Regards, Xetro
Janice Cotcher lui répond.
Angular & Linear Speed from a Sine Graph 2008-08-19
Kim pose la question :
Kim, a student: I am given a graph with a wave. The amplitude is 5cm and the period is 4cm. I am suppose to find the angular speed. What I need to know is the formula for angular speed and how do I use these numbers to get the correct answer.
Janice Cotcher lui répond.
A triangular field 2008-08-15
joneka pose la question :
Barb walked five times aroun a triangle field all together she walked 435 feet the first side of the triangle fiel is 23 feet the length of the other two sides are of the same measure. What are the dimensions of the triangle field?
Penny Nom lui répond.
[f(x)-f(1)]/(x-1) 2008-08-14
katie pose la question :
Evaluate (if possible) the function of the given value of the independent variable:

f(x)=(x^3)-x:

[f(x)-f(1)]/(x-1)

Penny Nom lui répond.
Angles in a regular tetrahedron 2008-08-07
Carla pose la question :
Hi guys, A regular tetrahedron has all its edges 8cm in length. Find the angles which an edge makes with the base. Thanks. Carla
Penny Nom lui répond.
Find the product of 2^35 and 5^38 in sci. notation. 2008-08-03
Peter pose la question :
I am preparing for a competition and a lot of the non-calculator problems are like find the product of 2^35 and 5^38 in sci. notation. How would you do that?
Penny Nom lui répond.
Right Sided Limit of an Exponential Function 2008-07-29
joseph pose la question :
I am trying to find the limit as X approaches 0 from the positive side of x to the sin of x which look like this lim x^sinx x->0+
Harley Weston lui répond.
Integral of X^2 2008-07-28
Hemanshu pose la question :
Integral of X^2
Janice Cotcher lui répond.
Simultaneous equations 2008-07-23
Franco pose la question :
Solve

3 D + E - F = -10
-2 D - F = -4
-3 D - 4 E - F = -25

Franco

Penny Nom lui répond.
The maximum range of a projectile 2008-07-22
kwame pose la question :
the range R of projectile fired with an initial velocity Vo ,at an angle of elevation (@ )theta from the horizontal is given by the equation R = (Vo(squared) sin2theta)/g. where g is the accelation due to gravity . Find the angle theta such that the projectile has maximum range .
Harley Weston lui répond.
A square and a circle 2008-07-20
kobina pose la question :
4 ft of a wire is to be used to form a square and a circle. how much of the wire is to be used for the square and how much should be used for the square in order to enclose the maximum total area
Harley Weston lui répond.
multiply = 3a - 3b by a+b 2008-07-15
chaleen pose la question :
multiply = 3a - 3b by a+b
Penny Nom lui répond.
A Falling Rock 2008-07-11
Rita pose la question :
This question has 3 parts. If a rock falls from a height of 20 meters on the planet Jupiter, its height H (in meters) after x seconds is about H(x) = 20 - 13x^2 (a) What is the height of the rock when x = 1.2 seconds? (b) When is the height of the rock 15 meters? (c) When does the rock strike the ground? Thank you
Janice Cotcher lui répond.
A difference quotient 2008-07-10
Rita pose la question :
Find the difference quotient of f, that is, find [f (x + h) - f (x)]/h, where h does not = 0 for the given function. Be sure to simplify.

f(x) = 1/(x + 3)

Janice Cotcher lui répond.
A dog tied to a round building 2008-07-08
maitham pose la question :
i have this question which i don't know how to solve it :

One dog was linked to the outer wall of a building round of 20 meters in diameter. If the length of chain linking the dog sufficient turnover of half the distance around the building, What area can guard dog?

they said that we can solve it by integral .. can you solve it for me?

Harley Weston lui répond.
A rectangular room 2008-06-30
Peter pose la question :
A rectangular room has a perimeter of 42 feet and an area of 108 square feet. What is the length, in feet, of the shorter side?
Penny Nom lui répond.
Remainders 2008-06-30
vivek pose la question :
what is the remainder when 2050*2071*2095 is divided by 23 ? this question needs to be done in as less time as possible.
Penny Nom lui répond.
A golf tournament 2008-06-24
Roland pose la question :
We have a golf tournament, 12 players, three rounds. We want to pair so that we have pairings with as few duplications as possible.
Janice Cotcher lui répond.
Two forces acting on a body 2008-06-21
Rita pose la question :
Before answering the question below, what exactly is, in basic words, resultant force? QUESTION: Two forces of 50 and 68 pounds act on a body to produce a resultant force of 70 pounds. Find, to the nearest 10 minutes or nearest tenth of a degree, the angle formed between the resultant force and the smaller force.
Harley Weston lui répond.
sin(2x)/sin(3x) 2008-06-19
matt pose la question :
how does sin2x break down (not with identities) and how would sin3x be created. My prob. is sin 2x/ sin 3x and I want to know how the double(or triple angle) would break down. I want to be able to cancel out sins. Thanks!
Harley Weston lui répond.
A line which is perpendicular to another line 2008-06-18
Emily pose la question :
How do you solve perpendicular bisectors with a slope and a point?
Penny Nom lui répond.
Escort cards on a rectangular table 2008-06-13
Susan pose la question :
What size rectangular table will we need to put 230- 2" X 3.5" escort cards on for my daughter's wedding reception with a spacing of .5 inch between them on all sides? Thanks so much for your help
Penny Nom lui répond.
The current in a river 2008-06-12
Joi pose la question :
To approximate the speed of the current of a river, a circular paddle wheel with radius 4 feet is lowered into the water. If the current causes the wheel to rotate at a speed of 10 revolutions per minute, what is the speed of the current? Express your answer in miles per hour.
Harley Weston lui répond.
The rate of change in the depth of the water 2008-06-12
Liz pose la question :
A rectangular pool 50ft long and 30ft. wide has a depth of 8 ft. for the first 20 ft. for its length and a depth of 3 ft. on the last 20ft. of its length and tapers linearly for the 10 ft in the middle of its length. the pool is being filled with water at the rate of 3ftcubed/ min at what rate is the depth of the water in the pool increasing after 15 hours?
Harley Weston lui répond.
Two circles 2008-06-10
cey pose la question :
the diameter of the larger circle is 20cm, and the smaller 10cm. what is the shaded area??
Janice Cotcher lui répond.
How do i find the area of an irregular figure? 2008-06-10
DAVE pose la question :
How do i find the area of an irregular figure?
Penny Nom lui répond.
A recursive formula for 9, -18, 36, -27, ... 2008-06-05
Rita pose la question :
I am having trouble understanding how to write formulas (I should say create formulas) from a given sequence. It does not matter what sort of sequence it is. The confusion remains. I have not found a textbook or easy math book that explains this process for the average Joe to grasp. Here are the two questions:

(1) Write a recursive formula for the sequence
9, -18, 36, -72, ...

(2) Write a recursive formula for the sequence
3, 3(sqrt{3}), 9, 9(sqrt{3})

Penny Nom lui répond.
The area of a regular pentagon 2008-06-02
Jenna pose la question :
I would like to know if there is a formula to find the area of a regular Pentagon that is not inscribed in a circle only apothem and side length are given.
Penny Nom lui répond.
The length of a shadow 2008-05-27
Simon pose la question :
A figure skater is directly beneath a spotlight 10 m above the ice. IF she skates away from the light at a rate of 6m/s and the spot follows her, how fast is her shadow's head moving when she is 8m from her starting point? The skater is (almost) 1.6m tall with her skates on.
Stephen La Rocque and Harley Weston lui répond.
A season schedule for 9 teams 2008-05-22
James pose la question :
would like to put together a season schedule for 9 teams with each team playing three or 4 games leading to a 1 to 9 ranking.
Victoria West lui répond.
The angles of an irregular pentagon 2008-05-20
victoria pose la question :
The sum of the measure of two angles is 240. If the remaining angles are congruent, what is the measure of each angle?
Penny Nom lui répond.
The weight of a concrete column 2008-05-11
russell pose la question :
a cylindrical form is filled with a slow curing concrete. The base of the form is 10 ft in radius, and height is 25 ft. while the concrete hardens, gravity causes the density to vary from a density of 90 lbs/ft^3 at the bottom to a density of 50 lb/ft^3 at the top. Assume that the density varies linearly from the top to the bottom, and compute the total weight of the resulting concrete column
Harley Weston lui répond.
A linear model 2008-05-09
Shelby pose la question :
How would you write a linear model to represent the population of a city that has a population of 547,725 and a growth rate of -25,195 per year with t represents the number of years since 1994?
Penny Nom lui répond.
How do you reduce an equation with multiple variables? 2008-04-30
Jonathon pose la question :
How do you reduce an equation with multiple variables?

For example, if 3x + y = k(x-3), what would x be equal to?

Penny Nom lui répond.
A bowl is the shape of a hemisphere 2008-04-28
josh pose la question :
a bowl is the shape of a hemisphere with diameter 30 cm and water is poured into the bowl to a height h cm. how do i find the volume of the water in the bowl
Harley Weston lui répond.
A lidless box with square ends 2008-04-28
Chris pose la question :
A lidless box with square ends is to be made from a thin sheet of metal. Determine the least area of the metal for which the volume of the box is 3.5m^3. I did this question and my answer is 11.08m^2 is this correct? If no can you show how you got the correct answer.
Stephen La Rocque and Harley Weston lui répond.
An octagonal prism 2008-04-27
Melanie pose la question :
My son is identifying geometric shapes in the real world? We are stuck on octagonal prism, rectangular prism and square prism. Can you help me out with some examples. Thanks
Penny Nom lui répond.
At what value of t is the maximum acceleration? 2008-04-25
Mary pose la question :
Velocity of a function (which is the first derivative of its position) is defined over the interval 0 to 12 using the following piecewise function: v(t)=-1 from 0 to 4, v(t)=x-5 from (4 to 8 and v(t)=-x+11 from (8 to 12. At what value of t is the maximum acceleration?
Stephen La Rocque lui répond.
An irregular tetrahedron 2008-04-24
RAUL pose la question :
I am looking an expression for an edge length as function of the other five edge lengths of irregular tetrahedron.
Walter Whiteley lui répond.
A volume of revolution 2008-04-24
Sabahat pose la question :
Hi, i have a region enclosed by both axes, the line x=2 and the curve y=1/8 x2 + 2 is rotated about the y-axis to form a solid . How can i find the volume of this solid?. (Please note that y equation is read as y =1 over 8 times x square plus 2.) I will be really grateful if you answer this question. :)
Harley Weston lui répond.
An open box 2008-04-23
Le pose la question :
Metal Fabrication; If an open box is made from a tin sheet 8 in square by cutting out identical squares from each corner and bending up the resulting flaps, determine the dimensions of the largest box that can be made.
Harley Weston lui répond.
What is the integral of 13sin^3(x)*cos^7(x)dx? 2008-04-22
Cathrine pose la question :
I am having trouble integrating this problem. It says to evaluate the integral but I don't know what to do or how to do it.

It is the integral of
13sin^3(x)*cos^7(x)d

Harley Weston lui répond.
f(x)=sin^3(3x^2) find f ' (x) 2008-04-21
Michael pose la question :
f(x)=sin^3(3x^2) find f ' (x)
Harley Weston lui répond.
A solid figure with 3 rectangular faces and 2 triangular faces 2008-04-17
Emily pose la question :
I am in the 3rd grade and my teacher gave us a question. What is a solid figure with 3 rectangular faces and 2 triangular faces?
Penny Nom lui répond.
The area bounded by 3 curves 2008-04-13
Sabahat pose la question :
Hi, I have enclosed a diagram.
The diagram shows the curve y=(2x-5)4. The point P has co-ordinates (4,81) and the tangent to the curve at P meets the x-axis at Q.

Find the area of the region (shaded in the diagram) enclosed between the curve, PQ and the x-axis . (Please note that the equation y is read as y=2x -5 whole raise to power 4.)

Stephen La Rocque lui répond.
f(x) =ax^blnx 2008-04-13
charles pose la question :
supposef(x) =ax^blnx is a real- valued function. Determine exact values(not decimal approximations) fro nonzero constants a and b so that the function f has a critical point at x=e^3 and a maximum value of 1/2e
Harley Weston lui répond.
The chain rule 2008-04-10
joey pose la question :
pls help me to Differentiate
y=(3x^2-4x)^8

Harley Weston lui répond.
A regular polygon 2008-04-09
chase pose la question :
name the regular polygon if the sum of the measures of the polygon is 1,440 and the measure of one angle is 144
Stephen La Rocque lui répond.
Side lengths and angles in a regular octagon 2008-04-09
Lori pose la question :
I am a builder and need to find the length of the sides of an octagon. I know the length between the parallel sides (26 feet). What is the length of each side? What is the angle measurement?
Harley Weston lui répond.
Matrix multiplication 2008-04-08
manashi pose la question :
i. why matrix division is not possible?
ii.when we add or subtract two matrix , getting the result by addind or subtracting correspondind elements....but in case of multiplication it is not but why?

Harley Weston lui répond.
How many bags of mulch? 2008-04-07
MJ pose la question :
I have an area that is 35' X 50' and I want to cover it with 2" of mulch.

Do I take length X Width X depth? Bags of mulch are sold in 2 or 3 cubic feet per bag.....I just want to know how to do the math. can you please send me the equation?

Penny Nom lui répond.
What fraction of 45 is 10? 2008-04-06
Scott pose la question :
45 x fraction? = 10
Penny Nom lui répond.
1-(1-P)^75=0.05 2008-04-05
Amy pose la question :
1-(1-P)^75=0.05
Stephen La Rocque and Penny Nom lui répond.
A volume of revolution 2008-04-04
ted pose la question :
Consider the region bounded by y=x^2 + 1, y=5-3x and y=5. Sketch and shade the given region; then set up but dont evaluate teh integrals to find the following:

a) The volume of the solid generated by rotating the region about the line y=5

b) the volume of the solid generated by rotating the region about the y-axis

Penny Nom lui répond.
lim as x approaches infinite of 5x + 2/x-1 2008-04-04
Jordan pose la question :
how to solve this.

lim as x approaches infinite of 5x + 2/x-1

Stephen La Rocque and Harley Weston lui répond.
A shape 2008-04-03
Joshua pose la question :
what shape has 4 faces, 4 corners and 9 edges.
Penny Nom lui répond.
The integral of dx / (4x^2 - 25)^3/2 2008-04-01
Meghan pose la question :
I have a question from the trigonometric substitution of my calculus course.

integral of dx / (4x^2 - 25)^3/2

Harley Weston lui répond.
A rectangular prism 2008-03-31
Chanlie pose la question :
i need need to find a rectangular prism with a total surface area of 210 units.
Penny Nom lui répond.
2 times 0 times 0 times 4 2008-03-30
ben pose la question :
what is 2 times 0 times 0 times 4?
Penny Nom lui répond.
A fish tank in the shape of an irregular pentagon 2008-03-29
richie pose la question :
i am building a fish tank. it is going to be an irregular pentagon. the sides are going to be
24"
24"
8"
8"
32"(approximately)

there will 3 right angles A, B, E

my question is how to figure out the degree of the angles that are not right angles (C,D)?

Chris Fisher lui répond.
A max-min problem 2008-03-27
LSL pose la question :
show that of all rectangle with a given area, the square has the smallest perimeter.
Penny Nom lui répond.
A hydraulic cylinder 2008-03-24
james pose la question :
I am trying to adjust the placement of a hydraulic cylinder that raises a dump bed up from the frame of a truck. How long would the cylinder (height of a triangle) have to be to raise the bed to a 70 degree angle? The base from pivot to cylinder is 132.5 inches.
Stephen La Rocque lui répond.
A train and a boat 2008-03-15
Sabrina pose la question :
A railroad bridge is 20m above, and at right angles to, a river. A person in a train travelling at 60 km/h passes over the centre of the bridge at the same instant that a person in a motorboat travelling at 20km/h passes under the centre of the bridge. How fast are the two people separating 10s later?
Harley Weston lui répond.
Repeating decimals 2008-03-10
Blaine pose la question :
Is it possible to put a repeating decimal number into a calculator to solve a problem?

EX: Write 39.3939... as a fraction.

Penny Nom lui répond.
The volume of a countertop 2008-03-08
Matthew pose la question :
I need to calculate volume down to cubic feet. I pour concrete countertops and cant figure out the formula to convert inches and feet. If I have a 2.5 inch by 2 foot by 16 foot concrete countertop how do I convert it to cubic feet and hence know how many bags of concrete at .6 cubic feet per bag I will need. I am trying to input the formula into a spreadsheet. thank you so much Matt
Penny Nom lui répond.
Two perpendicular lines 2008-03-07
Daphne pose la question :
Does anyone know how to find a line perpendicular to -3x=4y=20 and passes through the point (-12,0)?
Stephen La Rocque and Penny Nom lui répond.
A 6 pointed star 2008-03-04
Siddharth pose la question :
When 2 congruent equilateral triangles share a common center, their union can be a star If their overlap is a regular hexagon with an area of 60, what is the area of one of the original equilateral triangles?

a) 60 b) 70 c) 80 d)90 e)100

Stephen La Rocque lui répond.
Triangular prisms and pyramids 2008-03-03
LORENA pose la question :
hi i am in yr 6 and i need to know how many faces,vertices and edges these shapes have..
. TRIANGULAR PYRAMID
. TRIANGULAR PRISM

Penny Nom lui répond.
What point on the graph y = e^x is closest to the origin? 2008-03-03
elvina pose la question :
What point on the graph y = e^x is closest to the origin? Justify your answer.
Stephen La Rocque lui répond.
Simultaneous equations 2008-02-29
CONOR pose la question :
I was wondering if you could help me with this problem

7x - 5y = -1
3y = 4x

Penny Nom lui répond.
A Norman window 2008-02-25
Jason pose la question :
If the perimeter of a Norman window is 20 feet, what is the maximum area of the window?
Stephen La Rocque lui répond.
A ball bearing is placed on an inclined plane 2008-02-15
Leah pose la question :
A ball bearing is placed on an inclined plane and begins to roll. The angle of elevation of the plane is x. The distance (in meters) that the ball bearing rolls in t seconds is s(t) = 4.9(sin x)t^2. What is the speed of the ball bearing, and what value of x will produce the maximum speed at a particular time?
Penny Nom lui répond.
Two regions with equal area 2008-02-13
James pose la question :
There is a line through the origin that divides the region bounded by the parabola y=3x-5x^2 and the x-axis into two regions with equal area. What is the slope of that line?
Harley Weston lui répond.
A square and an equilateral triangle 2008-02-10
Emily pose la question :
A square pen and a pen shaped like an equilateral triangle have equal perimeters. Find the length of the sides of each pen if the sides of the triangular pen are fifteen less than twice a side of the square pen.
Stephen La Rocque lui répond.
Radians 2008-02-08
Brian pose la question :
I'm teaching Precalculus right now and will be covering the formula to convert angular speed into linear speed. My question is about the units involved. Using v=rw, if r is mearsured in feet and w is measured in radians/second then v is measured in ft/sec. What happened to the units of radians? I can't find a good way to explain why they are not part of units for velocity. Do you have a good explanation? Thanks,
Stephen La Rocque lui répond.
Multiplying decimal numbers 2008-02-05
alwyn pose la question :
Why should when you Multiplying Decimal numbers is value becoming less and less? don't you think even decimal number is a quantity and in no chance when it multiplies its should become less or nil !!!

In fact all Multiplying and or adding the value will go up and only when you subtract and divide it should become less !!

Stephen La Rocque and Penny Nom lui répond.
The radius of a circular arc 2008-02-04
Bill pose la question :
Hi,the Central Angle of a sector of a circle is 40 degrees. The circular arc of the curve has a chord length of 3000 ft. Find the radius(r) of the circular arc.
Penny Nom lui répond.
Numbers and Operations 2008-02-02
Charlotte pose la question :
On the TABE (Test for Adult Basic Education) test, one of the categories is number operation. I would like to know what consist of number operation.
Penny Nom lui répond.
Integration by parts 2008-01-30
seth pose la question :
hi i really dont understand integr ation by parts. for example, the integral(t^2sintdt. i have u=t^2 and v'=sint also u'=t^/3 v=-cost for the formula i have uv-integralvu' dx this is all well and good but i cant get it right.
Harley Weston lui répond.
Angle of Elevation 2008-01-29
Rita pose la question :
Uluru or Ayers Rock is a sacred place for Aborigines of the western desert of Australia. Chun-Wei uses a surveying device to measure the angle of elevation to the top of the rock to be 11.5 degrees. He walks half a mile closer and measures the angle of elevation to be 23.9 degrees. How high is Ayers Rock in feet?
Stephen La Rocque lui répond.
Inflection points 2008-01-25
Armando pose la question :
Hi, Im trying to write a program that takes an equation ( f(x) = 0 ) and returns a list of the inflexion points in a given interval. there must be (I think) a mathematical method or algorithm to do this, probably involving the (second) derivate of the function. However I have not found such a method yet. Any help on this will be much appreciated.
Stephen La Rocque and Harley Weston lui répond.
Maximize income 2008-01-18
Chris pose la question :
Lemon Motors have been selling an average of 60 new cars per month at $800 over the factory price. They are considering an increase in this markup. A marketing survey indicates that for every $20 increase, they will sell 1 less car per month. What should their new markup be in order to maximize income?
Stephen La Rocque and Harley Weston lui répond.
A large concrete shape 2008-01-16
Keith pose la question :
what is the cubic yards of an area that is not a perfect 1/4 circle? The dimensions are 100ft. x 60ft. x 125ft. curcumferal arch x 3ft. depth?
Stephen La Rocque lui répond.
Fact families with multiplication and division 2008-01-16
Donna pose la question :
I am trying to help my granddaughter with some math homework. I looked at what you have on fact families. Am I correct to assume that the same process applies to multiplication and division as well? Her question asks for fact families for 2, 8, & 16.
Penny Nom lui répond.
Balancing on a fulcrum (net torque and equilbrium) 2008-01-15
Eric pose la question :
If I have a 4" bar that has no weight value with a 275lb. weight on one side and a 125lb. weight on the other side what would be the folcrum point? Please keep the answer very simple. Also is there a fixed equation to figure folcrum points? Is there an equation for if the 4" bar had a weight value?
Stephen La Rocque lui répond.
Multiplying numbers in bases other than 10 2008-01-08
Imaiya pose la question :
In class we've been learning about numbers with a base other than ten. For example, base 7, base 5, and base 6. What's really stumping me is how to multiply these numbers, when they aren't a base of 10. I've had it explained to me once, but don't seem to understand it. Here are the equations, and I've listed the first few numbers for that particular base.

Base 5: 0,1,2,3,4,10,11,12,13,14,20 etc... The Equation: 34 x 42 Base 9: 1,2,3,4,5,6,7,8,10,11,12,13,14, etc... The Equation: 56 x 78

If you could explain to me how to multiply numbers with any type of base, I'd really appreaciate it. Also if you could show me how to do one of the questions that I listed, that would really help me =) Thanks so much, and once again, thank you for your great site!

Stephen La Rocque lui répond.
The area of a piece of property 2008-01-05
Greg pose la question :
I need to know the area of a six sided piece of property. I sent a map by regular mail.
Harley Weston lui répond.
The surface area of the rectangular solid 2007-12-30
Sara pose la question :
For my math homework i found the volume of a rectangular solid whose deminsions were in feet and the numbers were 10 by 20 by 30
10 = hight
20 = short side base
30 = long side base
and now i have to find the surface area of the rectangular solid....
i know how to do this but i can't get the right answer it should be 2,200 ft squared but i keep getting 13,00 feet squared

Penny Nom lui répond.
The integral of 1/ (x(x+1)^0.5) 2007-12-29
Nooruddin pose la question :
Integral of
dx / x(x+1)^0.5
(boundaries are 5 and 3)

Harley Weston lui répond.
The standard normal distribution 2007-12-29
GEORGE pose la question :
Statistics texts state that in a normal distribution, 1 standard deviation covers 68%, 2SD 95% 3SD 99.7%. However, on looking at the tables for normal curve areas, the percentage for a z value of 1 is 84.13. 68 % of values are covered at a z value of 0.47. The discrepancies exist even for other values but are smaller. So why is the rule of thumb so different from the Table for Normal Curve areas?
Harley Weston lui répond.
The centroid of a polygon 2007-12-29
David pose la question :
I am playing around with an idea. I've read previous answers to questions (using surveyor calculation methods) related to finding the area of an irregular shape parcel of land (or irregular polygon). Is there a formula or method for calculating the centroid (mid-point) for this type of polygon. I've seen plenty of methods for calculating the mid-point for a triangle or regular shape poloygon but few for an irregular shape polygon such as a parcel of land. Thanks...
Harley Weston lui répond.
Differentiate 2007-12-28
taiwo pose la question :
i am finding it difficult to use first principle to differentiate this question: y=xcos2x. can u help me.
Penny Nom lui répond.
An irregular octagon 2007-12-23
Sheldon pose la question :
I am attempting to construct an irregular octagon picture frame out of bamboo. The bamboo is 1" in diameter and the opening should be 20" H X 16"W.
What measurements should be used?

Penny Nom lui répond.
lim sinx/(x +tanx) 2007-12-16
shimelis pose la question :
i have problem how do you solve this equation lim sinx/(x +tanx)
Harley Weston lui répond.
Shooting an arrow over a wall 2007-12-11
Amy pose la question :
The path of a large arrow fired from a non torsion catapult can be modeled by y=-0.0044x^2 + 1.68x, where x is the distance the arrow travelled (in yards) and y is the height of the arrow (in yards). Given the height of a castle wall, find the safest distance from the wall to launch an arrow over a 120 yard-high wall.
Victoria West and Stephen La Rocque lui répond.
Polygons 2007-12-09
Nicholle pose la question :
a polygon with all sides the same length and all angles the same measure is called a ---- polygon.

What word goes in the blank.

Penny Nom lui répond.
LCM over 100 2007-12-09
Aris pose la question :
A teacher has more than 100 sweets. She thinks that if she give 6, 8 or 9 sweets to her students she will have no remaining in the case. What is the smallest number of sweets in the bag?
Stephen La Rocque lui répond.
System of equations 2007-12-06
Jenn pose la question :
change the equation,x-y=4 to form y=mx+b the solution to the system of equations y=2x and y=-x+3 is
Stephen La Rocque lui répond.
Chicken and goat feet 2007-12-05
Kim pose la question :
Old McDonald raises goats and chickens. The animals have a total of 100 heads adn 360 feet. How many goats and how many chickens does Mr. McDonald have?
Stephen La Rocque and Penny Nom lui répond.
Proving a quadrilateral is a rhombus 2007-12-03
Jeanie pose la question :
How do you prove that a quadrilateral is a rhombus because the diagonals of the quadrilateral are perpendicular and bisect each other using the 2-column proof method?
Stephen La Rocque lui répond.
An arched entry 2007-11-28
amber pose la question :
i am working with an arched entry. i know that the radius is 25' and the height is 20'-11". i need to know the length of the arch and degree of bend of the arch. how do i find these?
Stephen La Rocque lui répond.
Maximize the product 2007-11-25
David pose la question :
Hi i have this site call calcchat.com, but i dont understand how they explained this can you take a look? The question is: Direction: Find two positive numbers that satisfy the given requirements. The sum is S and the product is a maximum this is what they did 1) Let x and y be two positive numbers such that x + y = S 2)P = xy 3) = x (S - x) 4) =Sx - x^2 5)...etc. the thing i dont get is how did they go from step 2 to step 3 and also i know this sound dumb but how did they get step 2? =)
Harley Weston lui répond.
A rectangular plot of farmland 2007-11-25
Christy pose la question :
A rectangular plot of farmland will be bounded on one side by a river and on the other three sides by a single-strand electric fence. With 800m of wire at your disposal, what is the largest area you can enclose, and what are its dimensions?
Harley Weston lui répond.
A curve sketch 2007-11-22
Ahson pose la question :
Find critical points, determine the monotonicity and concavity and sketch a graph of f(x) with any local maximum, local minimum and inflection points labeled:

1. f(x) = x^4 - x^3 - 3x^2 + 1

Harley Weston lui répond.
Area of a 17-sided lot 2007-11-21
Lynda pose la question :
My uncle is wanting to buy this piece of land [a 17-sided polygon] but we are questioning the acerage total. the measurements are [on the attached diagram].
Stephen La Rocque lui répond.
The derivative 2007-11-19
ralf pose la question :
Find the derivative of the function
1-. y=1+2x8
2-. y=(1+2x )8

Harley Weston lui répond.
A rectangle in an ellipse 2007-11-18
David pose la question :
I need to find the max area of a rectangle inscribed in an ellipse with the equation x^2+4y^2=4.. What I have so far is f(x,y)=4xy g(x,y)=x^2+4y^2-4=0,

y=sqrtx^2-4/4 f'(x)=2x^2/sqrt-4x^2+2(sqrt-4+x^2).

What I need to know is how to finish the problem and find the actual mas area of the rectangle. David

Penny Nom lui répond.
lim [x + squareroot(x^2 + 3)] as x->-inf 2007-11-16
David pose la question :
Find the limit. (Hint: treat the expression as a fraction whose denominator is 1, and rationalize the numerator.)

lim [x + squareroot(x^2 + 3)] as x->-inf

i got to

lim -3/(x - squareroot(x^2 + 3)) as x->-inf

but i'm having trouble understanding why the answer is 0 plz explain thx

Harley Weston lui répond.
A rectangular prism 2007-11-16
nicholas pose la question :
What is the height the surface area to a rectangular prism with a volume of 350 milliliters?
Stephen La Rocque lui répond.
A limit of the form 0 times infinity 2007-11-13
Russell pose la question :
find the following limit

lim x((e^1/x) -1) as x --> infinity

Penny Nom lui répond.
Radial motion 2007-11-13
abdulai pose la question :
a circular saw with diameter 18.4cm rotates at 2400rpm. What is the angular speed of a point on the edge of the saw blade in radians per second?
Penny Nom lui répond.
Local maxima, minima and inflection points 2007-11-13
Russell pose la question :
let f(x) = x^3 - 3a^2^ x +2a^4 with a parameter a > 1.

Find the coordinates of local minimum and local maximum

Find the coordinates of the inflection points

Harley Weston lui répond.
Maximize his profit 2007-11-12
apoorva pose la question :
During the summer months Terry makes and sells necklaces on the beach. Last summer he sold the necklaces for $10 each and his sales averaged 20 per day. When he increased the price by $1, he found that he lost two sales per day.

a. Find the demand function, assuming it is linear.
b. If the material for each necklace costs Terry $6, what should the selling price be to maximize his profit?

Penny Nom lui répond.
Family of functions 2007-11-12
Russell pose la question :
Consider the family of functions
f(t)= Asin3t + Acos3t +Bsin8t + Bcos8t

find exact values of parameters A and B so that f(0) = 2 and f ' (0) = 1

Stephen La Rocque lui répond.
6 consecutive multiples of 6 2007-11-11
jeff pose la question :
find 6 consecutive multiples of 6 whose sum is the least common multiple of 13 and 18
Penny Nom lui répond.
Two integrals 2007-11-09
Akilan pose la question :
how to integrate these (tan x)^6(sec x)^4 and sinh(x)(cosh(x))^2.

Please send me how to do this question. Having exams on Monday. Please help.

Harley Weston lui répond.
Increasing and decreasing for functions 2007-11-09
David pose la question :
Direction: Identify the open intervals on which the function is increasing or decreasing.

f(x)=1/(x^2)
f'(x)= -2/(x^3)

i understand how to get up until there, and the undf. is x=0, but now i'm having problem setting up the number table chart. i cant remember how, and where to place the increase and decrease + - the chart, for example <---------0----------> where would the increase and the decrease be place?

Harley Weston lui répond.
The height of a triangle 2007-11-03
William pose la question :
I need to find the height of a triangle if the base is 15, one side is 14 and the other side is 13.
Stephen La Rocque and Penny Nom lui répond.
f(x+y) = f(x) + f(y) + 2xy 2007-11-01
Marcia pose la question :
For all real numbers x and y, let f be a function such that f(x+y) = f(x) + f(y) + 2xy and such that the limit as h -> 0 of f(h) / h = 7, find: f(0), use the definition of the derivative to find f'(x), and find f(x).
Penny Nom lui répond.
A circular table is pushed into the corner 2007-10-31
Will pose la question :
A circular table is pushed into the corner of a square room so that one point is 8 inches from one wall and 9 inches from the other. What is the radius of the table?
Penny Nom lui répond.
How to solve related rates problems 2007-10-27
David pose la question :
Can you plz explain how and where you come up with an equation to solve this?
Find the rate of change of the distance between the origin and a moving point on the graph of y = sin x if dx/dt = 2 centimeters per second.

Stephen La Rocque lui répond.
Area of a triangle formed by three points on a graph 2007-10-26
Betty pose la question :
My question is 'find the are of a triangle whose vertices have coordinates (3,5),(6,-5), and (-4,10)
Stephen La Rocque lui répond.
The volume of a regular octagonal prism 2007-10-20
Chris pose la question :
I am trying to figure out the volume of a regular octagonal prism, with side lengths of 4 inches. How do I do this?
Penny Nom lui répond.
A triangular lot 2007-10-17
Brian pose la question :
Seeking the square footage of a triangular property lot, dimensions are: 620 X 620X 720.
Penny Nom and Melanie Tyrer lui répond.
5x + 9 = 3x + 7 2007-10-16
kristina pose la question :
hi im kristina and i cannot figure out how 2 do these multi-step equations

5x + 9 = 3x + 7 i can on getting the wrong answer according 2 the book

Penny Nom lui répond.
lim x->1 (root x - x^2)/{1 - root x) 2007-10-16
Meghan pose la question :
Hi! I have a question from my Calculus textbook that I've been picking at for a while and I'm stuck.

lim x->1 (root x - x^2)/{1 - root x).

Stephen La Rocque and Penny Nom lui répond.
Four triangles in a square 2007-10-15
Kristina pose la question :
A square with side lengths of 6 cm is divided into 3 right triangles and a larger isosceles triangle. If the three right triangles have equal area, find the exact area of the isosceles triangle.
Stephen La Rocque lui répond.
lim (1- 2 cosx) / (sin(x- pi/3)) 2007-10-15
hanan pose la question :
lim (1- 2 cosx) / (sin(x- л/3))
Harley Weston lui répond.
Working with x 2007-10-12
Robert pose la question :
The question: The measure of an exterior angle of a regular polygon is given. Find the measure of an interior angle, and find the sides.
41. 36
42. 18
44. 'X'

The attachment has what she has done for 41 and 42. Need help with 44 (lots of help) Thank you in advance for your site and your help. Robert

Penny Nom lui répond.
13 year and 17 year locusts 2007-10-12
stefan pose la question :
how many years pass between the years when both 13 year and 17 year locusts are out at the same time?
Penny Nom lui répond.
A dinner club schedule 2007-10-12
irv pose la question :
The problem is that I have 12 couples, 1 through 12, in a dinner club, and each couple is host once with 3 of the other couples. he dinners will take place over 4 months. I am trying to not repeat any of the couples. We had the formula for 16 and 20 couples but not for 12. Are you able to help with the groupings for 12. Thanks very much. Irv
Victoria West lui répond.
A 9 team league 2007-10-11
Dary pose la question :
I have a 9 team pool league. Each team must play each other once only. But 2 teams cannot play at the same place location each week. I can't seem to match this up and I get stuck.
Victoria West lui répond.
The average rate of change of a function 2007-10-11
vern pose la question :
Find the average rate of change of the function over the given interval. Compare this average rate of change with the instantaneous rates of change at the endpoints of the interval. f(X)=sinX for the inverval [0,pi/6]?
Harley Weston lui répond.
Substitution method 2007-10-11
Kevin pose la question :
3xx+2y=-36-y=11
Stephen La Rocque lui répond.
Coin jar 2007-10-07
a student pose la question :
Sally empties his jar of coins. It contains $3.75 in nickels, dimes, and quarters. The number of dimes is twice the number of nickels and the number of quarters is three less than the number of nickels. Determine how many nickels, dimes, and quarters were in the in the jar.
Stephen La Rocque lui répond.
Solving four simultaneous equations (system of four linear equations) 2007-10-07
Johan pose la question :
I need some help in solving this question
x + 2y - 3z + 4w = 12
2x + 2y - 2z + 3w = 10
0 + y + z + 0 = -1
x - y + z - 2w = -4

Stephen La Rocque lui répond.
Triangle perimeter and area 2007-10-05
divakar pose la question :
The sides of the triangle are integers. The perimeter is 8. What is the area?
Stephen La Rocque lui répond.
Golf pairings 2007-10-02
Mike pose la question :
Regarding arranging golf players so no person plays with anyone more than once. You have given examples for 16 and 24 players. If it can be done, i need a solution for 20 players, 4 players per team one round per day for 5 days
Victoria West lui répond.
Equation of a circle circumscribing a triangle with given vertices 2007-10-01
Randy pose la question :
How do I determine the equation of a circle when it is circumscribed by a triangle whose vertices are (-1, 6), (3, -2), and (2, 5)?
Stephen La Rocque lui répond.
y squared over 3 times 8 over y 2007-09-30
John pose la question :
y squared over 3 times 8 over y
Stephen La Rocque lui répond.
Sample variance and population variance 2007-09-22
Willy pose la question :
I have a question in sample variance and population
my name is Willy
the question in the attached file...

Harley Weston lui répond.
Finding equations, intersection point of two lines at right angles 2007-09-22
Yaz pose la question :
Find the equation of the line joining A(-1,-9) to B(6,120). Another line passes through C(7,-5) and meets AB at rigth angle of D. Find the euation of CD and calculate the co-ordinates of D.
Stephen La Rocque lui répond.
Least Common Multiples 2007-09-20
Aiyanna pose la question :
What is The Lcm Of 3,7,and 8 Because my Teacher gave me That and he didnt even know the answer.... I Worked and worked but I Couldnt Find It.....
Penny Nom and Victoria West lui répond.
The perimeter of a regular polygon 2007-09-18
Ashwynn pose la question :
why does the area of regular polygons with a perimeter of 1000m increase as the number of sides increase?
Stephen La Rocque lui répond.
An irregular lot 2007-09-12
Jay pose la question :
How do I calculate the area of the following Lot?
N 15 degrees 1 minute 22 seconds E for 182.2 feet
S 89 degrees 46 minutes 26 seconds E for 218.4 feet
N 51 degrees 16 minutes 21 seconds E for for 307.1 feet
L= 31.63 feet with a radius of 50 feet

There is a discrepancy between county records and what the landowner is telling me

Stephen la rocque and Harley Weston lui répond.
Perpendicular lines 2007-09-06
dinesh pose la question :
we know that for two perpendicular lines m1*m2=-1perpendicular but this is not true for x-axis and y-axis?
Harley Weston lui répond.
The area of a circle knowing only the length of a chord 2007-09-05
James pose la question :
I need some help in the right directions with a problem. I was presented with a problem where I need to find the area of a circle knowing only the length of a chord.

the is a circle in the center of a larger circle (which the size of either could change) the only thing that matter is that the chord is 100 ft long and rests on top of the smaller circle.

Stephen la Rocque and Brennan Yaremko lui répond.
The tangent to y = x^3 at x = 0 2007-09-04
Amit pose la question :
consider the equation = x^3. The equation of tangent to this curve (which is smmetrical in Ist and IVth quadrant) at (0,0) is y=0, which is x-axis. but graphically one can visulize that x-axis intersects the curve, so how can it be the tangent to the curve. Please help.
Harley Weston lui répond.
Is there a way I can calculate correct square footage for odd shapes? 2007-08-31
Tom pose la question :
I manage a golf course, and we are about to over seed the course for the upcoming winter months. We will be over seeding all greens, fairways, and tee boxes. Noone of the area that we will be placing seed on is a regular shape. The closest shapes they would be is a circle, rectangle or square, but most of them are all odd CURVED shapes. I know what the equation is for Square ft., and I know how many Square ft. are in an acre, but I need to be precise with the amount of seed I order. Is there a way I can calculate correct square footage for odd shapes.
Harley Weston lui répond.
Can you solve this by factoring? 10r^2 - 35r = 10 2007-08-12
Aranxa pose la question :
how do u solve this equation by factoring:
10r^2 - 35r = 10

Stephen La Rocque lui répond.
Angular speed 2007-08-05
Virginia pose la question :
the engine of a sports car rotates at 5,000 revolutions per minute (rpm). calculate the angular speed of the engine in radians per second. use 2n radians = 1 revolution
Stephen La Rocque lui répond.
All five angles of a pentagon ABCDE are equal 2007-08-04
Jerry pose la question :
All five angles of a pentagon ABCDE are equal. The diagonals AC, AD and BD have the same length. Prove that all five sides of the pentagon are equal in length.
Penny Nom lui répond.
Using the chain rule to solve a derivative 2007-07-29
Charles pose la question :
I need to find the derivative fo the following function.   _______________________
\/ ______________
  \/ (x - 1) / (x + 2) + 1

Stephen La Rocque lui répond.
Equation of a perpendicular line 2007-07-28
Marina pose la question :
Find the equation, is standard form, with all integers coefficients, of the line perpendicular to x+3y=6 and passing through (-3,5)
Stephen La Rocque lui répond.
Multiplcation of two negative numbers 2007-07-26
Brett pose la question :
Someone asked a question about multiplication and division of two negative numbers yielding a positive result here: http://mathcentral.uregina.ca/qq/database/QQ.09.99/butler1.html I was not fully happy with the explanation b/c I want to give me daughter a real-world example and I can't seem to find one.

The following illustrates why multiplying negative numbers has become difficult to explain:

2 X 2 = 4

----(-4)---(-2)---0---2---4
In this example we start with 2 and then want 2 more of them. When we move across the number line from 2 to our answer, which is four, we have moved only 2 units to the right.

-2 X -2 = 4

----(-4)---(-2)---0---2---4
In this example we start with -2 and then want -2 more of them. When we move across the number line from -2 to our answer, which is four, we have moved 6 units to the right.

How can the phenomenon of multiplying two negative numbers being more powerful than multiplying two positive numbers be explained? -Brett

Stephen La Rocque and Harley Weston lui répond.
Guessing multiple choice answers 2007-07-24
carla pose la question :
You are taking a multiple choice quiz that consist in 3 questions, each question has 3 possible answers only one is correct. To complete the quiz you randomly guess the answer to each question . Find the probability of guessing exactly 2 answer correctly. b) at least to answer correctly . c)less than two answer correctly
Stephen La Rocque lui répond.
A complex number in polar form 2007-07-23
roland pose la question :
write the given complex number z in polar form lzl(p+qi) where lp + qil=1 for 3 - 4i.
Harley Weston lui répond.
f(x) = (x^4) - 4x^3 2007-07-22
Michael pose la question :
I'm a student who needs your help. I hope you'll be able to answer my question. Here it is: Given the function f(x)=(x^4)-4x^3, determine the intervals over which the function is increasing, decreasing or constant. Find all zeros of f(x) and indicate any relative minimum and maximum values of the function.
Any help would be appreciated. Thank you for your time.

Harley Weston lui répond.
A triangular trough 2007-07-19
janine pose la question :
A trough is formed by nailing together, edge to edge, two boards 11 ft. in length, so that the right section is a right triangle. If a 15 gallon of water are poured into the trough and if the trough is held level so that a right section of water is an isosceles right triangle, how deep is the water? (231cu.in=1 gal.)
Penny Nom lui répond.
Fractions of fractions 2007-07-18
Bridget pose la question :
Sam and Mary each owned one-half stock in a printing company. Sam sold 2/3 of his stock to Mary. What fractional part of the printing business does Mary now own?
Penny Nom lui répond.
Calculating the area (acreage) of a four sided lot 2007-07-18
A property owner pose la question :
I have a real estate property and the lot size is something I need to find out. I know the lengths of the four sides, but it isn't a rectangle, it is an odd shape. How do I find the acreage?
Stephen La Rocque lui répond.
The area of a triangular pyramid 2007-07-18
Jhezelle pose la question :
What is the formula of finding the surface area of a triangular pyramid?
Penny Nom lui répond.
A normal to a curve 2007-07-16
Samantha pose la question :
The function f is defined by f:x --> -0.5x^2 + 2x + 2.5
Let N be the normal to the curve at the point where the graph intercepts the y-axis. Show that the equation of N may be written as y = -0.5x + 2.5.
Let g:x--> -0.5x + 2.5
(i) find the solutions of f(x) = g(x)
(ii) hence find the coordinates of the other point of intersection of the normal and the curve

Penny Nom lui répond.
The isosceles triangle of largest area with perimeter 12cm 2007-07-16
sharul pose la question :
find the dimension of isosceles triangle of largest area with perimeter 12cm
Harley Weston lui répond.
Implicit Derivatives 2007-07-13
Charles pose la question :
I need help computing y' by implicit differentiation the question is: y^2 + x/y + 4x^2 - 3
Stephen La Rocque lui répond.
Derivative of a Function 2007-07-09
Bob pose la question :
What is the derivative of the function a sub n = [n/(n+1)]^n ?
Stephen La Rocque lui répond.
Finding the radius of an inscribed circle 2007-07-05
Maria pose la question :
I need to find the radius of a circle which is inscribed inside an obtuse triangle ABC. I know all the angles and all the lengths of the triangle.
Stephen La Rocque and Chris Fisher lui répond.
Using calculus to prove the formula for the area of a triangle 2007-07-04
Apratim pose la question :
Using calculus how can one show that the area of any triangle is 1/2 times its base times its height?
Stephen La Rocque lui répond.
An irregular hexagon 2007-07-03
Rebecca pose la question :
I need to find the shape of an irregular hexagon, but only have the lengths four of the sides.

It actually looks like a rectangle with a trapezoid on top of it.

The bottom of the rectangle is 150'. The sides are 75'. The length of the top is 130'. I do not have the lengths of the two smaller sides.

Is there any way to find the area of this shape?

Penny Nom lui répond.
A cylindrical slab of concrete 2007-06-30
Kenn pose la question :
I have a tank that needs a base of cement put in it. I dont know how to do this. The tank is 8 feet across and I want the cement to be 5 or 6 inches deep. Can you please tell me how much cement I need to do this.
Harley Weston lui répond.
The number of blocks on the face of a pyramid 2007-06-27
Heather pose la question :
I was wondering if there was an equation in order to calculate the number of blocks on the face of a pyramid. For example:

X
X X
X X X

Is there a specific formula in order to calculate that there are 6 blocks here? Thank You, Heather

Harley Weston lui répond.
Log base 2 of log base 2 of x 2007-06-27
alex pose la question :
y = log base 2 of lag base 2 of x The slope of the tangent to the given curve at its x-intercept is..?
Harley Weston lui répond.
Using Heron's Formula to help maximize the area of a triangle 2007-06-27
Claire pose la question :
Given one side of a triangle is 4 cm and the ratio 1:3 for the other 2 sides. What is the largest area of the triangle?
Stephen La Rocque and Harley Weston lui répond.
The area of an irregular polygon 2007-06-27
chris pose la question :
finding square footage of a polygon with unequal measurements
Stephen La Rocque and Harley Weston lui répond.
sin|x| and cos|x| 2007-06-25
Mac pose la question :
Can anyone tell me whether sin|x| and cos|x| is differentiable at x=0 ? As far as i know, cos(x) and sin(x) is differentiable at all x.
Penny Nom and Stephen La Rocque lui répond.
Limits as x approaches a constant 2007-06-25
Mac pose la question :
can you please tell me what is the reason they say "denominator is a negative quantity" in the solution 11 and "denominator is a positive quantity" solution 10 ??
If i guess correctly, for solution 10, its because of x^2 in the denominator.

Penny Nom lui répond.
Exponential form: x^y 2007-06-22
Kishor pose la question :
whats the easy way to calculate X raised to Y where y is much greater than x.
Stephen La Rocque lui répond.
Simultaneous equations : the Elimination method 2007-06-21
Patricia pose la question :
I need to find the value of X and Y using the Elimination method.

5/x + 3/y=4
25/x-2/y=3

Stephen La Rocque lui répond.
Simultaneous inequalities 2007-06-18
Freddy pose la question :
Watson Electric has production facilities in Valley Mills, Marlin,and Hillsboro. Each one produces radios,stereos,and TV sets. There production capacities are

Valley Mills: 10 radios, 12 stereos, and 6 TV sets per hour Marlin: 7 radios, 10 stereos, and 8 TV sets per hour Hillsboro: 5 Radios, 4 Stereos, amid 13 TV sets per hour

QUESTION

How many hours should each plant be scheduled to fill an order of 1095 radios, 1230 stereos, and 1490 TV sets?

Stephen La Rocque lui répond.
What is the name of the shape that is 3D and irregular? 2007-06-14
alan pose la question :
What is the name of the shape that is 3D and irregular?
Stephen La Rocque and Penny Nom lui répond.
Circumscribing a golden cuboid with a sphere: surface areas 2007-06-14
Ainslie pose la question :
A golden cuboid is defined as a rectangular prism whose length, width and height are in the ratio of phi : 1 : 1/phi.
Prove that the ratio of the Surface Areas of the golden cuboid to that of the sphere that circumscribes it is Phi : Pi.

Stephen La Rocque lui répond.
Angles of depression 2007-06-13
Phonda pose la question :
The pilot of a small private plane can look forward and see the control tower for a small airstrip. Beyond that is a large factory that is 3 milies from the airstrip. The angles of depression are 12.5 degrees and 4.8 degrees respectively. Find the airplane's altitude, to the nearest ten feet.
Stephen La Rocque lui répond.
Two tangent lines to y=x^3 2007-06-07
stephanie pose la question :
find the equations of two tangent lines to the y=x^3 function through the point (2,8)
Penny Nom lui répond.
Solving a quadratic equation using the Quadratic Formula 2007-06-05
Stuart pose la question :
Solve the following quadratic equation: 3xsquared-5x-4=0
Stephen La Rocque lui répond.
Finding the area of a lot without knowing the angles 2007-06-05
Cristin pose la question :
I have a land-locked piece of land. I need to know the square footage in order to determine the acreage. My deed only gives the lengths of its five sides.
Stephen La Rocque lui répond.
What angle does the tire turn in 60s? 2007-06-04
San pose la question :
An automobile, travelling at 80km/h, is towing a trailer. The wheels of the trailer are 40cm in diameter. Through what angle does the tire turn in 60second?
Stephen La Rocque lui répond.
The volume of a triangular prism 2007-06-01
Hannah pose la question :
How do I find the area of the base of a triangular prism, so that I can find the volume by using the equation on the attachment. The formula is that volume equals one half times (base times height) times triangle height. Thank you for your help. Hannah
Penny Nom lui répond.
The lcm of polynomials 2007-05-31
devin pose la question :
x^3-8 , x^3-4, x^2+4x+4
Penny Nom lui répond.
The number of grams of sulfuric acid 2007-05-31
cookie pose la question :
Hi,
Can you please let me know if I've done this correctly?

Calculate the number of grams of sulfuric acid in 0.500L of battery acid if the solution has a density of 1.28 g/mL and is 38.1% sulfuric acid by mass.

Stephen La Rocque lui répond.
Water tank volume 2007-05-29
Cecelia pose la question :
I HAVE A SEMI CIRCULAR TANK AGAINST A WALL MEASURING 6'10" ACROSS THE WIDEST PART AND 3' FROM THE WALL TO THE FRONT OF THE TANK. THE DEPTH IS 14". PLEASE HELP ME TO CALCULATE THE VOLUME OF WATER IN THE TANK.
Stephen La Rocque lui répond.
The limit of a rational function 2007-05-28
Imad pose la question :
     3 _______ 3 _______
lim \/ 1 + x -    \/ 1 - x
x->0 ---- ----------------------------
              x

Penny Nom lui répond.
A circular blob of molasses 2007-05-28
Julie pose la question :
A circular blob of molasses of uniform thickness has a volume of 1 m^3. The thickness of the molasses is decreasing at a rate of 0.1 cm/hour. At what rate is the radius of the molasses increasing when the radius is 8 m?

Thanks,
Julia

Penny Nom lui répond.
Discovering the incircle of an irregular polygon 2007-05-25
Joaquim pose la question :
I've been searching in some books and many websites, but I couldn't find a formula or algorithm for discovering the incircle of an irregular polygon, could you please help me?
Walter Whiteley lui répond.
System of equations 2007-05-24
Chris pose la question :
Find all real solutions (x,y,z,w) of the system of equations:
2y= x + x/17, 2z= y + y/17, 2w = z + z/17, 2x= w + w/17

Penny Nom lui répond.
Mulch price comparison (price per cubic yard versus price per cubic foot) 2007-05-23
Dianna pose la question :
This is a 2-fold question. I am getting some mulch that is measured in 1 cubic yard for $25.00. Walmart has a 2 cubic bag for $3.00. I need to cover a 376 sq ft area. I need to know which is the best deal and how much I need to buy to cover the 376 sq ft.
Stephen La Rocque lui répond.
Set up two simultaneous equations 2007-05-21
Admire pose la question :
The cost of producing windscreen wipers blades at a factory ais partly fixed (due to operating overheads) and partly dependent on the number of blades produced. It costs $300 to produce 1000, and $600 to produce 5000 blades. How much would it cost to produce 24000 blades?
Penny Nom lui répond.
Weight per area conversions 2007-05-13
Dermot pose la question :
A lawn weedkiller to be spread at the rate of 35g per Sq meter. My new spreader is calibrated in lbs per 1000 sq ft !!!!
Penny Nom and Stephen La Rocque lui répond.
Null hypothesis 2007-05-13
Dani pose la question :
A poll of 1068 American voters indicated that 48% supported te Democratic candidate for presidency. At the 0.05 level of significance, test the claim that less than 50% of American voters preferred the Democratic candidate.
Penny Nom lui répond.
Pattern for a truncated cone 2007-05-11
Mike pose la question :
I have been trying to get this cone flat so I can build this column. Can you please help me so I can figure this out? Thanks for your help.
Stephen La Rocque lui répond.
Probability 2007-05-07
Danielle pose la question :
A box contains 3 defective lights and 5 non-defective lights. The lights are tested one at a time without replacement. What is the probability that the 3 defective bulbs will be found in the first 3 tests? is it 1/56 by chance?
Stephen La Rocque lui répond.
Optimization - carrying a pipe 2007-05-05
A student pose la question :
A steel pipe is taken to a 9ft wide corridor. At the end of the corridor there is a 90° turn, to a 6ft wide corridor. How long is the longest pipe than can be turned in this corner?
Stephen La Rocque lui répond.
Edging surrounding a round pool 2007-05-03
Carol pose la question :
Hello, I am new at this and very rusty on my math. I am getting a 24 Ft. round pool and would like to put 2ft width stone (small) edging around it. How much would I need to buy. I have to buy it by the yard (cubic yard) I am not looking for an exact, just an approx. even would be great. Thanks Carol
Stephen La Rocque lui répond.
Two concentric circles form an annulus 2007-05-02
A student pose la question :
In the diagram below, two concentric circles form an annulus. The vertical line is tangent to the inner circle, and forms the diameter of a third circle.

Explain why the areas of the annulus and third circle are the same.

Penny Nom lui répond.
Angular velocity 2007-05-01
Bruce pose la question :
a tachometer of a Ford Explorer reads 2,100 rpm at 60 mph. Find the equivalent angular velocity in degrees per minute and radians per minute
Stephen La Rocque lui répond.
A tugboat's speed 2007-04-30
Amanda pose la question :
a tugboat must travel 24 miles against a 4 mile per hour current on the Potomac River and return. At what constant speed must the tugboat travel to make the trip in 12 hours. Round answer to the nearest tenth mph.
Stephen La Rocque lui répond.
Finding out if two line equations are parallel or perpendicular 2007-04-28
Vincent pose la question :
Can you determine if the two lines are parallel, perpendicular, or neither 1/2x+4y=3/5 y/1 + 10/5= 8x I think there perpendicular but how would you solve the equations
Stephen La Rocque lui répond.
Maximize the volume of a cone 2007-04-27
ashley pose la question :
hello, I've been stumped for hours on this problem and can't quite figure it out. The question is: A tepee is a cone-shaped shelter with no bottom. Suppose you have 200 square feet of canvas (shaped however you like) to make a tepee. Use calculus to find the height and radius of such a tepee that encloses the biggest volume. Can you help??
Stephen La Rocque and Penny Nom lui répond.
A cylinder inside a sphere 2007-04-25
Louise pose la question :
i need to find the maximum volume of a cylinder that can fit inside a sphere of diamter 16cm
Penny Nom lui répond.
Constructing an octagonal deck around a circular pool 2007-04-20
Cliff pose la question :
[I am building an] octagonal desk encompassing 17 foot diameter circle for pool. I have seen other octagonal calculations but none of these tell me how much allowance for a circle to fit within the octagon without losing the circle edge can anyone help thanks cliff
Stephen La Rocque lui répond.
Liquid is being poured into the top of a funnel 2007-04-19
neroshan pose la question :
Liquid is being poured into the top of a funnel at a steady rate of 200cm^3/s. The funnel is in the shape of an inverted right circular cone with a radius equal to its height. It has a small hole at the bottom where the liquid is flowing out at a rate of 20 cm^3/s. How fast is the height of the liquid changing when the liquid in the funnel is 15 cm deep? At the instant when the height of the liquid is 25cm, the funnel becomes clogged at the bottom and no more liquid flows out. How fast does the height of the liquid change just after this occurs?
Penny Nom lui répond.
Minimum cost for a fixed volume 2007-04-18
James pose la question :
My question goes: A silo is to be constructed and surmounted by a hemisphere. The material of the hemisphere cost twice as much as the walls of the silo. Determine the dimensions to be used of cost is to be kept to a minimum and the volume is fixed.
Penny Nom lui répond.
The area of a regular octagon 2007-04-17
Adam pose la question :
I need to find the area of a regular octagon with side length of 12.5m
Penny Nom lui répond.
Simultaneous equations 2007-04-16
kyrie pose la question :
simultaneous equation 4x + 3y = 21 2x * y = 8
Penny Nom lui répond.
The second derivative 2007-04-14
Gerry pose la question :
In mathematical context,what do you understand by the term "Second Derivative"
Penny Nom lui répond.
The area of a regular hexagon 2007-04-14
Chantel pose la question :
Could you please explain to me why the formula for a regular hexagon is 3√3/2 multiplied by side²? Thank you very much.
Stephen La Rocque lui répond.
The area of an irregular pentagon 2007-04-14
Steven pose la question :
I have been given a coursework in which i have to investigate various shapes to find the one that would give the largest area with 1000m perimeter. I'm stuck in a part where i have to calculate the area of a irregular pentagon. i don't know where to start from. I can make up the length of all the sides (5 sides), so long they add up to 1000, but i'm not quite sure what to do after that.
Penny Nom lui répond.
What is the limit of 3.x^(3/x) as x approaches +infinity? 2007-04-11
Teodora pose la question :
What is the limit of 3.x^3/x as x approaches +infinity ?
Haley Ess lui répond.
I am filling a hole in the shape of a cube 2007-04-11
Bethany pose la question :
If I am filling a hole in the shape of a cube with 64 cubic yards of dirt, how many feet deep will my hole be.
Stephen La Rocque lui répond.
Find the volume of the solid 2007-04-07
tricia pose la question :
a solid is constructed so that it has a circular base of radius r centimeters and every plane section perpendicular to a certain diameter of the base is a square, with a side of the square being a chord of the circle. find the volume of the solid at first i thought the length of a side of the square would be r, but that isn't awlays be true- only when the chord is in the center. so how can i solve this without any values? i dont understand the relationship between the chord and radius, except that the radius intercepts the chord at the midpoint. i know i hav to take the integral to get the volume, but how do i even find the area of one of the squares? please help, thanks, tricia
Penny Nom lui répond.
3 divided by 3 to it's fifth root 2007-04-06
Annie pose la question :
How do I transform the equation 3 divided by 3 to it's fifth root to simple radical form (getting the radical out of the denominator)?
Penny Nom lui répond.
The area of a parcel of land 2007-04-03
Ricky pose la question :
I need the attached land parcel (#0436) converted to acreage. This would be of great help to me. Thanks, Ricky
Penny Nom lui répond.
Interior and exterior angles 2007-04-02
Anuj pose la question :
If in a regular polygon, each exterior angle is twice the interior angle,find the number of sides?
Leeanne Boehm lui répond.
Golf schedule 2007-03-30
Jim pose la question :
I am organising a golf break for 12 golfers playing in 3 fourballs over 5 days. I would like to maximise the number of different golfers each player can play with. Any help would br greatly appreciated.
Penny Nom lui répond.
6- team schedule for softball 2007-03-29
Angie pose la question :
I have 6-teams that can only play 3 days a week. Each team must get 18 games in for the season. I have to split them being the home team and the away team sometimes.
Penny Nom lui répond.
A beam on a lighthouse 2007-03-28
Lisa pose la question :
A beam on a lighthouse 2000 metres away from the nearest point P on a straight shoreline revolves at the rate of 10 pi radians per minute. How fast is the beam of the light moving along the shoreline when it is 500 metres from P?
Stephen La Rocque lui répond.
Slopes of perpendicular lines 2007-03-27
Mykolyn pose la question :
How do you find the slope of a line that is perpendicular to the graph of: y= -1/2x+4?
Leeanne Boehm and Sara Ulmer lui répond.
Schedule for a 10 team league 2007-03-27
Pete pose la question :
I don't know if this question involves a math solution, but I am running out of ideas. I have a 10 team league. Each team must play each other once only. I can't seem to match this up and get stuck at the end. Please help. Thanks, Pete
Penny Nom lui répond.
A triangular prism 2007-03-26
Tom pose la question :
Hi, I need to find a container in the shape of a triangular prism that will fit four table tennis balls (this is for a math project!). These balls have a diameter of 4cm, so a radius of 2. I know that the formula for finding an incircle from a triangle is:
radius of incircle = 2area of triangle / perimeter of triangle
But I need to know the length of one side of the triangle from the incircle (the triangle needs to be equalateral). Can you help me find a formula for this? Thanks so much! Tom

Penny Nom lui répond.
The area of an irregular pentagon 2007-03-26
grace pose la question :
hi I am doing graphics and need to find the area of the section given to us (it is just drawn). the sides measure 15m, 35.4m, 5m, 17.8m, 35.7m. the angle between the 15m side and the 35.4m side is 17degrees, the angle between the 35.4m side and the 5m side is 73degrees. the angle between the 5m side and the 17.8m side is 151degrees, the angle between the 17.8m side and the 35.7m side is 90degrees and the angle between the 35.7m side and the 15m side is 95degrees. if you could tell me the area i would be extremely grateful. thanks very much.
Penny Nom lui répond.
y = sin(2x) 2007-03-22
bader pose la question :
sin(2x) find dx/dy
Penny Nom lui répond.
Which solid figure has 4 more edges than vertices? 2007-03-22
dawn pose la question :
which solid figure has 4 more edges than vertices?
Penny Nom lui répond.
The Reuleaux Triangle 2007-03-22
Dani pose la question :
Hi! My question is: Determine how the area of the Reuleaux Triangle of width h compares to the area of a circle of width h? Thanks! Dani
Penny Nom lui répond.
28 golfers playing golf over 7 days 2007-03-20
ian pose la question :
I have 28 golfers playing golf over 7 days I would like everybody to play with each other at least once Is there a schedule? Thank you Ian
Penny Nom lui répond.
A one pass calculation of standard deviation 2007-03-19
Murtaza pose la question :
Hi I searched the listings but couldn't find what i am looking for. I have a dataset and i am allowed only 1 pass through it. At the end of this pass i must have the mean and standard deviation. Calculating the mean is easy as also demonstrated in your listings. I think the same can be done for standard deviation as well. But the doubt that i have is this :- Will the calculated standard deviation be sequence dependent ? i.e. if i change the order in which the data items enter the system, will the std deviation also change ?? Thanks in advance Murtaza
Penny Nom lui répond.
A line that is perpendicular to another line 2007-03-16
Jesse pose la question :
How do you write an equation of a line that is perpendicular to y+2x+3 and passes through (3,4); I don't understand the steps of how to do this????
Stephen La Rocque and Penny Nom lui répond.
Triple angle tangent formula 2007-03-15
sam pose la question :
Hi I am trying to derive a triple angle formulae for tan. I know i need to use compound and double angle formulae but am finding it difficult to "clean" up my fraction to get the triple angle formulae can you show me a worked derivation?! thanks
Penny Nom lui répond.
The area of a property 2007-03-14
Greg pose la question :
Hi my name is Greg. I have a piece of property I am trying to buy, but need to find out the square feet of it first. The four sides are unequal lengths and none at a right angle or parallel. Here are the measurement in feet. The base of the square is 29.12 feet in width.
The right side of the square is 44.33 feet high.
The top of the square is 28.80 feet in width.
The left side of the square is 46.20 feet high.
I have not done this math in ages and really need your help.

Harley Weston lui répond.
A regular quadrilateral 2007-03-13
Jackie pose la question :
How can the measure of each angle of a regular quadrilateral be determined?
Haley Ess lui répond.
The period of a simple pendulum 2007-03-10
Melissa pose la question :
The period of a simple pendulum of length L feet is given by: T=2pi(sqrt(L/g))seconds. It is assumed that g, the acceleration due to gravity on the surface of the earth, is 32 feet per second per second. If the pendulum is a clock that keeps good time when L=4 feet, how much time will the clock gain in 24 hours if the length of the pendulum is decreased to 3.97 feet? (Use differentials and evaluate the necessary derivative at L=4 feet.) Answer is in seconds. Melissa
Penny Nom lui répond.
An irregular pentagon 2007-03-08
kelly pose la question :
If I have a pentagon with 2 sides of length 5 and one with a length of 2, what are the lengths of other 2 sides?
Stephen La Rocque lui répond.
Find the length of the belt 2007-03-08
Helen pose la question :
Find the length of the belt
Penny Nom lui répond.
An irregular polygon 2007-03-06
Dike pose la question :
How do i construct an irregular polygon
Penny Nom lui répond.
Finding the x-intercept 2007-03-02
Ash pose la question :
Y = -1/100X^2 =27/10X +0 = 0 THE Y INTERCEPT IS 50 HOW DO I FIND THE X INTERCEPT
Gabriel Potter lui répond.
Simultaneous equations with fractions 2007-02-28
Alyca pose la question :
Hello Math Central, I am a grade 10 student taking Academic math. Our unit right now is method of substitution and elimination. I'm stuck on this one question that I've been doing forever. Please help =)

*For this equation I have to do method of elimination, but it's so much harder with fractions...could some one please explain to me how to do it step by step?*
x y 2
-- - -- = - --
3 6 3

x y 1
-- - -- = 1---
12 4 2

Steve La Rocque and Ashley Mang lui répond.
sin(3a), cos(3a) and tan(3a) 2007-02-28
mailene pose la question :
hi, i...indeed,to..need..your..help
how..cn..i..prove..this,formula???
sin3a=3sina-4sin^3a
cos3a=4cos^3-3cosa
tan3a=3tan-tan^3a /1-3tan^2a

the..symbol...^is..the..expOnent

Haley Ess and Penny Nom lui répond.
The grade 6 math curriculum 2007-02-24
Ajay pose la question :
Do you have a list of topics a 6th grade students should learn in the 6th grade? I am "rusty" in math and would like to assist my son... Example: if an icetray has X cc of water and is split into 8 parts...what is the volume of each part...is this truly 6th grade level math?
Pam Fowler and Penny Nom lui répond.
At what rate is the area of the triangle changing? 2007-02-24
mac pose la question :
two sticks 3.5 feet long are hinged together and are stood up to form an isosceles triangle with the floor. The sticks slide apart, and at the moment when the triangle is equilateral, the angle is increasing at the rate of 1/3 radian/sec. At what rate is the area of the triangle increasing or decreasing at that moment?
Mac

Penny Nom lui répond.
Surface area of a triangular prism 2007-02-21
Katrina pose la question :
What is the formula to finding the surface area of a triangular prism?
Gabriel Potter lui répond.
Find the area of the triangle 2007-02-20
Christina pose la question :
Graph the function f(X)= x+1/x-1 and graph the tangent line to the function at the points A:(2,3) and B:(-1,0). The point of intersection of the two tangent lines is C. Find the area of the triangle ABC.
Stephen La Rocque lui répond.
What constitutes a face for a geometric solid? 2007-02-14
Erica pose la question :
What constitutes a face for a geometric solid? We have been getting conflicting answers from different textbooks. Some teachers are saying that cylinders and cones do not have faces. I'm thinking that cylinders have 2 and cones have 1. Please enlighten me. Thanks!
Walter Whiteley lui répond.
a specific 3-d shape 2007-02-12
Robby pose la question :
What three dimensional shape has 6 vertices, 5 faces, and two congruent triangular bases? I can't find the answer to this question anywhere. Your help will be appreciated. Thanks, Robby Craig
Pam Fowler, Gabe Potter and Walter Whiteley lui répond.
Exponential form of complex numbers 2007-02-12
Austin pose la question :
When dealing with imaginary numbers in engineering, I am having trouble getting things into the exponential form. The equation is -1+i now I do know that re^(theta)i = r*cos(theta) + r*i*sin(theta). Just not quite understanding the order of operations. Thanks
Penny Nom lui répond.
Simultaneous equations with envelopes 2007-02-08
Mick pose la question :
There were 17 envelopes bought, some were brown, some were white. The brown envelopes cost one cent more per envelope than the white ones. The total cost was 80 cents. How much of each type of envelope was bought? --Many thanks!
Stephen La Rocque lui répond.
Multiplying and dividing by zero 2007-02-05
colin pose la question :
i would like to if u multiply 1,000,000 by 0 does it equal o or 1.

if you have a number eg 15689 and you divide by o what would the answer be is it 1

Penny Nom lui répond.
The substitution method 2007-01-31
Victoria pose la question :
how do i solve this problem using the substitution method?
2x-5= -14
-7x+14y= -5

Stephen La Rocque lui répond.
The elimination method 2007-01-31
Addrianna pose la question :
x-2y=2
3x-5y=7

Stephen La Rocque lui répond.
Water in a triangular trough 2007-01-30
Trina pose la question :
the trough is 5 feet long and its vertical cross sections are inverted isosceles triangles with base 2 feet and height 3 feet. water is draining out of the trough at a rate of 2 cubic feet per minute. at any time t, let h be the depth and v be the volume of water in the trough.
a. find the volume of water in the trough when it is full
b. what is the rate of change in h at the instant when the trough is .25 full by volume?
c. what is the rate of change in the area of the surface of the water at the instant when the trough is .25 full by volume?

Penny Nom lui répond.
The altitude of regular n-side pyramid 2007-01-26
Wendimu pose la question :
The altitude of regular n-side pyramid is half of it's base side.What is the angle between the lateral face and it's base?
Penny Nom lui répond.
A regular pentagonal pyramid 2007-01-25
Vickifazio pose la question :
a regular pentagonal pyramid has a base perimeter of 1,085 ft. How long is one side of the base?
Stephen La Rocque lui répond.
Which lets you put equal groups together? 2007-01-20
Kathy pose la question :
Which lets you put equal groups together? division, multiplication, subtraction, estimation
Stephen La Rocque lui répond.
Irregular multi sided polygon 2007-01-20
Graham pose la question :
I have an irregular polygon. I know the length of all the sides and know approximately all the internal angles. Is there a formula or table that can calculate the area for me?
Steve La Rocque, Chris Fisher and Penny Nom lui répond.
The area of an irregular pentagon 2007-01-18
Ranjit pose la question :
I would like to find the area of an irregular pentagon where the total perimeter is 1000m. Is there a Heron's formula for this?
Stephen La Rocque lui répond.
Angular speed 2007-01-18
Cristina pose la question :
A car is moving at a rate of 50 miles per hour, and the diameter of its wheels is 2.5 feet.
a) Find the number of revolutions per minute the wheels are rotating.
b) Find the angular speed of the wheels in radians per minute.

Stephen La Rocque lui répond.
Comparing two fractions 2007-01-18
Kayla pose la question :
Why does eight over twelve compared to one half work when you use cross multiplication.
Penny Nom lui répond.
The volume of a frustum of a pyramid 2007-01-17
Sam pose la question :
Find the volume of a frustum of a pyramid with square base of side b, square top of side a, and height h.
Penny Nom lui répond.
A line perpendicular to another line 2007-01-13
Dusty pose la question :
Find an equation for the line perpendicular to: 3x - 4y + 1 = 0 passing through (4,7)
Leeanne Boehm and Penny Nom lui répond.
Products with symbols 2007-01-09
Jennifer pose la question :
Find the product of each
(8x)(-4)
(3x)(5y)(7z)
(8x^3y)-5X^2)
(6a)(3a)(-b^2)

Penny Nom lui répond.
Integrate x^8 (x^8 + 2)^2 ((x^8 + 2)^3 + 1)^4 2007-01-09
James pose la question :
How do you integrate x^8 (x^8 + 2)^2 ((x^8 + 2)^3 + 1)^4
Penny Nom lui répond.
Angular speed 2007-01-06
Robert pose la question :
A rear wheel and tire on a John Deere 8520 farm tractor has a 39-in. radius. Find the angle (in radians) through which a wheel rotates in 12 sec if the tractor is traveling at a speed of 22 mph.
Penny Nom lui répond.
What are the dimensions of the most economical container? 2007-01-04
Ashely pose la question :
A cylindrical container costs $2.00 per square foot for the sides and $3.00 a square foot for the top and bottom. The container must hold 100 cubic feet of material. What are the dimensions of the most economical container.
Stephen La Rocque lui répond.
What size pulley would I need? 2006-12-29
Chris pose la question :
If I have a motor that's spinning at 950 RPM's with a pulley that's 6in diameter with a belt running to a generator, What size pulley would I need on the generator to make it spin at 3600 RPM
Penny Nom lui répond.
A regular polygon inscribed in a circle 2006-12-19
Katy pose la question :
If a regular hexagon is inscribed in a circle of radius 6.72 centimeters, find the length of one side of the pentagon. How would I got about explaining this?
Penny Nom lui répond.
1, 3, 6, 10... 2006-12-15
Lisa pose la question :
What is the nth term for the sequence, '1, 3, 6, 10...' and could you say how you got the anwser.
Stephen La Rocque and Penny Nom lui répond.
Parallel and perpendicular lines 2006-12-07
Valeria pose la question :
I m having trouble with parallel and perpendicular lines a problem i need help with is y=6 find the slope if al line parallel to the graph of each equation
Stephen La Rocque lui répond.
Rolle's Theorem 2006-12-07
Erika pose la question :
If f(x) = (x^2)(square root of [3-x]) on the interval [0,3] is given, Does Rolle's Theorem apply? If yes, find any values of c such that f '(c)=0
Penny Nom lui répond.
A 15% tip 2006-12-06
Danielle pose la question :
When she eats at a restaurant,Viviana like to leave a 15% tip,multiplying the price of the meal by 0.15. Franklin usually leaves a 20% tip,multiplying the price by 0.20.They both round up to the nearest $0.05. How much tip would Viviana leave for a $24.85 meal?
Penny Nom lui répond.
The length of each side of a regular n-gon 2006-12-06
Shannen pose la question :
The problem is: Find the length of each side of a regular n-gon when a=80ft, n=20ft, and A is approx. 20,000 square feet.
What do "a" and "n" stand for and how do I find the side length of an n-gon?

Penny Nom lui répond.
A Norman window 2006-11-30
Joe pose la question :
a norman window is a rectangle with a semicircle on top. If a norman window has a perimeter of 28, what must the dimensions be to find the maximum possible area the window can have?
Stephen La Rocque lui répond.
Sally plays roulette 2006-11-21
Sherry pose la question :
Sally plans to bet 100 bets of $1.00 each on red roulette. the probability of the ball landing on red is 18/38 what is the probability that Sally will win at least half the time
Penny Nom lui répond.
Calculating volume of concrete, dirt, sand, anything. 2006-11-17
Debbie pose la question :
[This article contains a special calculator to help you find the volume (for example, the cubic yards) of material required to fill (or empty) a space if you know the length, width and depth.]
We are wanting to pour a 24'x30' slab that is 5" deep. How many cubic yards of cement do we need?

Penny Nom lui répond.
Tangent lines 2006-11-09
Melissa pose la question :
let f be a function with f(1)=4 such that for all points (x,y) on the graph of f the slope is given by (3x^(2)+1)/(2y)

a.)Find the slope of the graph of f at the point where x=1.
b.)Write an equation for the line tangent to the graph of f at x=1 and use it to approximate f(1.2)
c.) Find whether f is concave up or concave down when x=1. Is your answer in part b an overestimate or an underestimate?

Stephen La Rocque lui répond.
Least common multiple 2006-11-08
Melenie pose la question :
I have a daughter in 5th grade she can't seem to grasp the concept of finding LCM's does anyone have a quick easy way for her to learn?????
Stephen La Rocque lui répond.
Simultaneous equations 2006-11-06
An other pose la question :
e^2y-x+2=0
ln(x+3)-2y-1=0

Penny Nom lui répond.
ESL and math-speak 2006-11-03
Matt pose la question :
My name is Matt and I am an ESL teacher in Korea. Recently I have begun tutoring a student who will be going to Canada soon as a 9th grade student. She is a very capable student and is concerned that her mathematical abilities outstrip her ability to communicate in English. I am writing mainly to inquire what level of math-speak I should teach to this student. For instance: should I be explaining geometric terms, or simply algebra? And if algebra, should I include complex numbers and the quadratic formula? If there are things that she will be taught in Canada, I'd prefer her to learn them firsthand from a math teacher. However, knowledge that she will be presumed to have I would like to teach.
Karen McIver lui répond.
Pulleys and belts 2006-10-31
Lynn pose la question :

A belt has a linear speed of 32 mph. It is spinning three pulleys.
Pulley A has diameter 14 in. radius = 7 inches
Pulley B has diameter 26 in. radius = 13 inches
Pulley C has radius 3 ft. radius = 36 inches

Angular speed in Radians per min for
I got
Pulley A= .08 rads/min Pulley B= .04 rads/min Pulley C=.01 rads/min

How many cm would pulley C travel in 14 seconds?


Penny Nom lui répond.
Induction 2006-10-31
Ross pose la question :
Suppose that A and B are square matrices with the property AB= BA. Show that AB^n = B^n A for every positive integer n.
Stephen La Rocque and Penny Nom lui répond.
The area of an irregular hexagon 2006-10-30
Ashish pose la question :
I want to find out area of an irregular HEXAGON in square feet, whose sides are as follows,
Stephen La Rocque lui répond.
The surface area of a triangular prism? 2006-10-29
Amanda pose la question :
What is the formula for the surface area of a triangular prism?
Haley Ess lui répond.
Water is being pumped into the pool 2006-10-24
Jon pose la question :
A swimming pool is 12 meters long, 6 meters wide, 1 meter deep at the shallow end, and 3 meters deeps at the deep end. Water is being pumped into the pool at 1/4 cubic meters per minute, an there is 1 meter of water at the deep end.

a) what percent of the pool is filled?

b) at what rate is the water level rising?

Stephen La Rocque lui répond.
The area of a regular octagon 2006-10-22
Farhana pose la question :
I have a regular octagon with sides of 0.8m and I need to find the area,
Karen McIver and Penny Nom lui répond.
An approximation 2006-10-22
Ellen pose la question :
consider the curve -8x^2 +5xy+y^3 +149 =0 Write an equation for the line tangent to the curve at (4, -1) use this equation to approximate the value of K at the point (4.2, K)
Penny Nom lui répond.
How fast is the water level rising when the water is 1 meter deep? 2006-10-19
Don pose la question :
The cross section of a 5-meter trough is an isosceles trapezoid with a 2-meter lower base, a 3-meter upper base and an altitude of 2 meters. Water is running into the trough at a rate of 1 cubic meter per minute. How fast is the water level rising when the water is 1 meter deep?
Stephen La Rocque lui répond.
Lattice multiplication 2006-10-16
Patti pose la question :
I am a parent of a 5 th grader, and am a bit loss in the "new" math. I know how to multiply using the "old fashion" method. But, could you please explain lattice multiplication.
Diane Hanson lui répond.
How many feet tall is the tunnel 2006-10-14
Charles pose la question :
A tunnel in the Smoky Mountains is semicircular. At a distance of 12 feet from the center of the tunnel, the tunnel has a height of 16 feet. How many feet tall is the tunnel at its center?
Penny Nom lui répond.
Is a circle a regular shape? 2006-10-10
David pose la question :
Is a circle a regular shape or an irregular shape?
Stephen La Rocque lui répond.
Angular speed 2006-10-04
Jack pose la question :
A horse on a merry-go-round is 7m from the center and travels at a speed of 10km/h. What is its angular speed?
Stephen La Rocque lui répond.
Family of lines 2006-09-29
Sasha pose la question :
could a "Family of Lines" have both parallel and intersecting equations that aren't related directly but otherwise related by each other to create a curve
Chris Fisher lui répond.
Least common multiple 2006-09-25
A student pose la question :
What is the LCM of 10,45 and 38?
Stephen La Rocque lui répond.
A rectangular tank 2006-09-19
Milton pose la question :
What size rectangular tank will I need to hold 841.5 gallons of water. The water level is 3'-0" from the bottom of tank.
Penny Nom lui répond.
The volume of a triangular box 2006-09-12
Irene pose la question :
How do I find the volume of a triangular box?
Penny Nom lui répond.
The velocity of a pendulum, part II 2006-09-07
Erin pose la question :
We saw the question in your database about the velocity of a pendulum swinging.....It is the same exact question....but there is another question......it says....

"estimate the instantaneous rate of change of d with respect to t when t = 1.5. At this time, is the pendulum moving toward or away from the wall? Explain."

Harley Weston lui répond.
How many microliters (ul) are there in 0.5 ml? 2006-08-29
Monica pose la question :
How many microliters (ul) are there in 0.5 ml?
Stephen La Rocque lui répond.
The area of an irregular octagon 2006-08-23
Billy pose la question :
How do you figure the area of an irregular octagon? Measurements are 4 equal sides of =BD", and 4 equal sides of =BE". It measures 1 1/2" = from flat to flat.
Penny Nom lui répond.
Differentiate Y= sin3x + cos7x 2006-08-22
james pose la question :
Differentiate the function of x using the basic rules.

Y= sin3x + cos7x

Stephen La Rocque lui répond.
A golf schedule for three days 2006-08-18
Jon pose la question :

I was wondering if I could trouble you to assist me with the following: I am working on a playing schedule for 28 golfers. 7 groups of 4. I have 3 golf days to schedule.

Ideally, I would like to schedule all 28 golfers in 7 different groups of 4 on each day. Here is the catch.....no golfer in any group can be grouped togther more than once. Every group of 4 each day will have 4 new golfers that have never played together before. Is this possible?


Penny Nom lui répond.
How fast is the water level rising 2006-08-12
Erin pose la question :
Water runs into a conical tank at the rate of 9ft3/min. The tank stands point down and has a height of 10 ft. and a base radius of 5 ft. How fast is the water level rising when the water is 6 ft. deep? (V=1/3 pi r2 h).
Penny Nom lui répond.
A problem with exponents 2006-08-09
A student pose la question :
(8a to the negative 2 b cube c to the negative 4/4a squared b to the negative 3 c squared) to the negative 2
Stephen La Rocque lui répond.
The area of a lot 2006-08-05
Lyn pose la question :
I have no idea how to find the square feet of an uneven polygon. I am in the process of making a land agreement and need to know.
Please tell me how to figure out the footage and acreage of land as follows.
road-front 360'
back 360'
left side 220'
right side 180'

Penny Nom lui répond.
Gallons of water in a well 2006-07-31
Martha pose la question :
I have a shallow water well that is 30 feet in depth.

I have 8 feet of water in the well.

The water well is a 30 inch in diameter.

My question : How many gallons of water is in my tank?

Penny Nom lui répond.
An Integral 2006-07-30
Aniket pose la question :

I am Aniket studing in 12 th standard At Mumbai
I have following integration problem please give me a solution

integral of 1/under root of (5x2 - 2x) dx


Penny Nom lui répond.
Minimizing a cost 2006-07-25
Edward pose la question :
The cost of running a car at an average speed of V km/h is given by c= 100 + (V2 / 75) cents per hour. Find the average speed (to the nearest km/h) at which the cost of a 1000 km trip is a minimum.
Stephen La Rocque lui répond.
A perpendicular intersection of two barrel vaults 2006-07-21
Neal pose la question :
I'm wanting to build a series of architectural models of different roman and medieval buildings out of cardboard. Once I have perfected the models I want to print them out on card stock so that school kids (or anyone else) can make the buildings.
A feature of many of these models is the cross or groin vault (a perpendicular intersection of two barrel vaults).
A single barrel vault is easy to imagine as a plane (a rectangular piece of cardboard) that will be folded into a semi-circular arch.
The intersection of a second barrel vault and this one is presenting me with problems. The second plane needs to have an ellipse cut into it so that when it is folded into the arch, it will mate up with the curve of the first barrel vault.
Given that the two pieces of card have identical widths (and therefore identical arcs in cross section) is there a way to calculate the ellipse that needs to be cut so that it can be cut before the second arch is folded?

Edward Doolittle lui répond.
Calculating the belt length of a three pulley system 2006-07-16
Mark pose la question :
I have a 3 pulley system with sides abc and pulleys ABC. Pulley A has radius of 10cm, pulley B has radius of 20cm, and pulley C has radius of 3cm. The side lengths are: (center to center of pulleys) between pulleys AB = 75cm, between pulleys BC = 100cm, and between pulleys AC = 50cm. I set these side lengths up as (according to law of sines and cosines) a = 100cm, b = 50cm, and c = 75cm. What is the length of the belt required for this system? I need to know how I would set this problem up and solve.
Stephen La Rocque lui répond.
How many bags will it take to cover the area? 2006-06-14
Ken pose la question :
I have area of 160 sq ft. I want to put mulch in the area 2 inches thick. The bag of mulch comes with 2 cu ft of mulch per bag. How many bags will it take to cover the area?
Penny Nom lui répond.
How many yards will I need? 2006-06-06
Melba pose la question :
I'm am trying going to put mulch around the tree in my front yard. I am going for 6" deep and 4069.44 sq. in. around.
How many yards of dirt will I need?

Penny Nom lui répond.
Math manipulatives 2006-05-29
Mike pose la question :
Our school is looking to order a large amount of math manipulatives. Do you have a reliable, reasonably priced source that is not producing 'kits' that are intended for specific math programs like Math Makes Sense?
Diane Hanson and Walter Whiteley lui répond.
Simultaneous Equations 2006-05-24
Angie pose la question :

Question: solve the equations
2x-3y-z=0
3x-2y+z=-5
x+3y-2z=14

for x,y,z


Stephen La Rocque lui répond.
differentiate the volume of a cylinder with V respect to h 2006-05-24
A student pose la question :
differentiate the volume of a cylinder with V respect to h
Stephen La Rocque lui répond.
The area of a triangle 2006-05-21
A student pose la question :
how do you find the area of a triangle?
Penny Nom lui répond.
integral of tan^4 x 2006-05-14
Aqil pose la question :
integral of tan4 x
Penny Nom lui répond.
Twenty golfers 2006-05-08
Mona pose la question :
We have twenty golfers and four rounds of golf. We would like to figure out how to make foursomes so that each golfer plays with as many different golfers as possible.
Penny Nom lui répond.
A hallway in an old house 2006-05-06
Sabrina pose la question :
I am trying to determine how many square feet is in a room that is not square or rectangular. It has basically 5 walls of different lengths, it is acutely considered a hallway in an old house, how can I figure out the footage, the Length times Width requires to much wasted materials
Penny Nom lui répond.
Rate of ladder falling 2006-04-30
Harsh pose la question :
A ladder 4 m long rests against a vertical wall. If the bottom of the ladder slides away from the wall at a speed of 30 cm/s, how quickly is the top of the ladder sliding down the wall when the bottom of the ladder is 2 m from the wall?
Stephen La Rocque lui répond.
The perimeter of a regular octagon 2006-04-20
Martin pose la question :
I would like to make an octagon out of 2x4 lumber. I know that the lumber needs to be cut at 67.5 degree angles, but how do I determine the length of each piece if I want to make, say, a 2.5 ft diameter octagon?
Stephen La Rocque lui répond.
Mayan multiplication 2006-04-16
Marca pose la question :
I'm researching the Mayans, and my advisor, Dr. McDonald, stated that you may know whether the Mayans knew how to multiply. I'm finding conflicting information on the internet, and anything you could provide would be greatly appreciated.
Chris Fisher lui répond.
Given three angles and a side 2006-04-09
Jon pose la question :
How do you figure out the length of all sides of a scalene triangle if given the measure of all angles, and one side?
Stephen La Rocque lui répond.
How many people are in the race? 2006-04-07
Marty pose la question :
You're in a race with 1/5 of the racers ahead of you and 5/6 of them behind you. How many people are in the race?
Penny Nom lui répond.
Mastering the multiplication tables 2006-04-04
Ellie pose la question :
I need a detailed study plan fro helping my son master his multiplication tables.
Grade: 3 elementary

Paul Betts lui répond.
2x+5y=3 And -x+3y=-7 2006-04-03
Lloyd pose la question :
simplify 2x+5y=3 And -x+3y=-7
Penny Nom lui répond.
Find the point of inflexion for the curve y = e^x/(x^2-1) 2006-03-31
Sam pose la question :
Hi, i am trying to find the point of inflexion for the curve y = ex/(x2-1) and i got a really complex expression for y". I can't seem to solve x4-4x3+4x2+4x+3=0 so does that mean there is no point of inflexion?
Penny Nom lui répond.
A fence around a pen 2006-03-30
Daryl pose la question :
I hope you can help me out with the attached problem, It has been driving me crazy.
Stephen La Rocque and Penny Nom lui répond.
cos(3X) 2006-03-29
Joshua pose la question :
I'm having trouble proving that cos(3X)=cos3X -(cosX)(sin2 X)
Penny Nom lui répond.
Angular velocity 2006-03-28
Ben pose la question :
How do you (a) express angular speed in terms of linear speed and (b) express linear speed in terms of angular speed?
Stephen La Rocque lui répond.
The area of a block of land 2006-03-26
Ronald pose la question :

I have a building block of land with four unequal sides and only one right angle. I want to know the total area (in metres) and how the calculations were carried out.

The four sides are: Rear of property: 9.14 metres
left side: 36.9 metres
Right side: 32.61 Metres
front to street: 27.43 Metres

The front to street and right side constitute a right angle. but there are no others.


Penny Nom lui répond.
A 3-dimensional star 2006-03-15
Rachel pose la question :
I am trying to figure out the name of a figure that consists of 8 (I think) square pyramids. There is a net drawing of this figure somewhere and I can't find it because I don't know the name. When all 8 pyramids are connected, they form a '3-dimensional star'. We did this project when I was 14 years old and I am now a 27 year old teacher and I would love to do this project with my kids.
chris Fisher lui répond.
2x+3y=0 and 3x-y=0 2006-03-14
Lisa pose la question :
my name is lisa I am doing math for work and i have a math problem to solve and i need help with it here is the question 2x+3y=0 and 3x-y=0 this is one question can you help me please
Penny Nom lui répond.
The quadratic formula 2006-03-03
Miriam pose la question :
Please show work and indicate "answer", using the quadratic formula -

(x+2) (2x+3) = 6

penny Nom lui répond.
The volume & surface area of a rectangular pyramid. 2006-02-27
Cheryl pose la question :
My daughter is working on a math project & we are having a hard time finding the volume & surface area of a rectangular pyramid. Can you help us with this?
Penny Nom lui répond.
A triangle of 50p pieces 2006-02-22
Stuart pose la question :

Ok, so i am collecting 50p pieces and arranging them on my desk in the shape of a triangle.
eg
50p
50p 50p
50p 50p 50p
50p 50p 50p 50p

I want to work out how much money I'm saving just by knowing how many rows of coins there are. If i can work out how many coins there are just by knowing how many rows I have I can just divide by 2 to find out the amount in dollars.


Penny Nom lui répond.
The Least Common Multiple of two primes 2006-02-21
Kristine pose la question :
Is the Least Common Multiple to two prime numbers always their product? Why or Why not?
Stephen La Rocque lui répond.
Golfing with 8 or 12 2006-02-13
Carolyn pose la question :
I have a couple of questions that my dad asked me and I do not think it can be done in the manner he wants it to come out to.

He is going on a golf trip in June. There are going to be either 8 or 12 players playing 4 rounds of golf.

The questions that he says will not work and I do not know how to figure it out either is

4 Rounds of golf is for sure. 18 hole Rounds each

4 Players to a group

Either 8 or 12 players can every player play with every player in a group at least once during those 4 rounds.

If this can be done can you give me the combinations for both 8 and 12 , so I can help out my dad. Along with how you you figure this out.

Penny Nom lui répond.
Is there a solid shape with 5 faces, 8 edges, and 4 vertices? 2006-02-11
Annie pose la question :
Is there a solid shape with 5 faces, 8 edges, and 4 vertices?
Penny Nom lui répond.
An irregular quadrilateral 2006-02-04
Christopher pose la question :
Is there such a shape as an irregular quadrilateral with 4 equal sides?
Penny Nom lui répond.
The box of maximum volume 2006-02-01
Elizabeth pose la question :
A box factory has a large stack of unused rectangular cardboard sheets with the dimensions of 26 cm length and 20 cm width.
The question was to figure what size squares to remove from each corner to create the box with the largest volume.
I began by using a piece of graph paper and taking squares out. I knew that the formula L X W X H would give me volume. After trial and error of trying different sizes I found that a 4cm X 4cm square was the largest amount you can take out to get the largest volume. My question for you is two parts

First: Why does L X H X W work? And second, is their a formula that one could use, knowing the length and width of a piece of any material to find out what the largest possible volume it can hold is without just trying a bunch of different numbers until you get it. If there is, can you explain how and why it works.

Penny Nom lui répond.
The median 2006-01-27
Wael pose la question :
" median"; what does it mean and how do we calculate it?
Penny Nom lui répond.
Subdividing a polygon into triangles 2006-01-26
Adam pose la question :
is there an algorithm to divide a regular polygon into N equilateral triangles having the same area (no limit on N), or if not, an algorithm to divide a regular polygon into N triangles having the same shape and size?
Chris Fisher lui répond.
One boundary of a pond is parabolic in shape. 2006-01-20
Glenn pose la question :
One boundary of a pond is parabolic in shape. The boundary passes through the points A(-20,45), B(40,40) and E(30,35). The equation of the parabola is of the form y=ax2+bx+c. Find the equation of the parabola and the coordinates of the vertex of the parabola. Any assistance you could provide would be greatly appreciated.
Penny Nom lui répond.
An irregular octagon 2006-01-20
Robert pose la question :
I am building a poker table which is in the shape of an irregular octagon. I know the table measures 72 inches long and 48 inches wide with two parallel straight sides of equal length and six smaller sides of equal length ( three at each end of the table), what I don't know are the lengths of the any of the sides.
Harley Weston lui répond.
GCF, LCM, primes and the ladder method 2006-01-07
Linda pose la question :
How would I teach both finding the GCF and LCM with prime factors...I recall the ladder method vaguely.???
Penny Nom lui répond.
Differentiation, powers and logs 2006-01-06
Claudia pose la question :

Question: how do I find the derivative of

x* ln(x+(e^2))^2

x^lnx

x^(e^(-x^2))


Penny Nom lui répond.
Two related rates problems 2005-12-29
Shimaera pose la question :

#1. A manufacturer determines that the cost of producing x of an item is C(x)=0.015x2+12x+1000 and the price function is p(x)=250+2x/10. Find the actual and marginal profits when 500 items are produced.

#2. At 9 a.m a car is 10km directly east of Marytown and is traveling north at 100 km/h. At the same time, a truck leaves Marytown traveling east at 70 km/h. At 10 a.m, how is the distance between the car and the truck changing?


Penny Nom lui répond.
The Mean Value Theorem 2005-12-22
Candace pose la question :
Verify that the function satisfies the hypotheses of the Mean Value Theorem on the given interval. Find all numbers "c" that satisfy the Mean Value Theorem. 11. f(x)=3x2 + 2x +5 [-1, 1]
Penny Nom lui répond.
Simultaneous Equations 2005-12-21
Matt pose la question :
I have these two equations,
336 = 60a + 10b
and
432 = 84a + 6b
Am I right in saying both a and b are 4.8?

Penny Nom lui répond.
A max-min problem 2005-12-16
Julie pose la question :
A car travels west at 24 km/h. at the instant it passes a tree, a horse and buggy heading north at 7 km/h is 25 km south of the tree. Calculate the positions of the vessels when there is a minimum distance between them.
Penny Nom lui répond.
Josh and John were both exposed to the flu 2005-12-16
Dudley pose la question :
Josh and John were both exposed to the flu. John has a 25% chance of getting it, while Josh has a 75% chance of getting it. What are the chances that at least one of them has the flu?
Penny Nom lui répond.
Mrs. Faria lives on an island 2005-12-15
Julie pose la question :
Mrs. Faria lives on an island 1 km from the mainland. She paddles her canoe at 3 km/h and jogs at 5 km/h. the nearest drug store is 3 km along the shore from the point on the shore closest to the island. Where should she land to reach the drug store in minimum time?
Penny Nom lui répond.
A loan of $50,000 2005-12-14
Fre pose la question :
A loan of $50,000 taken today is payable within five years.
a. determine the annual payments within to be made to repay the loan if interest is charged at a rate of 10% compounded annually
b. show the amortization schedule

Penny Nom lui répond.
LCM 2005-12-12
Alex pose la question :
what is the LCM of 210 and 54 and the LCM of 42 and 126
Penny Nom lui répond.
Area of a lot 2005-12-03
Ben pose la question :

How do you calculate the area of the following Lot?

I figured the following angles from the deed, which read:

N 86 degrees, 45 minutes E, for 322 feet.

S 10 degrees, 30 minutes W for 113 feet.

N 84 degrees, 30 minutes W for 368 feet.

N 50 degrees, 42 minutes E for 76 feet.

N 40 degrees, 40 minutes E for 15 feet.

There is a discrepancy between two surveyors and I'd like to figure out how to calculate the Area of such a shape.


Harley Weston lui répond.
I = PRT 2005-11-16
Ryan pose la question :
Use the formula to find the value of the variable that is not given:
I=PRT;I=$2880, R=0.08, P=$12,000

Penny Nom lui répond.
Shapes 2005-11-12
Tonya and Hailey pose la question :
MY THIRD GRADERS TEACHER HAS SENT HOME HOMEWORK ASKING FOR THEM TO FIND REAL-WORLD OBJECT IN MANY DIFFERENTS SHAPES A FEW HAVE US STUMPED
Penny Nom lui répond.
Notation for the second derivative 2005-11-08
Mussawar pose la question :
my question is d/dx( dy/dx) = d2y/dx2. why it is not equal to d2y/d2x.
Penny Nom lui répond.
The number is increased by the sum of its digits 2005-11-07
Ernesto pose la question :
The number 1 is written on a blackboard. After each second the number on the blackboard is increased by the sum of its digits. is it possible that at some moment the number 123456 will be written on the blackboard?
Claude Tardif lui répond.
Velocity and acceleration 2005-10-27
Candace pose la question :
When taking the integral of the position function, you get the velocity function, and the same for velocity to acceleration. So when you do each of these, you get a function. But when you integrate on a graph, you get an area under a curve. The area is un units squared- where do the units go when you make it an equation? How can a function be an area?
Harley Weston lui répond.
(x/1+x)^x 2005-10-23
A student pose la question :
LIMIT x-.->0 (x/1+x)x how to solve it.
Penny Nom lui répond.
Least common multiple 2005-10-22
Dayna pose la question :
I'm stumped! I can't seem to come up with a least common multiple of 22 and 25. Is there one?
Penny Nom lui répond.
A triangular pyramid 2005-10-22
Stacee pose la question :
When stacking oranges to form a triangular pyramid, each orange touching 3 oranges below it. In order to have a pyramid with 6 layers, how many oranges should be on the bottom layer?
Penny Nom lui répond.
Can we take the derivative of independent variable 2005-10-18
Mussawar pose la question :
why we take derivative of dependent variable with respect to independent variable .can we take the derivative of independent with respect to dependent.if not why.
Walter Whiteley lui répond.
The area of an irregular semi-circle 2005-10-14
Bob pose la question :
Is there a way to compute the area of an irregular semi-circle i.e. one in which the arc length is not determined by the diameter; and is therefore not technically any part circular - yet still possessing an arched side?
Penny Nom lui répond.
Simultaneous equations 2005-10-13
Daniel pose la question :
5x + 3y = 22 4x - 7y = -20
Penny Nom lui répond.
A cone with vertex (1,1,2) 2005-09-26
Brandon pose la question :
Find the equation of a double cone with vertex (1,1,2) and which intersects the xy plane in a circle of radius 4.
Penny Nom lui répond.
U'(X) - U(X) = 0; U(0) = 2 2005-09-23
David pose la question :
Out of interest could you please answer the following questions?

U'(X) - U(X) = 0; U(0) = 2

and

U''(X) - U'(X) = 0; U'(0) = U(0) = 2


Harley Weston lui répond.
Constructing figures 2005-09-20
Kim pose la question :
I would like to know how to draw different shapes:
Regular Octagon with sides of length, 1 unit
Equilateral Triangles, with sides of length 1 unit
Regular Hexagons, with sides of length 1 unit
Isoseles Triangles, with hypotenuse of length 1 unit

Penny Nom lui répond.
A point is moving on the graph of x^3 + y^2 = 1 in such a way that 2005-09-17
Gina pose la question :
A point is moving on the graph of x3 + y2 = 1 in such a way that its y coordinate is always increasing at a rate of 2 units per second. At which point(s) is the x coordinate increasing at a rate of 1 unit per second.
Penny Nom lui répond.
How do you differentiate y=(x)^(x^x)? 2005-09-14
Calebius pose la question :
How do you differentiate y=(x)(xx)?
Penny Nom lui répond.
Seven consecutive reds in roulette 2005-09-04
Joe pose la question :
I know that the odds are a little less than 50% for either black or red comming up on a spin of the roullette wheel. My question is what are the odds of red or black comming up 7 times in a row?
Penny Nom lui répond.
The area of a lot 2005-08-29
Richard pose la question :
My wife and I are interested in buying property in Idaho but the owner can't give us a square footage of the lot. The dimensions are as follows:

121.0 on the left side
157.0 on the right side
135.0 on the bottom
162.0 on the top

The bottom right corner of the lot is a true right angle, the rest are not.

Penny Nom lui répond.
My fantasy football league 2005-08-16
Jim pose la question :
I have to make a schedule for my fantasy football league. There are 10 teams in the league and we play a 13 week season. The challenge is we want to play each team only once during the first 10 weeks. This way no team will play another team twice without having played each team at least once. Is this possible?
Penny Nom lui répond.
At what rate is the circumference of the circle increasing? 2005-08-08
John pose la question :

A mathematics professor is knitting a sweater. The main part of the sweater is knit in a large spiral, ending up with a diameter of 30 inches. She knits at a constant rate of 6/7 square inches per minute.

1. At what rate is the circumference of the circle increasing when the diameter is 2 inches?

2. How long will it take her to finish this piece of the sweater?


Penny Nom lui répond.
A golf outing for 16 golfers 2005-08-05
Bob pose la question :
I have a golf outing fast approaching and I’m having a problem matching the golfers so that everyone plays a round of golf with every other golfer in the group.

My problem is this: I have 16 golfers playing 5 rounds of golf. I would like a different foursome for every round of golf, that is each round (5 total) an individual is playing with 3 other players he hasn’t played with as of yet. Can you crunch the numbers and get back to me?

The way I see it, we have 20 combinations (5 rounds x 4 golfers) with numbers 1 thru 16.

Chris Fisher lui répond.
Three prime numbers p,q and r, all greater than 3, form an arithmetic progression: 2005-07-18
Ladis pose la question :
Three prime numbers p,q and r, all greater than 3, form an arithmetic progression: p=p, q=p+d and r= p+2d. Prove that d is divisible by 6.
Chris Fisher lui répond.
The area of a lot 2005-07-17
John pose la question :
A plot of land has the following dimensions: 391 ft. x 757 ft. x 208 ft. x 788 ft. Q: How many acres is this?
Harley Weston lui répond.
A lighthouse is located on a small island,... 2005-07-14
Brittnee pose la question :
A lighthouse is located on a small island, 3 km away from the nearest point P on a straight shoreline, and its light makes four revolutions per minute. How fast is the beam of light moving along the shoreline when it is 1 km from P?
Penny Nom lui répond.
The sum of the digits of 2^100 2005-06-11
Richard pose la question :
The sum of the digits was calculated for the number 2100, then the sum of the digits was calculated for the resulting number and so on, until a single digit is left.
Penny Nom lui répond.
The volume of a hopper 2005-05-28
Brian pose la question :
I would like to know the volume of this rectangular hopper. can you help
Penny Nom lui répond.
Logarithmic differentiation 2005-05-23
Richard pose la question :
I need to convince myself that I understand the process of differentiating y=xx.
The specific question is that if I have to take the logarithm of both sides of the equation how can differentiate the following?
y= {(x+2)(x+2)}/{(x+1)(x+1)} - {(x+1)(x+1)}/(xx),
I have an idea that the differential of this fairly complex function is itself ... am I right or wrong.

Penny Nom lui répond.
A triangular lot 2005-05-21
Linda pose la question :
I am looking to purchase a piece of property, and I need some help trying to figure out the square footage of the land. I know that normally if I multiply the length and width I could get my answer. However, this piece of land is an odd shape and has 3 measurements. Can you please help me.
The measurements for this piece of land is:

97.8'
201.2'
139.7'

Penny Nom lui répond.
Divisibility of a^2 + b^2 2005-05-16
Ampa pose la question :
given natural numbers a and b such that a2+b2 is divisible by 21, prove that the same sum of squares is also divisible by 441.
Penny Nom lui répond.
L'hopital's rule 2005-05-15
Abraham pose la question :
Find the limit of [(1/(x+4))-(1/4)]/x as x approaches zero. How do you use l"hopital's rule to find this limit. I know how to do it with multiplying everything by 4(x+4), and getting the answer, -1/16.But how do you apply derivatives with l'hopitals rule to this type of problem?
Penny lui répond.
sin x + sin 2x + sin 3x + sin 4x = 0 2005-05-10
Elia pose la question :
I tried many times, but can't get to solve the following question:
sin x + sin 2x + sin 3x + sin 4x = 0

Chris Fisher lui répond.
Some triangle problems 2005-05-06
A student pose la question :
1.use the heron's formula to find area of triangle ABC to nerest tenth

A.a=21 b=28 c=11

B.a=23 b=28 c=12

2.Find the nearest tenth the altitude of the longest side if a=3 b=3 c=5

3.the length of each side of a rhombus is 10 cm the length of one diagonal is 12 cm find the area to the nearest square centermeter

Penny Nom lui répond.
A Taylor series for ln(x) 2005-04-16
Anood pose la question :
i have to represent ln(x) as a power series about 2

i`m not getting the final answer which is ln 2+ sigma (((-1)(n+1)/ (n*2n))*(x-2)n). i don`t get the ln 2 part

i show you my trial

f(x)= ln x.

f-(x)=(1/x) .

f--(x)= (-1/x2)*1/2!

f---(x)= (2/x3)*1/3!

f----(x)= (-6/x4)* 1/4!

so the pattern shows me that f(n)= ((-1)(n+1))/xn *n)

so f(2)= sigma ((-1)(n+1))/2n *n) *(x-2)n

so as you see i don`t get ln 2


Penny Nom lui répond.
A flaw in a problem 2005-04-15
Bryce pose la question :

Question:

(x2-x2) = (x2-x2)
x(x-x) = (x+x)(x-x) [divide both sides by (x-x)]
x = x + x
x = 2x [divide both sides by x]
2 = x/x = 1

Where is the flaw in this problem?


Paul Betts lui répond.
Bayes Theorem 2005-04-03
wei pose la question :

In the January 11,1988,issue of the Oil&Gas Journal, R.A.Baker describes how the Bayesian approach can be used to revise probabilities that a prospect field will produce oil. In one case he describes, geological assessment indicates a 25% chance the field will produce oil. Further,there is an 80% chance that a particular well will strike oil given that oil is present on the prospect field.

  1. Suppose that one well is drilled on the field and it comes up dry. What is the probability the prospect field will produce oil?
  2. If two wells come up dry, what is the probability the field will produce oil?
  3. The oil company would like to keep looking as long as the chances of finding oil are greater than 1%. How many dry wells must be drilled before the field will be abandoned
  4. If the first well produces oil,what is the chance the field will produce oil?

Andrei Volodin and Penny Nom lui répond.
A wrestling schedule 2005-03-08
Ed pose la question :
Hello my name is Ed and I am a couch of a wrestling team in New Jersey. I was doing searches for formulas on how to make a matrix for team sports and it seems to be hard to find. I came some how to your site and thought I'll give it a try. Here is what we need or are trying.
Claude Tardif lui répond.
A nine week schedule 2005-03-08
Greg pose la question :
I'm trying to build a nine week schedule where 10 teams play each other once throughout the season.

I make it all the way to week nine, then I start having all sorts of conflicts.

HELP!!


Claude Tardif lui répond.
The area of a quadrilateral 2005-02-27
Jonathan pose la question :
I want to know how to find the area of a quadrilateral.
Penny Nom lui répond.
Understanding fractions 2005-02-11
C.J. pose la question :
I have some questions about the lesson, "Understanding Fractions" by Diane Hanson, Regina Catholic Schools. How successful was the lesson? Are there any changes you would recommend?
Diane Hanson lui répond.
A seating arrangement 2005-02-03
Kay pose la question :
If you have 5 married couples, how many arrangements can be made if they must sit across from each other?
Penny Nom lui répond.
Differentiating F(x,y) = 0 2005-01-23
Jacob pose la question :
In calculus, we often mention to the students that if F(x,y) = 0, then we can differentiate both sides and still get an equality. The problem is that we can't perform the same operation on F(x) = 0, say x = 0, otherwise 1 = 0, which is absurd. What is the reason?
Walyer Whileley and Harley Weston lui répond.
Finding the missing side of a triangle 2005-01-20
Jason pose la question :
I found a geometry problem that reads as follows:In triangle ABC,
Penny Nom lui répond.
A line from the center of the patch to the periphery 2005-01-01
Sandrine pose la question :
I am currently researching a patch disease of grasses. These patches are roughly circular. I need a term for a line from the center of the patch to the periphery. Since the patches are not perfectly circular, my supervisors tell me I cannot use the word 'radius'. What else could I use?
Denis Hanson and Harley Weston lui répond.
The area of a triangle 2004-12-30
Perry pose la question :
What is the area of a triangle with dimensions 3"X5"X7"? Could you provide the formula?
Penny Nom lui répond.
A Reuleaux triangle 2004-12-22
Bob pose la question :
I am trying to remember the name given to the regular figure constructed from three arcs. The figure is like an equilateral triangle except with arcs for each side.
Chri Fisher lui répond.
Three calculus problems 2004-12-09
Ashley pose la question :
Hi, I am having a lot of trouble with three calculus questions and was wondering if you could help
Penny Nom lui répond.
A belt around two pulleys 2004-12-07
Ian pose la question :
a belt is stretched around two pulleys whose centers are d units apart and whose radii are R and r respectively (obviously R+r<d). the challenge is to find the length of the belt, l as a formula in terms of R, r, and d only.
Penny Nom lui répond.
The lcm of 5, 3, 4, 7 and 2 2004-11-05
Christine pose la question :
what is the lcm of 5,3,4,7,2
Penny Nom lui répond.
The lcm of 1 and -1 2004-11-03
Nayiyan pose la question :
I would like to know what is the LCM of (-1) and (+1) is.
Penny Nom lui répond.
Multiplying two algebraic fractions 2004-10-31
A parent pose la question :
Multiply:

x+2 x²-4
----- x -------
x-2 x²+x-2


Harley Weston lui répond.
GCD and LCM 2004-10-31
Cartalina pose la question :
how do you calculate the "positive difference between the GCF and LCM of two numbers"?
Penny Nom lui répond.
The combined force of two vectors 2004-10-30
Brian pose la question :
Two teams are playing push ball with a large 8 foot diameter ball. One team exerts a force represented by the vector a = 2i + -5j, and the other team exerts a force represented by the vector b = -8i-3j.

1.Determine the direction of movement of the ball if the i axis is due east.

2.Determine the combined force magnitude.

Harley Weston lui répond.
The force of a 5000 tonne lorry 2004-10-27
Aaron pose la question :
A 50 tonne lorry is parked on a slope with a gradient of 4%. Due to the self weight of the lorry, what is the force

* acting down the slope
* acting normal (at right angles) to the slope

Penny Nom lui répond.
20 Golfers 2004-10-26
Ian pose la question :
I have 20 golfers playing over 6 days I would like to schedule all 20 golfers in 5 groups of 4 on each day here is the catch.... I would like each player to play with each other once during the 6 days.
Chris Fisher lui répond.
Implicit differentiation 2004-10-24
Emily pose la question :
If x^3+3xy+2y^3=17, then in terms of x and y, dy/dx =
Penny Nom lui répond.
sin(3A) 2004-10-20
A student pose la question :
Express sin3A in terms of sinA and cosA.
Penny Nom lui répond.
The volume of a pile of mulch 2004-09-22
Sam pose la question :
Is there a formula to determine the cubic feet of something in a pile. IE- I need to determine the cubic feet of a pile of mulch. The pile comes to a peak, so the length and width decrease as the pile increases.
Penny Nom lui répond.
Finding the height of a triangle 2004-09-14
A student pose la question :
I have to calculate the height of a triangle. The base is 6 (units)say ab One of the top sides is 4 (units)say ca The other side is 2 (units) say cb I don't know the angles.

How to find the height?

Chris Fisher and Penny Nom lui répond.
A schedule for 6 teams 2004-09-10
Greg pose la question :
I have a schedule I need to make of 6 teams. They must play each other once however no team can be the away team twice in a row. There must be 3 home games and 3 away games for each team.
Penny Nom lui répond.
The bar over repeating digits. 2004-09-07
Debbie pose la question :
I would like to know the name given to the bar that is written over the repeating digits of a decimal.
Penny Nom lui répond.
The distance formula 2004-09-04
Joe pose la question :
Why do we use square roots and squares in the distance formula Wholdnt they cancel each other out?
Claude Tardif lui répond.
The integrating factor method 2004-08-05
A student pose la question :
Whilst using the integrating factor method, I am required to integrate a function multipled by another function.

say f(t) = exp(kt) and some other function g(t); where exp = exponential and k is some constant.

Integral f(t)*g(t) dt or Integral exp(kt)*g(t) dt

What would the result of this integral be? I have never met an integral like this before. Would it simply be exp(kt)*g(t)/k? More specifically, the problem and my attempted answer is in PDF format:

In my attempted solution, I am unsure about the last two lines I have written out, as it relates to integrating a function multipled by another function.

Harley Weston lui répond.
Integrating e^sin(x) 2004-08-04
A student pose la question :
I need to know that how to solve the integral " e^sin x",
Harley Weston lui répond.
Differentiation 2004-08-04
A parent pose la question :
I am a parent trying to understand higher level of maths and would be very grateful if you could help with differentiating the following functions, identifying general rules of calculus:

a) f(x)=e^2^xIn(cos(8x))

b) f(x)=secx/SQRTx^4+1

Penny Nom lui répond.
Two vector problems 2004-07-14
Jaye pose la question :
Two force, 120lb and 200lb, act on a body and make a 52 degree angle w/ ea. other. What's the magnitude of the resultant of the forces and what is the measure, to the nearest degree, of the angle that it makes with the 200lb force?
Penny Nom lui répond.
What is the name of a 3D rectangle? 2004-06-07
Jack pose la question :
if a cube is a 3D square, what is the name of a 3D rectangle?
Penny Nom lui répond.
The Parliament of the land of Achronia 2004-05-29
Christine pose la question :
The Parliament of the land of Achronia consists of two houses. The parliament was elected in 2003 for a period of six years beginning on Thursday, the 1st of January 2004, when the two houses had their first sessions. According to the rules, the meetings of the first house must occur every twelve days for the duration of the term, and the meetings of the second house must occur every eighteen days. For example, the second meetings of the first and the second houses were held on the 13th and 19th of January respectively. A new law can be passed on any day when both houses meet, except on a Thursday. On how many days can the parliament members pass new laws during this six year term?
Penny Nom lui répond.
Some trig expressions 2004-05-23
A student pose la question :
Prove:

sin A + sin B = 2sin(A+B/2)cos(A-B/2)

cos A - cos B = -2sin(A+B/2)sin(A-B/2)

cos A + cos B = 2cos(A+B/2)cos(A-B/2)

sin A - sin B = 2cos(A+B/2)sin(A+B/2)

Penny Nom lui répond.
Maximizing the angle to the goal mouth 2004-05-15
Yogendra pose la question :
You are running down the boundary line dribbling the ball in soccer or hockey. Investigate where in your run the angle the goal mouth makes with your position is at a maximum.
Penny Nom lui répond.
Related rates and baseball 2004-04-26
Bethany pose la question :
A baseball diamond is the shape of a square with sides 90 feet long. A player running from second to third base at a speed of 28 feet/ second is 30 feet from second base. At what rate is the player's distance from home plate changing?
Penny Nom lui répond.
A changing rectangle 2004-04-03
A student pose la question :
The width x of a rectangle is decreasing at 3 cm/s, and its length y is increasing at 5 cm/s. At what rate is its area A changing when x=10 and y=15?
Penny Nom lui répond.
Some calculus problems 2004-04-01
Weisu pose la question :

I have questions about three word problems and one
regular problem, all dealing with derivatives.

  1. Find all points on xy=exy where the tangent line
    is horizontal.
  2. The width x of a rectangle is decreasing at 3 cm/s,
    and its length y is increasing at 5 cm/s. At what rate
    is its area A changing when x=10 and y=15?
  3. A car and a truck leave the same intersection, the
    truck heading north at 60 mph and the car heading west
    at 55 mph. At what rate is the distance between the
    car and the truck changing when the car and the truck
    are 30 miles and 40 miles from the intersection,
    respectively?
  4. The production P of a company satisfies the
    equation P=x2 + 0.1xy + y2, where x and y are
    the inputs. At a certain period x=10 units and y=8
    units. Estimate the change in y that should be made to
    set up a decrease of 0.5 in the input x so that the
    production remains the same.

If you could just give me some hints on these
questions, I'd really appreciate it. Thanks!


Penny Nom lui répond.
A partial derivative 2004-03-19
Penny Nom pose la question :
Is it possible to differentiate the following equation, if so could you please explain.

S=SQRT(T-(5/X^2))

I would like the derivative of S with respect to X.

Harley Weston lui répond.
Johann Muller 2004-03-14
A student pose la question :
What major contributions to spherical trigonometry did Johann Muller make?
Penny Nom lui répond.
Volume of a fuel tank 2004-03-01
Mike pose la question :
I have a fuel ank for my big truck that has a radius of 24inches and a length of 65 inches. I am trying to compute the volume of fuel in the tank. I tried pie x the radius squared times the length but no usable results. Can you help me please?
Penny Nom lui répond.
The derivative of x to the x 2004-02-14
Cher pose la question :
what about the derivative of x to the power x?
Penny Nom lui répond.
A pyramid-shaped tank 2004-02-13
Annette pose la question :
The base of a pyramid-shaped tank is a square with sides of length 9 feet, and the vertex of the pyramid is 12 feet above the base. The tank is filled to a depth of 4 feet, and water is flowing into the tank at a rate of 3 cubic feet per second. Find the rate of change of the depth of water in the tank. (Hint: the volume of a pyramid is V = 1/3 B h , where B is the base area and h is the height of the pyramid.)
Harley Weston lui répond.
Angular speed 2004-02-01
Rachel pose la question :
A 4 inch pulley on a motor makes 1500 revolutions/minute. It is connected to an 8 inch saw blade. a. Find the angular speed of the pulley in radians/minute

b. Find the angular speed of the saw in radians/ minute

Harley Weston lui répond.
Sin(3x), cos(3x) and tan(3x) 2004-01-28
Jon pose la question :
What is the identity for cos3x, sin3x, and tan3x? In class, we learned double angel identities and were asked to find out the identity to these three trig functions. If you can help, please do. Also, i know that the cos4x- sin4x is the same as cos2x. Is cos8x-sin8x = cos2x also true? Thank you.s
Chris Fisher lui répond.
A problem with sets 2004-01-20
Jason pose la question :

Given that the universal set S is the set of all sports fans, and

F={x|x is a football fan}
B={x|x is a basketball fan}
H={x|x is a hockey fan}
a)Describe (F^B)' (f intersect b)' in words
b)Draw a Venn Diagram and shade the region that represents the set of football fans or both basketball and hockey fans.


Penny Nom lui répond.
Some trig problems 2004-01-18
Weisu pose la question :

I have some questions about pre-calculus.

(1) (2(cos(x))^2)+3sin(x)-1=0
(2) sin(x)cos(x)=(1/2)
(3) 3sin(x)=1+cos(2x)
(4) tan(x)*csc(x)=csc(x)+1
(5) sin(arccsc(8/5))
(6) tan(arcsin(24/25))
(7) arccos(cos(11pi/6))
the last problem uses radian measure.


Penny Nom lui répond.
Volume of a tube 2004-01-14
Mike pose la question :
where can you find how to get volume of a tube: 3 in across x 3 inch high
5 in across x 5 inch high
same formula for both. ONE will get the other.

Penny Nom lui répond.
Geoboards 2004-01-09
Elissa pose la question :
What is a geoboard? I have a young girl in grade eight math who has a math question using a geoboard.
Diane Hanson lui répond.
The area of a lot 2004-01-09
Lea pose la question :

I have a problem. I am interested in purchasing a piece of property which has the following dimensions. It is 259.10 Feet on the north side x 61.20 Feet on the east side x 217.80 Feet on the south side x 196.70 Feet on the west side. It looks like an irregular rectangle shape. How do you figure out the square footage on this irregular sized property, and what is the correct square footage answer.

I have attached a survey map.


Harley Weston lui répond.
The trapezoidal rule 2003-12-31
Prashanth pose la question :
I have a set of XY co-ordinates(say 49 points),I want to find the area enclosed by the curve joining them and the axes.I do not have an equation for the curve and I am not able to fit a curve given the points.I use the trapezoidal rule (Y1+Y2/2)*(X2-X1), Is it right? can I do this with any other method?
Penny Nom lui répond.
The angles in a regular polygon 2003-12-21
Ernie pose la question :
If i have a measure of one interior angle of a polygon, how can i find the number of sides it has?
Penny Nom lui répond.
The area of a plot of land 2003-12-19
David pose la question :
I have a plot of land that I cannot find the square footage of. Actually my home sits on a large lot. Local building codes require that a home is seated on no less than 7500 square feet of land and have no less than 50ft of road frontage (the rear of the lot has 65ft of frontage on another road). I need to know if I have enough square footage to build a second home on the rear lot. The lot has five sides. Only one angle is a 90 deg. angle. The others I cannot determine. Of the 4 remaining angles, all 4 angles are obtuse, or greater than 90 deg. The dementions are 89.19' x 130' x 118.52' x 65' x 170.94'. The two legs that make the 90 deg angle are 89.19' and 170.94'. The measurements fall in order using the 90 deg angle as the point of origin and proceeding clockwise, 89.19', 130', 118.52', 65', 170.94'. I have used all the skill I can muster, however, it has been a long time since school and regretably I cannot find an answer.
Penny Nom lui répond.
Multiplier and multiplicand 2003-12-17
Karen pose la question :
The K-5 curriculum states that at a grade 5 level students should "understand and use the terms multiplier, multiplicand" The curriculum defines 'multiplicand' as " a number being multiplied by another number. A factor." There is no definition given for 'multiplier' Question: Define these words- are they just other words for 'factors'. If so, how important is it that students use these terms? It would seem like 'factors' would be the better choice of words to use. If multiplcand refers to eg. the first factor in the equation and the 'multiplier' the seond term, (if in fact it does)- is this not confusing for students when we teach them that multiplication is commutative?
Penny Nom lui répond.
The area of a triangle 2003-12-01
A student pose la question :
Find the Area of Triangle ABC
A(-3,2)
B(4,0)
C(0,8)

Penny Nom lui répond.
The volume of an irregular tetrahedron 2003-11-24
Peter pose la question :
How do I calculate the volume of an irregular shaped tetrahedron where:-
side a = 1.4 m
side b = 1.4 m
side c = 1.2 m
and height = 0.75 m at the junction of sides a and b.

Penny Nom lui répond.
A riddle 2003-11-19
Sarah pose la question :
Ok, our teacher gave us this riddle, and I cannot for the life of me figure it out. He said that there are three problems with the following proof:
Penny Nom lui répond.
Divisibility by 7 2003-11-14
A student pose la question :
how do you test a number to see if it is divisible by 7 or not?
Penny Nom lui répond.
A circle around an irregular polygon 2003-11-03
Dale pose la question :
How do I find the properties of a circle that is drawn around an irregular polygon of (n) sides with the lenghts of each side given and all end points of the polygon lye on the circumferance of the circle?
Chris Fisher lui répond.
How much time? 2003-10-29
Lori pose la question :
I am a elementary teacher 3-6 math. We have just implemented the Houghton Mifflin Math curriculum. It seems to be quite difficult for most of our students. In order to get through all the material we need to do a lesson a day. Although the students are not getting the material that fast. Should we move on without all the students understanding the material or should we wait and remediate until they get it and be behind for the year. This isn't just a few kids its most of the kids. We are also suppose to teach to all the standards so if we don't get to them the students will not do well on the standardized tests.
Diane Hanson lui répond.
Maine and Nevada 2003-10-24
Jimmy pose la question :

From 1989 to 1990, the population of Nevada increased b y 157,000, and that of Maine increased by 30,000. In 1990, the population of Nevada was 1,206,152, and that of Maine was 1,233,223. If the populations of the two states continue to increase at the same rates, when will the populations of Nevada and Maine be the same?

Write a verbal model for this problem
Write an equation for the model
Solve the equation and answer the question.


Penny Nom lui répond.
Standard Deviation 2003-10-07
Rebecca pose la question :

I have a task to complete, which is to calculate the mean and standard deviation of something. I have done this but am then asked to write a short explanation of my findings.

I know what the mean is about, and I thought I knew what the standard deviation meant too - shows the variation from the mean. However, on a task I completed earlier the feedback I got said 'you need to tell us that it is talking about the middle 66% of the data' - that has thrown me, I don't understand that. Can anyone help me get my head round this???


Penny Nom lui répond.
The sketch of a graph 2003-10-07
A student pose la question :
I was wondering how do you figure out if a graph has a horizontal tangent line. One of my homework problem was to sketch the graph of the following function; (4/3)x3-2x2+x. I set f''(x) ( the second derivative) of the function equal to zero and got the inflection point:(1/2,1/6). Also i am having trouble finding the concavity for x>1/2 and x<1/2, i am getting a different answer from the back of the book, the graph i draw looks completely different from the correct answer.
Penny Nom lui répond.
Arrangements around a circular table 2003-10-07
Lori pose la question :
How many arrangements can 6 people be seated round a circular table?
Penny Nom lui répond.
Indeterminate forms 2003-10-06
A teacher pose la question :
Is it possible for me to find any geometrical interpretation without using calculus to explain indeterminate forms?
Chris Fisher lui répond.
Functions, graphs and derivatives 2003-10-05
Jathiyah pose la question :
I wanted to know how would you tell (on a graph diplaying two funtions), which funtion is the derivative of the other?
Walter Whiteley lui répond.
The slope of a tangent 2003-10-01
A student pose la question :

find the slope of the tangent to each curve at the given point

f(x)=square root 16-x, where y=5


Penny Nom lui répond.
7,473,000,000 divided by 52000 2003-09-06
A student pose la question :
I am having trouble with a question. 7,473,000,000 divided by 52000. Our calculater gets 147311.5385. I come up with 143711.53846, and then the number starts to repeat itself
Penny Nom lui répond.
A helicopter rises vertically 2003-09-02
Kate pose la question :
A helicopter rises vertically and t seconds after leaving hte ground its velocity is given in feet per second by v(t) = 8t + 40 / (t+2)2 How far above the ground will the helicopter be after 3 seconds?
Penny Nom lui répond.
A 6 team schedule 2003-08-31
A scheduler pose la question :
I saw that someone put on your web site a team schedule and you helped them figure it out.Ý I have 6 teams that want to play each other once, and believe it or not, I cannot figure it out.
Penny Nom lui répond.
The quadratic formula 2003-08-31
Alex pose la question :
I am using Houghton Mifflin's Precalculus with Limits book, 2nd edition. However, the first chapter encompasses Algebra review, and I am stuck on a problem. All that's required is to solve the following and verify using a calculator:
3y2+6y+2=0
I have solved the problem using the quadratic formula, but from what I remember, the quadratic formula is used in the case of equations following the AX2+BX+C=0 pattern. As the problem I am attempting uses a y-variable, can I still use the quadratic formula? Since I am not sure what route to take in solving this problem, I am hoping you can assist me.

Harley Weston lui répond.
A rule with a variable 2003-08-24
Crystal pose la question :
what is a rule with a variable
Penny Nom lui répond.
A golf tournament 2003-08-23
Dale pose la question :
I am running a golf tournament with twelve golfers. We will be playing four rounds of golf. I don't want to schedule the same two players together more than twice (if possible). How would I make a schedule that each player would play at least once with every other player? I would appreciate any help that you can give me.
Denis Hanson lui répond.
Two precalculus problems 2003-08-04
Kate pose la question :

Please help me verify the identity:
cos2x(sec2x-1)=sin2x

Also I am having trouble withdetermining whether f(x) is odd, even, or neither
f(x)=x3-x


Penny Nom lui répond.
The area of an odd shaped piece of property 2003-07-24
Barry pose la question :
What is the formula for figuring the square footage of an odd shaped piece of property?
Penny Nom lui répond.
Natural logarithms 2003-07-22
Amanda pose la question :
I'm going into my senior year of high school. I will be taking AP calculus, and my teacher gave us some homework over the summer. However, there are two things that I do not understand how to do. The first is, she wants us to be able to generate a unit circle by hand using 30, 60 and 90 degree triangles. I have used the unit circle in trigonometry, however I was never taught how to draw it. Secondly, I need to know how to do natural logarithms without a calculator. I was not taught how to do this, and the worksheet I was given only showed me how to complete them using a calculator.
Claude Tardif lui répond.
A ruler and compass construction 2003-06-29
Mandy pose la question :
I have been searching for instructions on how to do this using a compass. Can you please send instructions step by step on how to bisect a segment into any given number of segments. (example, a way that it will work for bisecting segment into 3, 4, or even 5 congruent parts)
Penny Nom lui répond.
Odd powers of sine and cosine 2003-06-25
Antonio pose la question :
Can you please tell me how to integrate a trig function involving sine and cosine? I know if the powers of both the sine and cosine are even and nonnegative, then I can make repeated use of the power-reducing formulas. But for the question I have on my hand, the powers of both sine and cosine are odd: ( sin3x + cos7x ) dx.
Harley Weston lui répond.
Rules of exponents 2003-05-05
Carl pose la question :
Hi, I am a student who would like to recall how to multiply exponents. Here is such an equation:
6.02569 X 1025 X 5.254 =?

Also, adding exponents. Don't I just add subtract the exponents separately?
Such as 523 +15-12 =??

Penny Nom lui répond.
Write sin(3x) in terms of sin(x) 2003-05-05
A student pose la question :
Write sin 2x in terms of sin x
Penny Nom lui répond.
Integrating e^x sin(x) 2003-05-03
Lech pose la question :
I am having trouble integrating the following expression by parts:

ex sin(x)

I used the integrator at http://www.integrals.com/ to find the solution,

? 1/2 ex cos(x) + 1/2 ex sin(x).

This is easy to confirm by differentiation, however I am confounded as how to arrive at the answer.


Penny Nom and Claude Tardif lui répond.
The volume of air flowing in windpipes 2003-05-02
James pose la question :
The volume of air flowing in windpipes is given by V=kpR4, where k is a constant, p is the pressure difference at each end, R is the radius. The radius will decrease with increased pressure, according to the formula: Ro - R = cp, where Ro is the windpipe radius when p=0 & c is a positive constant. R is restricted such that:
0 < 0.5*Ro < R < Ro,
find the factor by which the radius of the windpipe contracts to give maximum flow?

Penny Nom lui répond.
A regular polygon 2003-02-26
Melissa pose la question :
The measure of each interior angle of a regular polygon is eight times that of an exterior angle of the polygon. How many sides does the polygon have?
Penny Nom lui répond.
Multiples of all four of the numbers 2, 3, 4, and 5 2003-02-11
Stephanie pose la question :
How many of the first 1000 positive integers are multiples of all four of the numbers 2, 3, 4, and 5?
Penny Nom lui répond.
I have three circles... 2003-01-30
Tony pose la question :
I HAVE THREE CIRCLE THAT IS CIRCLE TOGETHER: IN CIRCLE A, THE NUMBERS ARE: 11 I KNOW IS IN CIRCLE A, BUT I HAVE THE: 5 THAT IN A AND C, I HAVE THE 2 IN THE CIRCLE C AND B AND AND A, THE CIRCLE C I KNOW THAT 10 IS IN THE CIRCLE THE 4 IN CIRCLE A: AND B: IN CIRCLE B, I KNOW NUMBER 13 IS IN CIRCLE B; BUT I HAVE THE 3 IN CIRCLE B AND C AND I HAVE THE 2 IN CIRCLE B AND C AND A ,THE 4 IN CIRCLE B AND A.
HOW DO I FIND THE SUM IN CIRCLE C AND IN B IN BOTH CIRCLE A AND B AND B AND C NOT IN CIRCLE B, AND NOT CIRCLE C.

Penny Nom lui répond.
6 digit numbers from 0,0,2,2,4,4 2003-01-23
Amanda pose la question :
How many 6 digit numbers can you make from the numbers 0,0,2,2,4,4, giving that 0 cannot come first. The number has to contain 2 twos, 2 fours, and 2 zeros.
Penny Nom and Claude Tardif lui répond.
The area of a regular n-gon 2003-01-22
Sophie pose la question :

We have been given a piece of maths coursework. A farmer has exactly 1000 metres of fencing and wants to fence off a plot of level land. she does not mind what the shape is but it must have a perimeter of 1000m. She wants to fence off the plot of land which contains the maximum area. Investigate the shape, or shapes that could be used to fence in the maximum area using exactly 1000 metres of fencing each time.

I have investigated many shapes, and I feel that a circle will have the biggest area. However we have also been asked to investigate shapes with 20 and 30 sides. My dad said that there is a formula for finding out any area of land. Do you know of this formula, if so I would be very grateful if you were to email it to me.


Penny Nom lui répond.
The least common denominator 2003-01-21
Brittan pose la question :
Hi there I need help! My name is Brittany and i am in the 6th grade.

I need help finding the least common denominator(LCD), and the book says Find the LCM of the denominators and i've done that and then it says write equivalent fractions,using the LCM as the least commonn denominator.The directions say Use the LCD to write each pair as like fractions. and the problem is 1/8 and 5/40. Could u explain how in the word u do this? Thanks a lot

Brittany

Penny Nom lui répond.
Collinear Points 2003-01-13
Gary pose la question :

Which of the 4 points are collinear when you construct the following concurrent lines or rays of a triangle?

  1. P(1), the point where the angle bisectors intersect.
  2. P(2), the point where the altitudes (or extensions) intersect (inside or outside of the triangle).
  3. P(3), the point where the medians intersect.
  4. P(4), the point where the perpendicular bisectors (or extensions) of the three sides of a triangle intersect.


This is for my 9-12 high school class in geometry.
My name is Gary


Thanks for your help.
Gary


Harley Weston and Chris Fisher lui répond.
A rectangular prism 2003-01-09
Julie pose la question :
How many faces on a rectangular prism and how many bases? Can the base also be a face and can a face also be a base?
Diane Hanson lui répond.
Integration of 1/(2+cos(x)) 2003-01-07
A student pose la question :
integral from pi to 0 of

dx/(2+cos x)

i used the substitution t=tan(x/2) and i ended up with

integral from +infinity to 0 of

2dt/(t2+3)

which looks like an inverse tan function , and i ended up with sqr(27)/2 pi , which is not the same as my calculator's answer , so i suspct i am doing some thing wrong. can some one tell me where i am going wrong please.
Penny Nom lui répond.

Calculations 2002-12-15
Mahdi pose la question :
This problem is rather general, but it usually makes a lot of problems. I almost have no difficulty in math and physics questions, but unfortunately I'm not at all accurate in simple one or two digit calculation. I even sometimes make silly mistakes in simple sums like 7+4=12 or 4+5=11! Is there any effective way to reduce these mistakes?
Walter Whiteley lui répond.
Construct a pentagon 2002-12-12
Mary pose la question :
We were wondering what the instructions/formula, step-by-step, is to make a pentagon.
Chris Fisher and Penny Nom lui répond.
Differentiating inverses 2002-11-20
Amy pose la question :
f(x)= x3+x+1, a=1 find g'(a) (g = f -1). I am having trouble finding g(a).
Penny Nom lui répond.
Why is 5 divided by 1/7 greater than 5 divided by 2/3? 2002-11-19
Elizabeth pose la question :
  1. Without performing the division, explain why 5 divided by 1/7 is a number greater than 5 divided by 2/3.
  2. Is the least common multiple of two prime numbers always their product? Why or why not?

Diane Hanson and Penny Nom lui répond.
The volume of water in a hose 2002-11-13
David pose la question :
I trying to find out how much water 500 ft of 3/4 inch hose will hold. I'd also like to know how you figured the answer.
Penny Nom lui répond.
An octagon of perimeter 1,000 meters 2002-11-05
Stephen pose la question :
my son has to work out the area of a octagon with a outside perimeter of 1,000 metres,
Penny Nom lui répond.
A triangular prism 2002-10-30
Daniel pose la question :
I was wondering what a triangular prism was.
Diane Hanson lui répond.
Baseball, basketball and football 2002-10-08
Debbie pose la question :
The school newspaper is interviewing 6th grade students to see what sports they follow regularly on TV. Of the 70 students interviewed; 40 enjoyed basketball; 40 enjoyed baseball; 40 enjoyed football; 20 enjoyed basketball and football; 22 enjoyed baseball and basketball; 27 enjoyed football and baseball; and 12 enjoyed all three sports. How many students out of the 70 interviewed didn't follow any one of the three sports?
Leeanne Boehm, Penny Nom and Walter Whiteley lui répond.
A max/min problem 2002-09-21
Evelina pose la question :
A window is the shape of a rectangle with an equilateral triangle on top. The perimeter of the window is 300 cm. Find the width that will let the maximum light to enter.
Penny Nom lui répond.
Fractional exponents 2002-09-20
Jill pose la question :
The problem is with fractional exponants:
10 1/3 mult. by 10,000 The 1/3 is an exponant of 10.

Penny Nom lui répond.
The entire earths' population would fit in the state of Texas 2002-09-18
Roz pose la question :
I have been told that the entire earths' population would fit in the state of Texas and each person would have 1/2 acre. Is this true.
Chris Fisher lui répond.
Formulating equtions 2002-09-14
Theresa pose la question :
Hello. In math class we have to formulate equations for certin questions for extra credit. We are allowed to ask anyone and anyone can do it for us and explain it to us. Could you please do that? Here are the questions
  1. Jordan has 3 times as much money as Reva. Together they have $44. How much money does each have?

  2. The Tigers played 48 games. They won twice as many as they lost. How many did they win?

  3. There are 900 students in Sewickley Academy. There are 20 more girls than boys. How many girls are there?

  4. A board 400 cm long is cut into 2 pieces. One piece is 66 cm longer than the other. Find the lenght of the shorter piece.


Penny Nom lui répond.
The quadratic formula 2002-09-11
Kathleen pose la question :
im having a problem trying to understand quadratic formulas can u please help me

i have problems like

3x2 - 4x - 11 = 0 how do i work out that problem? i have the formula written down but it doesnt seem to register for me.

Penny Nom lui répond.
The area of a triangle 2002-09-07
Phill pose la question :
How do you find the area of a equilateral and other triangles?
Penny Nom lui répond.
A schedule for a fantasy football league 2002-09-01
Steve pose la question :
i am in a ten(10) team league, split evenly into two five team divisions. we play a thirteen(13) game schedule and would like to play each team in your own division twice, and each team in the other division once. according to my rudimentary mathematics that equals 13 games. a good schedule should look somthing like this: team 1-5 = division 1 team 6-10 = division 2 team 1 should play teams 2-5 twice and teams 6-10 once.
Claude Tardif lui répond.
Distance in 3-space 2002-08-16
David pose la question :
The question is: how do I figure out the distance of one object in 3D space to another object in 3D space? I have an object at say x = 5.872, y = 2.876, and z = 7.290; and the other object is at x = 1.129, y = -8.213, and z = -11.127. I have been suggested to use the pythagorean theory on this, but since there are three variables, I don't understand how.
Penny Nom lui répond.
Two equations 2002-07-26
Derek pose la question :
1. 3x + 2y = 4
2. -7x + 2y = 24

finding x and y.


Penny Nom lui répond.
7th grade math lesson plans 2002-07-23
Peter pose la question :
Where can I get samples of 7th grade math lesson plans and curriculum for the whole year?
Leeanne Boehm lui répond.
The base 10 multipliction table 2002-07-07
A student pose la question :
These are two questions from Math for Elementary Teachers and they have me stumped.

You have two coins that are worth 30 cents. One of the coins is not a nickel. What are the two coins?

The product of the diagonals of any 2x2 matrix in the base 10 multiplication table are equal. Why?


Penny Nom lui répond.
Integrating x^x 2002-06-18
Jeremy pose la question :
I am a student at the University of Kansas and I am wondering if there is a general anti-derivative for x x (i.e. the integral of x x dx)? I've looked in a bunch of Table of Integrals and have found nothing (can you guys find it?), so I'm sort of wondering if this may be a research type question.
Claude Tardif lui répond.
A good rule of thumb when driving 2002-06-13
Lisa pose la question :
A good rule of thumb when driving is that you should be about one car length away from the car in front of you for every 10 miles per hour that you are travelling. Suppose you follow this rule perfectly (so you are exactly the correct distance away). You are waiting at a stop light with your front bumper just touching the car in front of you. The light turns green and the car in front accelerates at a constant rate "r". Calculate how you should accelerate in order to follow the rule.
Penny Nom lui répond.
Regular polyhedra 2002-06-07
Sandra pose la question :
The other day a colleague and I were talking about polyhedra. Is regular a term applied to polyhedra or just polygons? If so, then what would define a regular polyhedron? Would it mean all faces are regular or would it mean that all faces are identical and regular? That is, could a pyramid with equilateral triangles for lateral faces and a square base be considered regular or must the base also be an equilateral triangle?
Chris Fisher lui répond.
Constructing the square root of 3 2002-06-07
Allan pose la question :
I am a Math 7/8 teacher. I was wondering how you would show a student how to find the exact location of the square root of three on the number line using just a compass and a straight edge.
Penny Nom lui répond.
The area of a circle 2002-06-03
Jessica pose la question :
I am doing a maths assigment for university, which is aimed towrds primary school students(k-6). I was wondering if you could give me some information as to how I could describe to students the rule for finding the area of circle, using a circle cut up into equal sectors (like a pizza). I know it has something to do with the fact that you can make these shapes into a parallelogram, but I am a bit uncertain as to how I can express this idea clearly and articulately to students.
Penny Nom lui répond.
A spotlight shines on a wall 2002-05-25
Barb pose la question :
A spotlight on the ground shines on a wall 12m away. If a man 2m tall walks from the spotlight toward the bldg at a speed of 1.6 m/s, how fast is his shadow on the bldg decreasing when he is 4m from the bldg?
Penny Nom lui répond.
A schedule for 24 golfers 2002-05-25
John pose la question :
I am working on a schedule for 24 golfers. 6 groups of 4. I have 8 golf days (twice per week for a month).

Ideally, I would like to schedule all 24 golfers in 6 different groups of 4 on each day. Here is the catch.....no golfer in any group can be grouped togther more than once. Every group of 4 each day will have 4 new golfers that have never played together before. Is this possible?


Chris Fisher lui répond.
Conditional probability 2002-05-19
Manny pose la question :
In a certain school, it is known that 80% of the students use the internet for school projects, 60% use e-mail on a regular basis, and 90% use the internet for school projects or for e-mail on a regular basis. a student from this school is selected at random

Determine thge probability that the student used e-mail, given that the student used the internet for school projects.
ANS: how can i solve this question by useing the vin diaagram


Andrei Volodin lui répond.
What is Calculus About? 2002-05-13
A student pose la question :
I am a high school senior and have to write an essay answering the question "What is Calculus?" I need some ideas. Thanks
Walter Whiteley lui répond.
A rectangular marquee 2002-05-07
Alyaa pose la question :
a marquee with rectangular sides on a square base with a flat roof is to be constructed from 250 meters square of canvas. find the maximum volume of the marquee. i find this topic so hard
Harley Weston lui répond.
How will I use calculus in my career? 2002-05-06
Meridith pose la question :
How will I, hopefully a future secondary mathematics teacher, use calculus in my career if I'm not teaching calculus?
Walter Whiteley lui répond.
A triangular prism 2002-04-28
A student pose la question :
Need the formula for the surface area of a triangular prisim?
Penny Nom lui répond.
Triangular numbers 2002-04-26
Anika pose la question :
Can you please tell me what a triangular number is?
Penny Nom lui répond.
Arc length 2002-04-17
Vix pose la question :
Find the point on the curve r(t)=(12sint)i-(12cost)j+5tk at a distance 13pi units along the curve from the point (0,-12,0) when t=0 in the direction opposite to the direction of increasing arc length.
Harley Weston lui répond.
Related rates 2002-04-17
Molly pose la question :
A tanker spilled 30 ft cubed of chemicals into a river, causing a circular slick whose area is expanding while its thickness is decreasing. If the radius of the slick expands at the rate of 1 foot per hour, how fast is them thickness of the slick decreasing when the area is 100 feet squared?
Penny Nom lui répond.
Area of a triangle 2002-04-07
Puzzled pose la question :
What is the surface area of a triangle 13 ft x 13 ft x 18 ft?
Penny Nom lui répond.
Pairs of equations 2002-04-04
A student pose la question :

high school level
student is asking

y=4x x=-4y 
x+y=5 3x+2y=20   

y=x-1 3x-y=4 
x+y=3 2x-3y=-9   

x+5y=4 
3x+15y=-1 

. . . 

Penny Nom lui répond.
Take It! 2002-04-03
Bryan pose la question :
You are playing Take It! for $180,00 with a total stranger. There are 180 identical balls in a big vase. Each player in his turn, reaches into the vase and pulls out 1,5,or8 balls. These balls are discarded. The player who takes the last ball from the vase wins the $180,000. A flip of the coin determines that you will go first. Are you glad? How many will you take out on the first move, and how will you proceed to win the prize?
Claude Tardif lui répond.
If you conect all the vertices of a regular n-gon... 2002-04-01
Murray pose la question :
If you conect all the vertices of a regular n-gon with lines you will have (n-3)(n/2) lines inside the polygon. I want to find out how many sections these lines divide the polygon into and how many intersections they have with each other.
Claude Tardif lui répond.
Some 5 card hands 2002-03-28
A student pose la question :
From a standard deck of cards how many 5 card hands are possible consisting of a. exactly 4 hearts

b. two cards of one kind and three of another(like a full house).

Penny Nom lui répond.
Multiplying decimals 2002-03-10
Kaitlin pose la question :
I just recently had a test on multiplying decimals and I did not do really well on it. Only 5 kids out of 19 got a passing grade. My teacher says that she will give us a different test and we can take it again, but I'm still not sure of it. My teacher has taught me a lot and she had done a wonderful job of it but I'm just confused with where to put the decimals and stuff like that. I have a math practice workbook that I do for fun and they have nothing on decimals in it. I have checked on websites to try to find worksheets but I really couldn't find a good one. Can you please help me either give me some tips or clues or send me a worksheet.
Penny Nom lui répond.
A circular wading pool 2002-03-04
Patrick pose la question :
The community of melfort is planning to build a circular wading pool in the park. The pool will cover an area of 1000m2. The building committee has decided to put a 5m cement pad around it. How much additional area will the cement pad take up?
Harley Weston lui répond.
The slope of a tangent line 2002-03-04
Ridley pose la question :
Suppose a function f(x) has the line 3x+4y=2 as its tangent line at x=5. Find f'(5).
Harley Weston lui répond.
sin 2x = cos 3x 2002-02-25
Allan pose la question :
solve:

sin 2x = cos 3x

Primary question: how do you handle the cos 3x?


Paul Betts and Chris Fisher lui répond.
The substitution method 2002-02-24
Joe pose la question :
whats the answer to this question? 3x+y=11
x+2y=-3

its substitution method i am having alot of trouble figuring it out. send the answer as soon as possible. thank you


Penny Nom lui répond.
Getting to B in the shortest time 2001-12-19
Nancy pose la question :
A motorist in a desert 5 mi. from point A, which is the nearest point on a long, straight road, wishes to get to point B on the road. If the car can travel 15 mi/hr on the desert and 39 mi/hr on the road to get to B, in the shortest possible time if......

A.) B is 5 mi. from A

B.) B is 10 mi. from A

C.) B is 1 mi. from A


Penny Nom lui répond.
Simultaneous equations 2001-12-17
Matthew pose la question :
4x + y = 4
2x - 3y = 5

what is x and y


Penny Nom lui répond.
For 7 the multiplicand is 143 2001-12-12
Cory pose la question :
For 7 the multiplicand is 143

I am asked to solve this, if you could help me it would be greatly appreciated!


Penny Nom lui répond.
Normal lines 2001-12-11
Kristie pose la question :
Why are perpendicular lines called normal lines?
Chris Fisher lui répond.
Two equations in two unknowns 2001-12-04
Courtney pose la question :
y = 3x + 2
y = 4x - 5

solve for x

Penny Nom lui répond.
A lighthouse and related rates 2001-11-29
Melissa pose la question :
A lighthouse is located on a small island 3 km away from the nearest point P on a straight shoreline, and its light makes 4 revolutions per minute. How fast is the beam of light moving along the shoreline when it is 1 km from P?
Penny Nom lui répond.
Multinomial theorem 2001-11-28
Murray pose la question :
Could you please state and explain the multinomial theorem (I already know the binomial theorem etc, to give you an idea of where i am)
Harley Weston lui répond.
A health club 2001-11-25
Maria pose la question :
A health club with a membership of 650 people operates a running track and an indoor swimming pool. A survey of the membership indicates that 68% use the running track, 44% use the swimming pool, and 8% use neither. If a member is chosen at random, what is the prbability that the member uses:

a) Both the track and the pool?

b) Only the track?


Penny Nom lui répond.
L'Hopital's Rule 2001-11-25
Mike pose la question :
I'm a high school senior. I'm in ap calculus and was wondering if you know the proof of L'Hopital's Rule? Our teacher said he has never been able to find it in a math book and was unable to derive it himself.
Claude Tardif lui répond.
A tangent line 2001-11-21
A student pose la question :
write an equation of the line tangent to the graph of

ey + ln(xy) = 1 + e at (e,1)

Harley Weston lui répond.
Rewriting and evaluating formulas 2001-11-19
A conserned mother pose la question :
Due to the wide range of temperatures experienced in Canada, engineers who construct roads must allow for expansion and contraction of the road surface. The following formula is used to calculate the amount of expansion E to allow for: E= kL(T-t), where k is the constant of expansion for the road surface. L is the length of the section of highway in metres. T is the temperature of the air in degrees Celsius. t is the temperature, in degress Celsius, at which the section of the highway was constructed.
Walter Whiteley lui répond.
12 pairs playing bridge 2001-11-09
Diana pose la question :
I have 12 pairs playing bridge against one another for 12 games. I need to have each pair partnered with another pair -- but only once. I'm looking for a schedule for play for all 12 games. They should only be able to play against another team only once also. (ex: 1/2 v 3/4 then 4/2 v 3/1) Thank you for this opportunity to solve my dilemma.
Claude Tardif lui répond.
Asymptotes 2001-11-09
Frank pose la question :

given the function:

f(x) = (x2) / (x-1)

the correct answer to the limit of f(x) as x approaches infinity is:

y = x+1

all math references point to this answer and the method they all use is long division of x-1 into x2

however if one were to multiply both the numerator and denominator by 1/x and then take the limit, one gets:

y=x

how can the descrepency between the two answers be explained?


Chris Fisher and Penny Nom lui répond.
A 3 dimensional 5 pointed star 2001-11-08
Kent pose la question :
I am looking for a formula that will give me a layout for a 3 dimensional 5 pointed star. I want to form it out of sheet metal, using 5 polygons and soldering them at the apex. Can you please help me with this? I would like to be able to give the formula the height of the star from the bottom two points to the top point and also how deep the star is. Thank you very much!
Judi McDonald lui répond.
Manufactures golf balls 2001-11-06
Kevin pose la question :
A company manufactures golf balls. Golf balls are each numbered consecutively 0 to 4, and then the numbering starts over with zero again. Every 12th golf ball is yellow. Every 8th golf ball is tested for defects. The company produces 120,000 golf balls in an eight-hour shift.

A. How many yellow golf balls are tested for defects during the day? Explain your reasoning.

B. How many of these yellow balls that are tested are imprinted with the numeral 2? Explain your reasoning.


Penny Nom lui répond.
A lighthouse problem 2001-11-02
A student pose la question :
A lighthouse at apoint P is 3 miles offshore from the nearest point O of a straight beach. A store is located 5 miles down the beach from O. The lighthouse keeper can row at 4 mph and walk at 3.25 mph.

a)How far doen the beach from O should the lighthouse keeper land in order to minimize the time from the lighthouse to the store?

b)What is the minimum rowing speed the makes it faster to row all the way?

Harley Weston lui répond.
Negative times negative is positive 2001-10-26
Mary pose la question :
I have a question about adding and multiplying positive and negative numbers. When we add two negative numbers the answer is negative BUT when we multiply two negative numbers the answer is positive. I don't understand. Why?
Penny Nom lui répond.
Why do you flip the inequality sign? 2001-10-26
Sean pose la question :
I have a question about an inequality problem. Here it is: Solve and graph 5 - 3x => 17. (=> is greater than or equal). Please tell me why you flip the inequality sign when divinding by a negative number.
Penny Nom lui répond.
Concavity of f(g) 2001-10-25
Troy pose la question :
Suppose f & g are both concave upward on (-infinity,infinity). Under what condition on f will the composite function h(x)= f(g(x)) be concave upward?
Walter Whiteley lui répond.
Multiplying vectors 2001-10-22
Murray pose la question :
Could you please explain why a vector times a vector is a scalar and how to derive the formula vector a * vector b = ab cos(a,b)
Harley Weston lui répond.
Rules of exponents 2001-10-14
Carissa pose la question :
how do you work this out? Investigate the relationship between a,b,c and d if 2a*2b=4c/4d?
Leeanne Boehm lui répond.
Maximize the area 2001-10-13
Mike pose la question :

I have no clue how to do this problem. Here is what the professor gave to us:

A=LW

C=E(2L+2W) + I(PL)

Where P = # of partitions

E= cost of exterior of fence

I = cost of interior of fence

C = total cost of fence

.
.
.

Harley Weston lui répond.
4 sinx cosy = 1 2001-10-10
A student pose la question :
How would i differentiate the following example in terms of t (x and y are functions of t)

4 sinx cosy = 1


Claude Tardif lui répond.
Locusts 2001-10-08
A parent pose la question :
JOHN CONJECTURED THAT BOTH 13-YR.&17-YR. LOCUSTS CAME OUT THE SAME YEAR. ASSUME IT IS CORRECT. HOW MANY YEARS PASS BETWEEN THE YRS. WHEN BOTH 13 YR. & 17YR. LOCUSTS ARE OUT AT THE SAME TIME? EXPLAIN. NEXT, SUPPOSE THERE WERE 12YR.,14 YR.,& 16YR. LO- CUSTS,& THEY ALL COME OUT THIS YR. HOW MANY YRS. WILL IT BE BEFORE THEY ALL COME OUT TOGETHER AGAIN?
Leeanne Boehm lui répond.
The height of the lamppost 2001-10-02
Werner pose la question :
I am working on question 51,section 3.7 ,page 191 of Stewart's Single Variable Calculus. The question involves a lamp post which is casting a shadow around the eliipse whose formula is x2 + 4*y2 = 5. I have found the derivative of the elllipse both explicitly: x/4(((5-x2)/4)0.5) and implicitly : y' = - x/(4*y).
Harley Weston lui répond.
(x^2-5x-6)/(x-6) 2001-10-02
Bill pose la question :
given f(x) = (x2-5x-6)/(x-6) find f'(6).
Harley Weston lui répond.
GCD and LCM 2001-09-24
David pose la question :
Find the GCD adn LCM of the following numbers. Assume p 2 to the power p, 3 to power q, 5 to power r

and

2 to power q, 3 to power r, 7 to power p


Penny Nom lui répond.
Multiplying binomials 2001-09-08
A student pose la question :
(x+1)(x+2)=?
(2x-1)(x-4)=0
please tell me this question!!

Leeanne Boehm lui répond.
Don't worry too much about your difficulties in math... 2001-09-07
Matt pose la question :
Do you know who said,"Don't worry too much about your difficulties in math, I assure you that mine are still greater"? Thanks for your help. this for extra credit and I can't find it ANYWHERE!
Penny Nom lui répond.
Earthquake warning 2001-09-03
Dillon pose la question :
In the city of Skangai there are 6 million people.Each person has to tell 2 more people that theres going to be an earthquake. After they do they leave the city. There will be no duplications.It takes 3 minutes to tell each person. How long will it take for the whole city to know about the earthquake?
Penny Nom lui répond.
Conformable matrices 2001-08-05
Wayne pose la question :
Can someone explain the concept of conformable matrices in a way that is easy to understand ? One definition says to multiply matrices rows and columns must conform, ie, 5 x 3 matrix times a 3 x 5 matrix. In the next example, however a 8 x 1 matrix and a 8 x 3 matrix are said to be conformable!
Steve Kirkland lui répond.
Intersection of perpendicular cylindrical surfaces 2001-07-31
Charlie pose la question :
Please consider two right circular cylinders, perpendicular one to the other, and of unlike radii in a 3 dimensional Cartesian space with mutually perpendicular x,y,z axes. If one cylinder is centered on the y axis with radius ra, and the other on the z axis with radius rb, then the expression for the first surface would be x2 + z2 = ra2, y = any number. Likewise, the second cylinder's surface would be x2 + y2 = rb2, z = any number.

It is my intent to define the curve at the intersection of these two cylindrical surfaces. From sketching the conditions it appears that this intersection resembles an ellipse folded about its minor axis.


Chris Fisher lui répond.
The Mean Value Theorem 2001-07-23
Corrie pose la question :
I need to find if the mean value theorem exists. and if so, find all values c guaranteed by the theorem.

f(x) = |x2-25| on the interval [-10,0]


Harley Weston lui répond.
A calculation with 6 numbers 2001-06-16
Edwin pose la question :
I'm asked to come with, and program (in Ansi -C) an algorithm that calculates all the possible results of a calculation with 6 numbers and one result. For example: I want all calculations with the numbers 3, 3, 8, 8, 2, 9, and with a result of 786. all numbers may be used once, arithmetical operations allowed are + - / *, fractions are not allowed. The problem here is what is a fast method to do this (i.e. what's algorithm that can to this).
Claude Tardif lui répond.
Area between curves 2001-06-13
Phil pose la question :

question 1

find the area bound by the curves y = x2 + 2x + 3 and y = 2x + 4

question 2

Find the volume generated by rotating the curve x2 + y2 = 9 about the x-axis

Harley Weston lui répond.
Danging couples 2001-06-06
Danielle pose la question :
How many boy-girl dancing couples could be formed if 85 boys and 102 girls attend a school dance?
Penny Nom lui répond.
Conservation, consumption and population growth 2001-06-04
Steve pose la question :
I'm trying to quantify the relation between conservation/consumption and population growth. For instance let's consider California:

The 2000 census states that California's population grew from 29,760* in 4-1990 to 33,871 in 4-2000. I want to find r or rate of growth per year. Based on the exponential growth formula for population growth:
.
.
.


Penny Nom lui répond.
Common solution 2001-06-02
Samantha pose la question :
  1. Solve for common solution: x+y=6 2x-3y=2

  2. Solve for y in terms of x: 3x-y=4

Penny Nom lui répond.
Multiplication 2001-05-30
Lindsay pose la question :
Who came up with multipling? its so easy
Claude Tardif and Penny Nom lui répond.
Your DJ Business 2001-05-30
Linda pose la question :
Your DJ Business has 6 rap, 10 rock, 6 alternative, 8 oldies, and 5 country CD singles. How many different 10-song sets can the DJ play, if she plays 2 singles from each category? and How many different 10-song sets if she plays exactly 3 rap singles and 4 rock singles in each set?
Leeanne Boehm lui répond.
A triangular pyramid 2001-05-14
Kumar pose la question :
In a Pyramid OABC, all the three sides of the base: a, b, c are known. Also known are the angles between the three sides from the vertex O: Angles AOB, BOC, COA.

With this information, is it possible to calculate the lengths of the three sides: OA, OB & OC.
Chris Fisher lui répond.

National consumption function 2001-05-09
Brian pose la question :
If consumption is $11 billion when disposable income is 0 and the marginal propensity to consume is dC/dy = 1/(2y+4)1/2+0.3(in billions of dollars), find the national consumption function.
Harley Weston lui répond.
Maximize profit 2001-05-09
Brian pose la question :
The marginal cost for a certain product is given by MC = 6x+60 and the fixed costs are $100. The marginal revenue is given by MR = 180-2x. Find the level of production that will maximize profit and find the profit or loss at that level.
Harley Weston lui répond.
The average value of a continuous function 2001-05-08
Esther pose la question :
The average value of a continuous function y = f(x) on the interval [a,b] is given by ________________?
Harley Weston lui répond.
Area of a regular polygon 2001-05-01
Carl pose la question :
What is the formula for finding the area of a regular Hexagon and a regular Pentagon
Penny Nom lui répond.
Population growth 2001-05-01
Gina pose la question :
Suppose the population of a country increases at a steady rate of 3% a year. If the population is 50 million at a certain time, what will it be 25 years later? Define the recurrence relation that solves this problem.
Penny Nom lui répond.
A Taylor series 2001-04-27
Karan pose la question :
Given the following information of the function
  1. f''(x) = 2f(x) for every value of x

  2. f(0) = 1

  3. f(0) = 0
what is the complete Taylor series for f(x) at a = 0

Harley Weston lui répond.
Oil revenue 2001-04-21
Brian pose la question :
Suppose that t months from now an oil well will be producing crude oil at the rate of r(t), not a constant, barrels per month and that the price of crude oil will be p(t), not a constant, dollars per barrel. Assume that the oil is sold as soon as it is extracted from the ground.
  1. Find an expression for the total revenue from the oil well, R(t).

  2. A certain oil well that currently yields 400 barrels of crude oil a month will run dry in 2 years. The price of crude oil is currently $18 per barrel and is expected to rise at a constant rate of 3 cents per barrel per month. What will be the total revenue from this well? {Hint: Model the degraded production rate with the equation:
    r(t) = (A-Bt)e0.04t}

Harley Weston lui répond.
Differentiation 2001-04-17
Esther pose la question :
Could you please tell me what the first derivative is of the following:

y = 2/(2x+e2x)

Is it (1+xe2x)/(2x+e2x)2 or perhaps -4(1+e2x)/(2x+e2x)2 ? I am a little confused between the two!


Harley Weston lui répond.
Prisms 2001-04-10
A fourth grade class pose la question :
A fourth grade class is curious about the names that have been given to various solid geometric figures based on their shapes: are there names for solids based on a rhombus, a trapezoid, a parallelogram, a decagon, and/or a quadralateral?
Chris Fisher lui répond.
Integration by parts 2001-04-09
A student pose la question :
how do you integrate x tan-1x dx, i know it can be done by integration by parts maybe, but i'm not sure....
Claude Tardif and Harley Weston lui répond.
The domain of a function 2001-04-08
Mina pose la question :
Let f(x) = (2x2+3x-17)/(x+5)
What is the domain of f? What are the values of x for which f'(x) does not = 0?

Harley Weston lui répond.
The normal to a curve 2001-04-08
Varenne pose la question :
I am having SO much trouble tackling this question and don't know what the right answer is... can you help me out? The question is

Find the equation of the normal to the curve y=(x-2)2/(1-x)2 that is parallel to the line x+4y+7=0

Harley Weston lui répond.
Common tangents 2001-04-08
Anne pose la question :
I have been working on this problem for a while but I'm not sure I'm getting the right answer: Find the common tangents of 2y=x2 and 2y=-x2-16 Thanks for the help. :)
Harley Weston` lui répond.
Where do the lines y=2x-4 and y=x-1 intesect? 2001-04-06
Bryce pose la question :
solve the following problem by setting them equal to each other. Solve for x and y. Where do the lines y=2x-4 and y=x-1 intesect?
Penny Nom lui répond.
Volume and surface area of a cylinder and a triangular prism 2001-03-31
Kevin pose la question :
I was wondering how do you find the volume and surface area of a cylinder and a triangular prism.
Penny Nom lui répond.
Airflow in windpipes 2001-03-25
Ena pose la question :
The volume of air flowing in windpipes is given by V=kpR4, where k is a constant, p is the pressure difference at each end, R is the radius. The radius will decrease with increased pressure, according to the formula: Ro - R = cp, where Ro is the windpipe radius when p=0 & c is a positive constant. R is restricted such that:
0 < 0.5*Ro < R < Ro,
find the factor by which the radius of the windpipe contracts to give maximum flow?

Harley Weston lui répond.
Cube roots on a calculator 2001-03-24
Will pose la question :
Hi my name is Will.I have a question about the calculator TI-83 Plus or the use of a scientific calculator. It is about using them to turn a cubed root to a decimal. When using that for a rational and irrational number. Rational numbers is a number that terminates or repeates. A irrational number goes on and on and uses ... (like pie). Like when you type the cube root of 8 it gives you 2, and that is a rational number. The squre root of 2 is 1.41421356... that is irrational. So why is it when you put the cube root of 16 in the calculator it says 2.5198421 that is rational it terminates at the ninth digit and my float is set for the tenth? But why when you do it by hand you get the 2 times the cube root of 2 and that is irrational? Why is that?
Judi McDonald lui répond.
Systems of equations 2001-03-16
joy pose la question :
How do u solve problems using systems of equations?
~ finding x and y~

ex:

26 = 3x - 2y
42 = 4x + y


Penny Nom lui répond.
A jogger 2001-03-12
Bill pose la question :
At time t=0 a jogger is running at a velocity of 300 meters per minute. The jogger is slowing down with a negative acceleration that is directly propotional to time t. This brings the jogger to a stop in 10 minutes.

a) write an expression for the velocity of the jogger at time t.

b) what is the total distance traveled by the jogger in that 10-minute interval.


Harley Weston lui répond.
The substitution method 2001-03-05
A student pose la question :
Solve each system of equations by the substitution method. Show your work.
  1. y = 8
    7x = 1 - y

  2. y = x - 1
    4x - y = 19

Penny Nom lui répond.
Powers 2001-03-04
A student pose la question :
Hey, can you show me how you do ..

(2xy)to the 3rd power (x) to the 2nd power?


Penny Nom lui répond.
Spins of a roulette wheel 2001-03-02
Bob pose la question :
Here's a problem I'm working on myself.....If you look at six consecutive spins of a roulette wheel, how many combinations of red and black are possible? I.E. BRRBRB, BBBBBR, BBRRRB......ETC.....
Claude Tardif lui répond.
The domain of the derivative 2001-02-22
Wayne pose la question :
I know that the domain of f'(x) is a subset of the domain of f(x). Is it necessarily true that the subset will have at most one less element than the domain of the original function?
Harley Wesston lui répond.
Faces 2001-02-21
Sandy pose la question :
How many faces are there on a sphere?

What are the faces of a cone?

What is the definition of a "face" of a 3D object?


Walter Whiteley lui répond.
The area of platts 2001-02-20
Steve pose la question :
I have to calculate the areas of different "platts" of land from a map, and the only thing I have are the four side measures. There are some cases where no sides are parallel, no lengths the same, and no angles are given, and I was wondering how to calculate the area of such figures. I've attached a rough sketch of a typical platt's shape.
Walter Whiteley lui répond.
Differentiation of y = x n 2001-02-17
Jashan pose la question :
i am studying differentation at the moment i have drawn some graphs such as y=x 2. i have found the formula for the gradient of this curve, this being 2x obtained by using differentation, but i need to know the general case for the formula where y=xn in order for me to understand this topic more throughly, i would also like to know how u derived this general formula
Harley Weston lui répond.
A four digit number 2001-02-17
Dean pose la question :
Need to find a 4 digit number that can be divided by 2, 3, 4, 5, 6, 7, 8, 9, and will always have a remainder of 1 in the answer.
Penny Nom lui répond.
A quartic equation 2001-02-15
George pose la question :
Let P(x) = x4 + ax3 + bx2 + cx + d. The graph of y = P(x) is symmetric with respect to the y-axis, has a relative max. at (0,1) and has an absolute min. at (q, -3)

a) determine the values for a, b c, and d using these values, write an equation for P(x)
b) find all possible values for q.


Harley Weston lui répond.
Multiplying one and two digit numbers 2001-02-11
Marty pose la question :
When multiplying do you put the number you are multiplying by on the top or the bottom. Example if you are multiplying by 6's would you write the problem 6
x 2
or 2
x 6
Maybe it just doesn't matter. But when you get to double digits, the double digit always goes on the top?

Penny Nom lui répond.
What is wrong with these probelms? 2001-02-09
Cindy pose la question :
Hi, I am a planning on becomming a teacher and i am asked to find out what is wrong with these probelms and how i would go about showing a student what is wrong with them!!

Error in patterns:
13/35=1/5; 27/73=2/3; 16/64=1/4

4/5+2/3=6/8; 2/5+3/4=5/9; 7/8+1/3=8/11

2/3*3=6/9; 1/4*6=6/24; 4/5*2=8/10


Leeanne Boehm and Penny Nom lui répond.
Kinds of lines 2001-02-09
Robyn pose la question :
What are the different kinds/types of lines?
Leeanne Boehm lui répond.
An irregular polygon 2001-02-09
Jason pose la question :
I have a 5 sided irregular polygon I am trying to figure out the area of. There are no right angles in the polygon as far as I can tell. I do not know any angles.
Chris Fisher lui répond.
Find an exprression for f(x) 2001-02-07
A 12th grade AP Calc student pose la question :
Let f be the function defined for all x > -5 and having the following properties. Find an expression for f(x).

i) f ''(x) = 1/ (x+5)1/3 for all x in the domain of f

ii) the line tangent to the graph of f at (4,2) has an angle of inclination of 45 degress.


Harley Weston lui répond.
The "goesinta" box 2001-02-07
Bridget pose la question :
We are looking for the "proper" name of the "box" that is drawn around the dividend on a division problem:

  ___ 3)12 

It has always been a family joke to call it the "goesinta" box, but a search of several reference sources hasn't yielded an answer --

This is a question of curiosity rather than a question for homework.


Claude Tardif lui répond.
(-2b+3)(-b-1) 2001-02-06
Melissa pose la question :
Alright, lets say you are multiplying (-2b+3)(-b-1) and you are using "FOIL" first you would get : 2b2+2b-3b-3 then you add like-terms. when multiplying,if the bases are the same, add the exponents...does the same thing apply when you are adding the results of the multiplication even though it's addition?

would the answer be 2b3-3b-3 or would it be 2b2-b-3?


Penny Nom lui répond.
Two number theory questions 2001-01-30
Richie pose la question :
i have two questions and they are:
  1. the product of three consecutive prime numbers is 7429. Find the product of the smallest and largest of these three integers.

  2. the least common multiple of two relatively prime positive integers, a and b, is 144. if a (a>or=2) is as small as possible, compute the value of b.

Penny Nom lui répond.
Fractions of fractions 2001-01-30
Katie pose la question :
When you multiply two positive fractions less than 1, how does the answer compare to the size of the fraction? Why?
Penny Nom lui répond.
Building a circular silo 2001-01-24
Natasha pose la question :
We wish to build a circular silo with internal diameter 10 feet. How much concrete will we need to pour the foundation, if we only need a 1 foot wide and 1 foot deep ring on which the silo walls will sit? Assume the 4 inch thick silo wall rests on the middle of the ring.
Penny Nom lui répond.
1 + 1 = 1 2001-01-23
Stephanie pose la question :
My friend has this as a bonus question the other day and I want to figure it out. I don't know how 1+1 in any form could equal 1. Please let me know how you come about geting that.
Claude Tardif lui répond.
7x6 by doubling 2001-01-23
A student pose la question :
Can you think of a way to use doubling to multiply 6x7?
Allen Herman lui répond.
A problem with distance 2001-01-16
A student pose la question :
For what values of a is the distance between P(a, 3) and Q(5, 2a) greater than the square root of 26.
Penny Nom lui répond.
8, 4, 2, 18, 1, 9 2001-01-11
Neyra pose la question :
Place each number below in one of the blanks to create the most meaningful and realistic story possible.
Claude Tardif lui répond.
Is n^2 - 2 a multiple of n - 4? 2001-01-10
John pose la question :
Find all positive integers n so that n2 - 2 is a multiple of n - 4.
Sukanta Pati lui répond.
Multiplication of fractions 2001-01-08
Angela pose la question :
Questions:

1. represent multiplication of fractions via an area model

2. describe why, not just how, to round decimals


Penny Nom lui répond.
Height of the lamp 2000-12-31
Joey pose la question :
The figure shows a lamp located three units to the right of the y-axis and a shadow created by the elliptical region x2 + 4y2 < 5. If the point (-5,0) is on the edge of the shadow, how far above the x-axis is the lamp located?
Harley Weston lui répond.
How do you integrate secant(theta)? 2000-12-22
Robert Williamson pose la question :
How do you integrate secant(theta)?

I know the answer is ln [sec(theta) + tan(theta)] but how do you get there?


Claude tardif lui répond.
A limit using l'hopital's rule 2000-12-13
Wassim pose la question :
I need to know how to solve the:
limits of (x ( to the power lamda) -1 )/LAMDA when lamda tends to zero ( the answer is that the functional form is ln x) and I still don't know how using hopital rule leads to this answer.

Harley Weston lui répond.
Is this a right triangle? 2000-12-08
Alicia pose la question :
How would you set-up and answer a problem like these one? Triangle ABC has vertices A(-2,2), B(1,-2), and C (1,2). Use slopes to determine if the triangle is a right triangle.
Penny Nom lui répond.
Scheduling a golf vacation 2000-12-05
Michael pose la question :
I'm having a problem scheduling matches for a golf vacation. We have 12 people playing 7 rounds of golf in 7 days. We play 2 man teams vs. 2 man teams everyday.Is there a formula so that you play WITH a different partner everyday and AGAINST as many different people as possible? Thank You for any help you may be.
Penny Nom lui répond.
A non-integrable function 2000-12-03
Mark Spilker pose la question :
I have a proof that I cannot do here it goes.
Let F(x)= 1 if x is a rational number 0 if x is an irrational number Prove the function is not intregrable on the interval (0,1). Hint: Show that no matter how small the norm of the partition, the Riemann Sum for the SigmaNotation F(wi) DeltaiX is not unique.

Harley Weston lui répond.
An integration problem 2000-11-30
A student pose la question :
If a>0 and the integral from b to 0 of 1/(1+x) equals 1/2 the integral from a to 0 of 1/(1+x), express "b" in terms of "a"
Claude tardif lui répond.
The astronomers' calendar 2000-11-29
Show_Li pose la question :
The calendar that astronomers use began on Jan. 1, 4713 B.C. on that day, the julian calendar, the lunar calendar, and roman tax system calendar all coincided. This won't happen again until 3267 A.D.
(a). How many years ago was the astronomer's calendar started?
(b). Find the number of years between times that the three calendars coincided.

Penny Nom lui répond.
Order of operations 2000-11-26
Margaret Pratt pose la question :
My daughter has a math question and I am afraid I am of no help. Can you help? 2x5/2+1-5= She arrives at 8 as the answer and has been told this is incorrect. Any help you can give would be appreciated.
Penny Nom lui répond.
A complex calculation 2000-11-24
Angie pose la question :
Multiply (3-2i)2=32-2(3)(2i)+(2i)2
Penny Nom lui répond.
Comparing an integral and a sum 2000-11-21
Douglas Norberg pose la question :
A fellow teacher asked me about a problem she wanted to give to her students. It involved whether to take a million dollars or a penny doubled a number of times. I was able to determine the number must have been .01 * 230 which is about $10 million and a lot more than $1 million. To check that I was right I used a spreadsheet and did a Riemann sum.

When I finished I reasoned that I had done the task in several steps and I could have done it in 1 step. Thus I integrated .01 * 2x from 0 through 30 but the number I got was $15,490,820.0324. Why the difference?


Harley Weston lui répond.
Euler's Formula 2000-11-11
Denise Roberts pose la question :
I'm trying to design a unit (possibly involving a Webquest) on Polyhedra and I cannot find a formula I once used. The formula involved the number of vertices (V), edges (E), and faces (F) of the polyhedra.
Walter Whiteley lui répond.
Find the rule 2000-10-27
Kelsa and Linda pose la question :
Find the rule: It had 13 in a circle then 2 blank circles then 31 in a circle then two more blank circles. Could you please help us.
Penny Nom lui répond.
Concavity 2000-10-22
Alex pose la question :
the question is: on what interval is f(x)=(x2)(ex)?

ive found the 2nd derivative which is ex(x2+4x+2) and i did the quadratic to get -2-20.5 and -2+20.5, but i dont know what the interval is.
Harley Weston lui répond.

Connecting to a water line 2000-10-20
Vanja pose la question :
My question is...A house is to be connected to a new water main that runs along the line y=2/3x-1. The connection point at the house has coordinates (2,9), where the units represent metres. What lenght of plastic pipe is needed to connect to the water main at the closest point?
Penny Nom lui répond.
Dividing fractions 2000-10-18
Paula pose la question :
Why do you have to change the division sign to a multiplication sign and invert the fraction that follows the division sign in order to get the answer to a division problem when you're working with fractions?
Penny Nom lui répond.
Multiplying decimals 2000-10-16
Lily pose la question :
I need help with mutiplying decimals (example) 2.28*0.48( * = times).
Harley Weston lui répond.
Mode 2000-09-22
James Barton pose la question :
I have always been told that a mode is the "one" number that appears most in the set of numbers: ex.{1,3,4,6,3,2} the mode is 3. What if you have {1,1,3,4,5,5}is there a mode. I was taught long ago that there is no mode, Not i am having to teach there is two modes. 1 and 5. If this is the case if we have {1,1,2,2,3,3,4,4,5,5} that every number is the mode. True or false. This is being ambigiuous if we say all are the mode. Because no one number is used more than the others.
Claue Tardif and Harley Weston lui répond.
Permutations and Multiplication Principle 2000-09-22
Candice pose la question :
A forester selects 4 pink and 4 white dogwoods. The trees are to be planted in row. If a tree is distinguished by color only, in how many ways can the eight dogwoods be planted? How many of these arrangements have at least two trees of the same color side by side?
Denis Hanson and Claude Tardif lui répond.
Dividing a region in half 2000-09-21
Kerry pose la question :
There is a line through the origin that divides the region bounded by the parabola y=x-x2 and the x-axis into two regions with equal area. What is the slope of the line?
Penny Nom lui répond.
A cycloid in Cartesian form 2000-09-20
Billy pose la question :
The parametric equation of cycloid is given:
x=r(t-sint)
y=r(1-cost)

How to eliminate t?


Harley Weston lui répond.
A proof that 1=2 2000-09-19
sporky pose la question :
Why does the proof for 1=2 not work?

x = 1
x2 = 1
x = x2
1 = 2x (derivitive)
1 = 2(1)
1 = 2 ???

please tell me where the false logic is.


Walter Whiteley lui répond.
Circular Permutations 2000-09-18
J. Stuart pose la question :
I am able to understand the simple theory behind circular permutations (that is, there are (n-1)! ways to arrange n people in a circle. One question, however, is giving me some difficulty. It reads: In how many ways can four married couples be arranged around a circular table if each man must sit beside his wife? Any suggestions you have would be greatly appreaciated.
Penny Nom lui répond.
Two linear equations 2000-09-14
David Dean pose la question :
2a + 1b = 3.39 3a + 3b = 6.59
What formula do I use to find what a = ?

Harley Weston lui répond.
Derivatives, there must be an easier way 2000-09-06
Brad Goorman pose la question :
The direction read: Take the derivative of each expression.

y = {1+[x+(x2 +x3)4]5}6


Harley Weston lui répond.
Velocity of a pendulum 2000-08-28
Mekca pose la question :
A pendulum hangs from the ceiling. as the pendulum swings, its distance,d cm, form one wall of the room depends on the number of seconds,t, since it was set in motion. assume that the equation for d as a function of t is: d=80+30cos3.14/3t, t>0.

estimate the instantaneous rate of change of d at t=5 by finding the average rates for t=5 to 5.1, t=5 to 5.01, and t=5 to 5.001.


Harley Weston lui répond.
Some trigonometry 2000-08-11
Angela pose la question :
I have some PreCal questions. I am a student at the secondary level. I would be very grateful for your help.

Solve the equation for theta (0 <= theta < 2pi).

tan2(theta) = 3 I know sec2(theta) -1 = tan2(theta)

.
.
.
Harley Weston lui répond.

PreCalculus 2000-08-09
Angela pose la question :
Use absolute values to define the interval or pair of intervals on the real line.

 
<--|--- |--- ]--- |--- |--- |--- [---| ---|-->    
   18  19  20  21  22  23  24  25  26    

A car is moving at the rate of 50 miles per hour, and the diameter of its wheels is 2.5 feet.
a) Find the number of revolutions per minute that the wheels are rotating.
b) Find the angular speed of the wheels in radians per minute.


Harley Weston lui répond.
L'Hospital's Rule 2000-07-19
Dan Krymkowski pose la question :
The limit of the following as x goes to infinity is 2*y. Y is a constant.

lim 2*x*log(x/(x-y)) = 2*y


Harley Weston lui répond.
Calculator and telephones keypads phones 2000-06-27
Thomas Smith and Veronica Yates-Riley pose la question :
On a keyboard, why is the number "7" on the top of the keypad as opposed to the number "1" at the top a telephone keypad?
Penny Nom lui répond.
Divisors of 2000 2000-06-06
Amanda Semi pose la question :
  1. find the product of all the divisors of 2000
  2. dog trainer time has 100m of fencing to enclose a rectangular exercise yard. One side of the yard can include all or part of one side of his building. iff the side of his building is 30 m, determine the maximum area he can enclose

Claude Tardif lui répond.
A derivative problem 2000-06-04
Jeff Ellis pose la question :
If F(x)=(4+x)(3+2x2)2(2+3x3)3, find F'(0)
Harley Weston lui répond.
Using the inverse sine function 2000-05-31
Nelson Rothermel pose la question :
This has me completely baffled. I have to use the laws of sine or cosine to find the angles of a triangle when I have 3 sides, so I can't go 180-x-y when I have 2 angles. Now, I have a triangle with values of 3, 7, and 9. Here are the steps I used (A,B,C are angles; a,b,c are opposite sides):

angle A (16.1951 degrees): cos-1*((b2+c2-a2)/(2*b*c))
angle B (40.6011 degrees): sin-1*(b*sin(A)/a)
angle C (56.7962 degrees): sin-1*(c*sin(A)/a)

If you notice, A+B+C does not equal 180. According to the book, A and B are correct, but C is supposed to be 123.2038 degrees. Why doesn't it work???


Harley Weston lui répond.
Why does division start from the left? 2000-05-24
Salil Dave pose la question :
Addition, subtraction and multiplications start with right most digit and proceed left, but division starts from left-most digits and goes right ... why?
Harley Weston lui répond.
Angular velocity 2000-05-24
Ashley Milliman pose la question :
The crankshaft of a particular automobile engine has an angular velocity of 1,500 rpm at 30 mph. The crankshaft pulley has a diameter of 10 cm, and it's attached to an air conditioner compressor pulley with a 7 cm diameter and an alternator pulley with a 5 cm diamter

At what angular velocities do the compressor and alternator turn?

.
.
.

Harley Weston lui répond.
Calculus Research Questions 2000-05-22
William Wright pose la question :
I am a Calculus Teacher, and me and my class ran into these two problems without solutions in my manual, we got answers, but are unable to check them. If anyone gets this email and can respond to this with the solutions it be greatly appreciated.
.
.
.

Harley Weston lui répond.
Radioactive decay 2000-05-18
Catherine Sullivan pose la question :
Please help me with the following: The radioactive isotope carbon-14 is present in small quantities in all life forms, and it is constantly replenished until the organism dies, after which it decays to carbon-12 at a rate proportional to the amount of C-14 present, with a half life of 5730 years. Suppose C(t) is the amount of C-14 at time t.
  1. Find the value of the constant k in the differential equation: C'=-kC
  2. In 1988 3 teams of scientists found that the Shroud of Turin, which was reputed to be the burial cloth of Jesus, contained 91% of the amount of C-14 contained in freshly made cloth of the same material. How old is the Shroud according to the data?

Harley Weston lui répond.
Multiplying fractions 2000-05-14
A student pose la question :
75/2 x 1/100 = 3/8

Please show how to came up with the answer?
Penny Nom lui répond.

A matrix equation 2000-05-14
A student pose la question :
Right now, we are dealing with matrices and we are supposed to solve the following problem on our graphing-calculators: 2a+3b-4c+d=20
a-2b+3c-5d=-14
3a+4b-2c+3d=19
5a-b+6c+4d=-5

Penny Nom lui répond.
Trig functions 2000-05-09
Melissa pose la question :
Find all solutions in the interval (0,2pi)
2cos2x-3cosx-4=0

Paul Betts and Harley Weston lui répond.
Parallel and perpendicular lines 2000-05-08
Jimmy Lumpkins pose la question :
Need a method for solving the following problem: Find the equation of a line through point P that is parallel or perpendicular to another line.
Paul Betts lui répond.
Related Rates 2000-05-07
Derek pose la question :
How can you show that if the volume of a balloon is decreasing at a rate proportional to its surface area, the radius of the balloon is shrinking at a constant rate.
Harley Weston lui répond.
Finding a formula 2000-05-05
Erica Hildebrandt pose la question :
If a farmer has a field and his plots are laid out in the following grid where each # represents a plot:
4 5 12 13 20
3 6 11 14 19
2 7 10 15 18
1 8 9 16 17

Of course the plot numbers aren't meaningful as I have described above. In fact they may not be numbers at all. The only constants I have are the total number of rows and columns. Using the total number of rows and columns and my current position row and column, how can I write a formula that tells me column 3 row 3 = 10, column 4 row 2 = 14, etc. I can see the pattern but can't quite get the formula. I believe I will need 2 different formulas one for even and one for odd rows.
Paul Betts and Penny Nom lui répond.

An improper integral 2000-05-04
A high school senior pose la question :
Hi, I am a high school senior and I need help stugying for a final. I am stuck on one of the questions on my review sheet. Does the improper integral from 5 to infinity of (38/97)x converge or diverge? If it converges I also need to know how to find the approximate value accurate to .01 of its actual value.
Harley Weston lui répond.
Thearcius Functionius 2000-05-03
Kevin Palmer pose la question :
With the Olympics fast approaching the networks are focusing in ona new and exciting runner from Greece. Thearcius Functionius has astounded the world with his speed. He has already established new world records in the 100 meter dash and looks to improve on those times at the 2000 Summer Olympics.

Thearcius Functionius stands a full 2 meters tall and the networks plan on placing a camera on the ground at some location after the finish line(in his lane) to film the history making run. The camera is set to film him from his knees(0.5 meters up from the ground) to 0.5 meters above his head at the instant he finishes the race. This is a total distance of two meters(the distance shown by the camera's lens).
Harley Weston lui répond.

An indefinite integral 2000-05-03
Bonnie Null pose la question :
I am to find the indefinite integral of: (ex - e-x)2 dx
Claude Tardif lui répond.
Minimizing the metal in a can 2000-05-02
May Thin Zar Han pose la question :
A can is to be made to hold 1 L of oil. Find the dimensions that will minimize the cost of the metal to manufacture the can.
Harley Weston lui répond.
Two calculus problems 2000-05-01
Kaushal Shah pose la question :
How Do WE Integrate the following Functions,
  1. Integral xtanx dx
  2. How was natural base "e" discovered and why e=2.7.......

Claude Tardif lui répond.
An expected value 2000-04-24
Carl Pride pose la question :
Suppose that in a statistics class of size 23, each student has a probability of passing of 73 percent.

sample:

What is the expected number of students who will pass??


Harley Weston lui répond.
All the roots of x^6 - 64 2000-04-15
Dakota pose la question :
Find ALL zeros of P(x) = x 6 - 64
Walter Whiteley lui répond.
The area of a triangle using calculus 2000-04-15
Todd Bowie pose la question :
Hi, I am not a student but am reviewing calculus for an upcoming interview. I would like to know how to derive the area of a triangle using calculus. Thanks!
Patrick Maidorn lui répond.
2^32 + 1 2000-04-08
Un Etudiant pose la question :
232 +1 can be divided by 641 , why?
Claude Tradif lui répond.
Star-multiplication 2000-04-07
Greg Potts pose la question :
I have this question to answer and I don't know where to start. 1*9=0, 9*8=72 and 2*8=9, then 9*9 =?
Penny Nom lui répond.
y = x^x^x^x... 2000-04-05
Michael Hackman pose la question :
Find the derivative of: y = x^x^x^x... on to infinity.
Claude Tardif lui répond.
Manipulatives 2000-04-04
Jodi Stuker pose la question :
I am a student looking for a good lesson plan using manipulatives to teach multiplication to LD students. They are currently functioning on approximately 3rd grade level in math but are age 12-14.
Penny Nom lui répond.
Riemann sums 2000-03-30
Joshua D. Parham pose la question :
If n is a positive integer, then
 
     lim   (1/n)[1/(1+1/n) + 1/(1+(2/n) + ... + 1/(1+n/n)] 
n->infinity 

can be expressed as the integral from 1 to 2 of 1/x dx
Penny Nom lui répond.

Rule of 78 2000-03-22
Dan Baumgarten pose la question :
Can you explain the rule of 78 and the reverse rule of 78? Thanks.
Claude Tardif lui répond.
Functions that satisfy f' = f 2000-03-16
Kevin Palmer pose la question :
Recently my calculus teacher asked his students to try and find any functions whose derivatives where the exact same as the original function.

The only function then I have determined that statement to be accurate in is all the natural exponential functions. Ex. f(x) = ex, f'(x) = ex

If possible could you please email me all the functions that you can find in which the original function and its derivative is identical.


Claude Tardif lui répond.
Maximize 2000-03-12
Tara Doucet pose la question :
My question is Maximize Q=xy^2 (y is to the exponent 2) where x and y are positive integers such that x + y^2 ( y is to the exponent 2)=4
Harley Weston lui répond.
Simultaneous equations 2000-03-11
Laura Molck pose la question :
My name is Laura Molck and I am in Year 11 in Australia. Please help me with the following. I know that they are all simultaneous equations which I can do but I have trouble with the formulae to work the equations. Can you please help!!

1. A tent manufacturer produces 2 models, Outback and Bushwalker. From earlier sales records it is known that 20% more of the Outback model is sold than the Bushwalker. A profit of $200 is made on each Outback sold, but $350 is made on each Bushwalker. If during the next year a profit of $177,000 is planned how many of each model must be sold?


Penny Nom lui répond.
Systems of linear equations 2000-03-10
Ann Marie Devereux pose la question :
hi there!!, I guess I have a problem!!!
  • 3x+4y=10 (over)
    4x+y=9

  • 2x=5y+3 (over)
    x=3y+1


Penny Nom lui répond.
A mixture problem 2000-03-06
Rebecca Edwards pose la question :
A tank in which cholocate milk is being mixed contains a mixture of 460 liters of milk and 40 liters of chocolate syrup initially. Syrup and milk are then added to the tank at the rate of 2 liters per minute of syrup and 8 liters of milk per minute. Simultaneously the mixture is withdrawn at the rate of 10 liters per minute. Find the function giving the amount of syrup in the tank at time t.
Harley Weston lui répond.
Two calculus problems 2000-03-03
Tara Doucet pose la question :
  1. The height of a cylinder with a radius of 4 cm is increasing at rate of 2 cm per minute. Find the rate of change of the volume of the cylinder with respect to time when the height is 10 cm.

  2. A 24 cm piece of string is cut in two pieces. One piece is used to form a circle and the other to form a square. How should the string be cut so the sum of the areas is a maximum?


Harley Weston lui répond.
Triangular numbers and square numbers 2000-03-02
Emily McCallum pose la question :
Would someone find me some math activities using triangular numbers and square number. Especially acitivities that actually form the shapes. They need to be at the 5th grade or 6th grade level. But you can not find anything that can be tauht at these level, that fine. I just need to be able to teach this new subject or kind of math to my kids.
Rick Seaman and Penny Nom lui répond.
Factoring, primes, GCF and LCM 2000-02-27
Ruth Kroek pose la question :
My son is in grade seven, he has to do a Factoring Booklet the areas covered are:
  • Prime #'s
  • Composite #'s
  • Rainbow Factoring
  • Finding Multiples (consecutive multiples)
  • Finding GCF of 2-3 numbers uning Rainbow factoring
  • Finding LCM of 2 numbers using consecutive multiples
  • prime factor trees
  • prime factor ladders
  • finding GCF of 2 numbers using Prime number Method
Although his text 'Math Power' gives some information, we are at a loss ..

Penny Nom lui répond.
Slant height of a cone 2000-02-24
Jocelyn Wozney pose la question :
I need help with this problem for my high school calculus class. Any help you can give me will be greatly appreciated-I am pretty stumped. "Express the volume of a cone in terms of the slant height 'e' and the semi-vertical angle 'x' and find the value of 'x' for which the volume is a maximum if 'e' is constant.
Harley Weston lui répond.
Triple angle formula 2000-02-23
Sara pose la question :
Can one derive a triple angle formula for sine and cosine? If so, how?
Chris Fisher lui répond.
Some integration problems 2000-02-23
Tim Valentine pose la question :
I am having a great deal of difficulty with the following integrals, can you help? I think they need the use of trig substitution or integration by parts but I cannot figure out how to begin. Thanks!

The integral of 1/(2+3x2) dx.

and

The integral of x * square root of (4x+5) dx.


Harley Weston lui répond.
A moving point on the graph of y=sinx 2000-02-22
Veronica Patterson pose la question :
Find the rate of change of the distance between the origin and a moving point on the graph of y=sinx if dx/dt=2 centimeters per second.
Harley Weston lui répond.
The quotient rule 2000-02-21
Charlene Anderson pose la question :
Question: I came across a question in our book that states: Let Q(x) = N(x) / D(x) Then re-write Q(x) in a form that can utilize the Power and Product Rules. Use this rearranged form to derive the Quotient Rule.

The Quotient Rule can be derived from the Power Rule and the Product Rule.

One must also use the chain rule too, right?


Harley Weston lui répond.
Cross multiplication 2000-02-16
J E Swinton pose la question :
Why does cross multiplication work?

How come canceling work?


Penny Nom lui répond.
Least Common Multiple 2000-02-16
Kathy Heath pose la question :
The way I understand it is for 2,3,9 the least common multiple is 18. Am I right? If so, I guess the least common multiple for 3,4,7 is 105. Please let me know if I'm right so I can tell my daughter.
Denis Hanson lui répond.
Play ball 2000-02-03
Jessie pose la question :
Here's a calc question that is probably a lot easier than I am making it. If you have a legendary "baseball problem" for the related rates section of Calc I, and you are given that the runner is running from 2nd to 3rd base at a given rate, and the umpire is standing at home plate, and you are given the distance between the bases on the field, how do you find the rate of change of the angle between the third base line (from the point of the umpire) and the runner? Here is a sample prob: Runner is moving from 2nd to 3rd base at a rate of 24 feet per second. Distance between the bases is 90 feet. What is the rate of change for the angle (theta, as described previously) when the runner is 30 feet from 3rd base?
Harley Weston lui répond.
Functions 2000-01-23
Tara pose la question :
Hi my name is Tara, I have two math problems that I need help with in my calculus math class.

  1. If f(x)= x - 2 show that (x+3)f(x)-(x+2)f(x+1)+4=0

  2. Graph this function and use the graph to determine the range y=2x2 - 8x - 3

Harley Weston lui répond.
The limit of f(x)/x 2000-01-22
Laurent Jullien pose la question :
I would appreciate help to prove that a twice continuously differentiable convex function from R+ to R has the property that f(x)/x has a limit when x tends to infinity.
Claude Tardif lui répond.
Pyramids and prisms 2000-01-18
Tyler pose la question :
What's the definition of a Triangular Prism and a Triangular pyramid.
Penny Nom lui répond.
Three dimensional rectangle 2000-01-11
Dennis Murphy pose la question :
I would like to find out the name of a Three dimensional rectangle.
Harley Weston lui répond.
How many coins is one calculator? 2000-01-09
Mandy Brockmann pose la question :
Tom weighed 1 pen and his calculator with 4 coins. Then he weighed his calculator and 3 pens with 12 coins. How many coins is one calculator?
Claude Tardif lui répond.
A probability experiment 2000-01-05
Vanessa pose la question :
Duels in the town of Discretion are rarely fatal. There, each contestant comes at a random moment between 5 a.m. and 6 a.m. on the appointed day and leaves exactly 5 minutes later, honor served, unless his opponent arrives within the time interval and then they fight. What fraction of duels lead to violence?

There must be a minimum number of 100 trials and things like graphing calculator, dice, spinners, and whatever are allowed.
Harley Weston lui répond.

Why study calculus? 2000-01-05
Trlpal pose la question :
I am a high school senior enrolled in a pre-calculus class. Could you tell me what the benefits of taking calculus are and why it would be important to take the class.
Walter Whiteley and Harley Weston lui répond.
Derive the quadratic equation 2000-01-01
Thuy pose la question :
My teacher asked us this question in which i can't understand when she asked to derive the quadratic equation. What does this mean?
Penny Nom lui répond.
Climbing stairs 1999-12-22
Harman Chaudhry pose la question :
Paul, Michelle and Michael all climb up 54 steps. Paul climbs the steps one at a time i.e. 1, 2, 3, etc. Michelle Climbs the stairs 2 at a time i.e. 2, 4, 6, etc. Michael climbs the stairs 3 at a time i.e. 3, 6, 9, etc. How many steps are used by exactly two people?
Penny Nom lui répond.
Rectangular hyperbola 1999-12-15
Aarti Chand pose la question :
Why do they call a rectangular hyperbola, rectangular and where the normal hyperbola looks like a rectangle and the rectangular hyperbola looks like a sqaure?
Chris Fisher lui répond.
A decreasing ellipsoid 1999-12-15
A student instructor pose la question :
The volume of an ellipsoid whose semiaxes are of the lengths a,b,and c is 4/3 *pi*abc. Suppose semiaxes a is changing at a rate of A cm/s , the semiaxes b is changing at B cm/s and the semiaxes c is changing at C cm/s . If the volume of the ellipsoid is decreasing when a=b=c what can you say about A,B,C? Justify.
Harley Weston lui répond.
Two calculus problems 1999-12-13
Alan pose la question :
I have 2 questions that are very new to me, they were included on a quiz and the material was never covered. Our teacher never explained the purpose and detailed explanation of how to solve the problem. Could you help? Thanks.

Question 1:
A ball is falling 30 feet from a light that is 50 feet high. After 1 sec. How fast is the shadow of the ball moving towards the light post. Note that a ball moves according to the formula S=16t^2

Question 2:
How many trapezoids must one use in order for the error to be less than 10^-8 if we want to find the area under the curve Y=1/X from 1 to 2. Find the exact area, Graph the function and use the trap rule for the "N" that you found.


Harley Weston lui répond.

A calculus problem 1999-12-08
JT Wilkins pose la question :
These are the questions:

  1. Show that there exists a unique function that meets the following requirements:

    a) f is differentiable everywhere
    b) f(0)= f'(0)= 0
    c) f(x+y)= f(x)+ f(y), for all real values of x,y

  2. Consider the function F: R-->R (All Reals)

    F(x) = 0, for x irrational & 1/q, x=p/q gcd(p,q)=1 q > 0

    a)determine the values x where f is continuous, respectively discontinuous.
    b)determine the values x when f is differentiable and for each of these values compute f'(x).


Penny Nom lui répond.
Advanced Calculus 1999-12-07
Kay pose la question :
Hi, my name is Kay. Please help---these problems are driving me crazzzzy!!!! Your help would be greatly appreciated!
  1. Let a,b be contained in R, a
  2. .
    .
    .

Claude Tardif lui répond.
Systems of equations 1999-12-06
Roger Hornbaker pose la question :
I am having problem figuring out x and y solutions.
  1. 5x + y = 4
    3x - y = 4

  2. 3x + 2y = 6
    - 3x + y = 0

Penny Nom lui répond.
order 4+ determinants 1999-12-06
Joe Kron pose la question :
Why is it never shown how to calculate the value of 4x4 (or larger size) deteminants by the diagonal multiply methods that are generally shown for 2x2 and 3x3 determinants? The method I'm talking about is called Cramer's Rule??? Is this method not extensible to order 4+ and if not why not? Anyway the method always shown for order 4+ is called "reduction by minors" which is not the answer to this question.
Walter Whiteley lui répond.
Multinomial coefficients 1999-12-03
Suraj Das pose la question :
Is there a formula for the expansion of (a+b+c) to the nth power? Does it have to do with Pascal's triangle?
Penny Nom lui répond.
The chain rule 1999-12-03
Jennifer Stanley pose la question :
This problem is making me dizzy. I would greatly appreciate a little help!

Express the derivative dy/dx in terms of x.

y=u^2(u-u^4)^3 and u=1/x^2
Harley Weston lui répond.

Two calculus problems 1999-12-01
O'Sullivan pose la question :
Question #1 Assume that a snowball melts so that its volume decreases at a rate proportional to its surface area. If it takes three hours for the snowball to decrease to half its original volume, how much longer will it take for the snowball to melt completely? It's under the chain rule section of differentiation if that any help.

I've set up a ratio and tried to find the constant but am stuck.

Question #2 The figure shows a lamp located three units to the right of the y-axis and a shadow created by the elliptical region x^2 + 4y^2 < or= 5. If the point (-5,0) is on the edge of the shadow, how far above the x axis is the lamp located?

The picture shows an x and y axis with only the points -5 and 3 written on the x axis. the lamp is on the upper right quadrant shining down diagonally to the left. There's an ellipse around the origin creating the shadow. It's formula is given as x^2 + 4y^2=5.


Harley Weston lui répond.
Cos x = -1/2 1999-12-01
Pierre Boivin pose la question :
When I factor[ 2cos (square) - 5cos -3], I get (2cos + 1)(cos - 3). 2cos + 1 = 0, 2cos = -1, cos = -0.5,. Using inv cos on calculator, I get 120 degree related angle.

When I graph I get two values, between 90 and 180 degree and between 180 and 270 degrees. How do I find those two values. How do use 120 degree in relation with the x axis.
Penny Nom lui répond.

Two derivatives 1999-11-16
Gina Renicker pose la question :
The derivative of:

y=e(xlnx) and y=x2arctan(x1/2)
Harley Weston lui répond.

Estimating the population mean 1999-11-13
John Barekman pose la question :
Statitistics: Estimating the population mean when the standard deviation is known:

I am not sure which n to use in the formula for the confidence interval equation:

x +/- z*(standard deviation/sqrt(n))

If we have data of ten people, and if we have the data of ten sets of ten people each, what is the difference in the n that we use? What is the difference between the standard deviation and the standard error? Are we using the number of sampling means or just the number of samples?
Harley Weston lui répond.

Trolls and Gargoyles 1999-11-02
TexGrimm pose la question :
How can you seat 6 monsters - 3 Trolls and 3 Gargoyles- at a circular table if the trolls look alike and the gargoyles look alike? Does your formula work for 9 monsters - 4 trolls and 5 gargoyles?
Penny Nom lui répond.
Area of a circle and an inequality 1999-10-30
Adam Anderson pose la question :
I have two problems.

The first: prove that the area of a cirlce is pi times radius squared without using calculus.

The second: show that ln(x) < x - 1 for all x > 0.


Harley Weston lui répond.

Calculating a square root 1999-10-28
Jonathan pose la question :
What is the formula for calculating a square root?
Penny Nom lui répond.
Clockwise or Counterclockwise? 1999-10-27
Tim pose la question :
A particle moves around the circle x2 + y2 = 1 with an x-velocity component dx/dt = y
  1. Find dy/dt

  2. Does the particle travel clockwise or counterclockwise around the circle? Why?

Harley Weston lui répond.
Derivatives with logs 1999-10-26
Kate pose la question :
What is the derivative of 5 to the 5x-2 at x equals 0.8?
Harley Weston lui répond.
l'Hospital's Rule 1999-10-18
Yannick Gigandet pose la question :
How can I solve these two limits :
  1. lim when n approches 1 of n[a1/n -1]

  2. lim when x approches 0 of (eax - ebx) / x
Thanks for the answer!

Harley Weston lui répond.
(-5)^2, -5^2 and -(5)^2 1999-10-13
Jennifer Brown pose la question :
What is the difference between the following problems:

(-5)2, -52 and -(5)2

Our text book (Beginning Algebra, fourth edition, published by McGraw Hill, by Streeter, Huthison and Hoetzle) says the second and third problem are exactly the same. I don't see how that can be. Is there a mathematical rule that explains this?
Penny Nom lui répond.

Length of a line 1999-10-10
Dagmara Sarudi pose la question :
My question has to do with the length of a diagonal. This problem came up when I thought about the shortest distance between two points, for example walking from one point to another in my neighborhood. I can choose a zig zag route and assuming the blocks I walk are exactly the same length, it shouldn't matter what route I took, the distance I travel should still be the same when I reached my goal. If, on the other hand I could travel in a diagonal line, the distance would be shorter. But what if, in my zig zag motion, the sections get so small the route approaches a diagonal. Shouldn't it be that each separate section added together equals the value of the two original sides? Or would it suddenly equal the value of the diagonal (which, of course was shorter than the two sides added together)?
What gives?


Chris Fisher and Harley Weston lui répond.

A trig limit 1999-10-06
Yannick Gigandet pose la question :
What is the limit, as x approaches pi/3, of (1-2cosx) / sin(x-(pi/3)) ?
Penny Nom lui répond.
towers of cubes 1999-10-05
Sanker pose la question :
I need help to solve this Rules for bulding towers of cubes
rule 1 The number of cubes on the bottom layer is always one less than the number of squares on the grid
rule 2 Each new layer is made with one cube less than the layer underneath it.
  1. Investigate how many different arrangements there are of 4 cubes on top of 5 cubes on a two by three grid

  2. investigate the number of different arrangements of six cubes on top of seven cubes on a two by four grid

  3. investigate the relation between the number of arrangements of cubes and the size of the grid
    • when there are two layers of cubes
    • when there are more than two layers of cubes

Walter Whiteley lui répond.
Regular and irregular shapes 1999-10-03
Samuel Tighe pose la question :
What is the difference between a regular shape and an irregular shape? Are a rectangle and a triangle regular or irregular shapes?
Walter Whiteley lui répond.
Two limits 1999-10-02
Jennifer pose la question :
How do I find

lim (1-cosx)/(x^2) as x-> 0

and

lim (tan3x)/x as x->0
Harley Weston lui répond.

Jose and Arthur 1999-09-26
Joel Hammer pose la question :
Jose can run around the track in 40 seconds. Arthur runs with Jose but in the opposite direction. They meet every 15 second. How long will it take Arthur to make one trip around the track?
Denis Hanson lui répond.
Distance between the windows 1999-09-19
Lawrence pose la question :
An observer on level ground is at distance d from a building. The angles of elevation to the bottom of the windows on the second and third floors are a and b respectively. Find the distance h between the bottoms of the windows in terms of a b and d
Harley Weston lui répond.
2 to the x and x squared 1999-09-17
John pose la question :
For what values of x is 2 to the exponent x greater than x squared?
Harely Weston lui répond.
A Multiple Choice Test 1999-08-31
Cintra Ramnarine pose la question :
I have ten questions. There are two multiple choice answers to each question. What are the chances of answering all questions correctly.
Harley Weston lui répond.
A Trigonometry Question 1999-08-28
Diane Simms pose la question :
My question is can the following be factored. I am a teacher who needs the factors to this right away. 2 Sin2X + 2 SinX CosX - 1= 0
Harley Weston lui répond.
Invert and multiply 1999-08-28
Debbie Walter pose la question :
I have another question please, WHY do we invert and multiply when dividing fractions? I know that's what we do but WHY? What is the reasoning behind it?
Penny Nom lui répond.
Parametric Equations 1999-08-06
Nicholas Lawton pose la question :
Show that an equation of the normal to the curve with parametric equations x=ct y=c/t t not equal to 0, at the point (cp, c/p) is :

y-c/p=xp^2-cp^3
Harley Weston lui répond.

A calculus problem 1999-07-22
Nicholas Lawton pose la question :
The curve y= e^x(px^2+qx+r) is such that the tangents at x=1 and x=3 are parallel to the x-axis. the point (0,9) is on the curve. Find the values of p,q and r.
Harley Weston lui répond.
The shortest ladder 1999-06-26
Nicholas pose la question :
A vertical wall, 2.7m high, runs parallel to the wall of a house and is at a horizontal distance of 6.4m from the house. An extending ladder is placed to rest on the top B of the wall with one end C against the house and the other end, A, resting on horizontal ground. The points A, B, and C are in a vertical plane at right angles to the wall and the ladder makes an angle@, where 0<@
Harley Weston lui répond.
Scheduling Meetings 1999-06-25
Beth Carver pose la question :
We have 6 groups meeting at three different places simultaneously. The meetings take place once a month each year. Is there a way to have each group meet with each of the other groups at least twice in a year, four times in each place?
Chris Fisher and Harley Weston lui répond.
Sin 4A 1999-06-22
Ryan Cochrane pose la question :
If sinA = 4/5, and A is a first quadrant angle, find sin4A
Harley Weston lui répond.
Even and Odd Function 1999-06-17
Kent pose la question :
There is one function with the domain of all real numbers that is both even and odd. Please give me the answer to this question before I go insane.
Penny Nom lui répond.
A circle in a square 1999-05-26
Jose V Peris pose la question :
A circle is inscribed in a square. The circumference of the circle is increasing at a constant rate of 6 inches per second. As the circle expands, the square expands to maintain the condition of tangency.

find the rate at which the perimeter of the square is increasing.

find the rate of increase in the area enclosed between the circle and the square at the instant when the area of the circle is 25(pi) square inches.
Harley Weston lui répond.

Related rates 1999-05-13
Tammy pose la question :
The sides of a rectangle increase in such a way that dz/dt=1 and dx/dt=3*dy/dt. At the instant when x=4 and y=3, what is the value of dx/dt? (there is a picture of a rectangle with sides x and y, and they are connected by z, which cuts the rectangle in half)
Harley Weston lui répond.
A Polar Plot 1999-05-06
Irene pose la question :
Consider the polar equation r=2-3Cos(theta/2) In the interval [o, 4Pi], how would you find the area of one of the leaves and also the length of one of the edges of a leaf?
Harley Weston lui répond.
Roman Numerals 1999-04-29
Michelle Jenkinson pose la question :
Someone proposed this question to me and I do not know the answer, so I was wondering if you could help. How, using Roman Numeral, did people add, subtract, multiply, and divide with no zero or negative numbers?
Penny Nom lui répond.
Parallel and perpendicular lines 1999-04-23
Crystal Pilling pose la question :
My name is Crystal Pilling and I am in 9th grade algebra. We are currently studying parallel and perpendicular lines. I am having trouble with this problem: 3/4x - 5y= 16, (5,-6) I have to find a line that is perpindicular to this line on a graph.
HELP ME PLEASE!!!!!

Penny Nom lui répond.
Radius of convergence 1999-04-21
Nowl Stave pose la question :
Why is the radius of convergence of the first 6 terms of the power series expansion of x^(1/2) centered at 4 less than 6?
Harley Weston lui répond.
The average rate of change of a function 1999-04-20
Tammy pose la question :
Suppose that the average rate of change of a function f over the interval from x=3 to x=3+h is given by 5e^h-4cos(2h). what is f'(3)? I would appreciate any help with this question.
Harley Weston lui répond.
Area of an irregular shaped objects. 1999-04-07
Jesse Townsend pose la question :
How do I find the surface area of an irregulary shaped object such as someone's knee from the thigh to shin?

Thanks.
Walter Whiteley lui répond.

Where three sequences meet. 1999-03-06
Ali pose la question :
My name is Ali and I am in the 5th grade.

I have a math question:

What is the least positive integer meeting each of the following conditions:

  • Dividing by 7 gives a remainder of 4
  • Dividing by 8 gives a remainder of 5
  • Dividing by 9 gives a remainder of 6
Help!

Denis Hanson lui répond.
Divisibility by 9 1999-02-21
Razzi pose la question :
I've been having a hard time trying to solve the following problem and I was wondering if you could help me.

For any positive integer a let S(a) be the sum of its digits. Prove that a is divisible by 9 if and only if there exist a positive integer b such that S(a)=S(b)=S(a+b).
Chris Fisher and Harley Weston lui répond.

Finding a rule for a sequence 1999-02-17
Lindsey Masters pose la question :
I'm doing a maths investigation and i have a sequence which goes:-

13,16,25,32,45,56,73.

Our teacher told us we have to find a rule by looking at the differences of the terms until we find a constant. The first differences are:-

3,9,7,13,11,17.

The differences of these are:- ......

Please could you tell me how to work it out so that I could work out the rules of similar sequences.
Penny Nom lui répond.

The Board Problem 1999-02-15
Avery pose la question :
Mr. Avery has 3-foot boards and 4-foot boards. If he puts the 3-foot boards in a line, they have the same length as the 4-foot boards put in a line. Altogether he has between 16 and 25 boards. How many 3-foot boards does he have?
Jack LeSage and Penny Nom lui répond.
Grade 4 curriculum 1999-02-09
John pose la question :
My daughter is in grade 4 and her math teacher has asked her to write out the number 1 to 10,000. She is currently at 4567 and is sick of it. When I complained that this was not a usefull exercise she said that the Ministry of Education curriculum requires this and gave a photocopy of the page. It states: "read and write whole number to 10,000 in standard, expanded, and written forms (eg., 9367=9000+300+60+7 = nine thousand three hundred sixty-seven). Has the teacher interpreted this the wrong way?

Thanks John
PS. Do you have a web address for Ontario Math and Science Teachers organization?
Jack LeSage lui répond.

Lunes 1999-02-04
Kai G. Gauer pose la question :
A prof once told me that a certain type of lune is quadrable given that the diameter is an integer. She used the construction of a right isosceles triangle within a semicircle and later constructed another semicircle on the base of the first semicircle and used area subtraction to show equality to a smaller triangle with quadrable area. What happens when the original inscribed triangle is no longer isosceles? She mentioned something about other lunes also being quadrable; but not all. What are the dimensions of other such lunes? Note: I'm not certain if I still have my hercules account; please simply post on Q&Q.
Thanks!

Chris Fisher lui répond.
Modular Arithmetic 1999-02-04
Leslie Kupper pose la question :
I am trying to do a project on modular arithmetic. I was wondering if there were any websites that include a sample lesson plan on modular arithmetic for any grade level. Let me know where and how to find them. Thanks.
Harley Weston lui répond.
Cannonballs 1999-01-27
Roger King pose la question :
How many cannonballs can be stacked in a triangular pyramid?
Penny Nom lui répond.
The Quadratic Formula 1999-01-22
Eric Morgan pose la question :
Hi my name is Eric Morgan and I m in 8th Grade honors math (middle) and I'd like to know how to prove the quadratic formula -b±sqrt of b(a)(c)over 2(a)
Penny Nom lui répond.
Angles in Polygons 1999-01-21
Jen pose la question :
  • How do u find the interior angles of a pentagon when you are given 4 of the angles and you need to find the fifth?

  • If you are given the measure of each exterior angle of a regular polygon, how do you figure out how many sides the polygon has?

  • i need all information on polygons and how to find their angles!!!

Jack LeSage lui répond.
Graphing the Derivative 1999-01-18
Milena Ghebre pose la question :
This question has been nagging me for sometime now.

Is there a way of finding out the derivative of a function, just by looking at the graph of it?
Walter Whiteley lui répond.

Calculus 1999-01-16
Kaylea Rankin pose la question :
Differentiate the following.

y = 1 /(2+3/x)
Jack LeSage and Penny Nom lui répond.

Absolute value of i 1999-01-06
Wayne Bagley pose la question :
I would like to know what is the absolute value of i. I need an answer suitable for the secondary level.
Harley Weston lui répond.
Complex numbers and the quadratic formula 1998-12-25
Richard Peter pose la question :
My age is 16, and my name is Richard. My question relates to the topic complex numbers & the quadratic formula.

I would like to know how to solve quadratic equations in which the discriminant is less than 0 (i.e. we get two complex solutions to the quadratic)

3x2+2x+5 = 0

and how mathematicians like euler contributed to this field. If it would be possible I would also like to know how this type of quadratic equations can be graphed?
Harley Weston lui répond.

Duplation method of multiplication 1998-11-26
Sara Whitford pose la question :
I am wondering why 16 and multiples of 16 do not work in the ancient duplation method of multiplication used by the Egyptians. I discovered the method in the Journeys math text gr 7 level. Just curious. Am I doing something wrong??
Jack LeSage and Harley Weston lui répond.
The bricklayers formula 1998-11-24
Rachel Kaplan pose la question :
I have to do a report on the bricklayers formula N = 7LH. Can you give me any information on this.
Harley Weston lui répond.
Golf Problem - The Sequel 1998-11-13
Bob pose la question :
Sixteen golfers, seven rounds. What are optimum combinations for each golfer to play with as many different golfers as possible?

Many thanks,
Bob Payson
Denis Hanson lui répond.

Triangular Numbers 1998-10-30
Matt pose la question :
i would like to know about triangular numbers and it history i would also like to know about the history of prime numbers thank you
Chris Fisher lui répond.
Divisibility by 11 1998-10-28
Pat Duggleby pose la question :
I am an upgrading instructor at a drop-in program in Regina. One of my students is taking General Math 30 through correspondence, and we have run into some confusing instructions. The section is about divisibility rules, and we did just fine up until the rule for Divisibility by 11. The statement is as follows:
If the difference between the sum of the odd-numbered digits and the sum of the even-numbered digits, counted from right to left, is divisible by 11, then the number is divisible by 11.
.
.
.

Penny Nom lui répond.
A Number Trick 1998-10-26
Brenda Meagher pose la question :
I'm a 37 year old mother and i have returned to school for personal reasons. I was faced with this question.
Choose any number less than 10, multiply it by 8547, then take your answer and multiply it by 13. I chose the number 8.

8547
x 8
--------
68376

and

68376
x 13
---------
205128
68376
---------
888888

Any number chosen from 1 to 9 will result in the same form of answer.If the number two is used you will end up with 222,222.etc So my question is, is there another number that will results in the answers.
Harley Weston lui répond.

A Hexagon 1998-09-22
Lee Curtis pose la question :
Could someone calculate this for me?
If a hexagon is 38 feet 8 inches across, how wide would each of the six sides be?

Thank you,
Lee Curtis
Chris Fisher and Harley Weston lui répond.

Human Calculator 1998-09-14
Pam Sloan pose la question :
I recently saw a special on NBC Dateline about a man known as the human calculator. They called him Mr. Math. I want to know more about his books, seminars, and activities. Also, what is his real name?
Patrick Maidorn lui répond.
The area and the circumference of a circle. 1998-08-27
Jason Wright pose la question :
I was looking at the relationship of the area of a circle and the circumference when I realized that 2*pi*r is the derivative of pi*r^2. I was wondering if there is any connective deep dark meaning as to why this appears to be related. Thanks for any help you can give me!
Walter Whiteley lui répond.
Concurrent Lines in a Triangle 1998-08-10
Chris Woolf pose la question :
The question is Name four types of concurrent lines, rays, or segments that are associated with triangles.
Chris Fisher lui répond.
Area and Volume 1998-07-28
James Pulver pose la question :
I am going into 12th grade and am practicing for the SAT II. I have come across a problem that I cannot solve. It states that the front, side , and bottom faces of a retangular solid have areas of 24 square centimeters, 8 square centimeters, and 3 square centimeters, respectively. What is the volume of the solid. I need to know how to solve similar problems so is there a formula to go from area to volume?
Jack LeSage lui répond.
Volumes of Revolution 1998-07-24
Lorraine Wall pose la question :
I'm on the section fpr The Computation of Volumes of Solids of Revolution and the following question is giving me problems:

-Consider the region in the first quadrant bounded by the x-and y-axes, the vertical line x=3, and the curve y=1/(xsquared + 3) I can determine the volume of the solid by rotating the region about the y-axis using the shell method but I can't seem to be able to get started with the volume when rotated about the x-axis.
Harley Weston lui répond.

Calculus problems 1998-07-13
Lorraine pose la question :
I'm stuck again. Can you help?

This involves integration using the method of partial fractions

the integral of:
7x(to the 5th) - 2x(cubed) + 3 dx
--------------------------------------
x(to the fourth) - 81

Do I have to do long division to reduce the numerator to the fourth power?

the integral of:
4- 16x +21x(squared) + 6x(cubed) - 3x(fourth) dx
----------------------------------------------------
x(cubed)(x - 2)(squared)

Lorraine
Harley Weston lui répond.

A Calculus Problem 1998-06-28
Lorraine pose la question :
I'm a post-secondary student taking calculus by correspondence. I'm stuck on the following question (and similar ones) Can you help?

Evaluate the following indefinite integral:

d(theta)
----------
1 + sin (theta)

(It says to multiply both numerator and denominator by: 1 - sin(theta)

Thanks
Lorraine
Harley Weston lui répond.

A trig limit 1998-05-28
Ann pose la question :
This problem is a calculus 1 limit problem-high school level. I'm teaching myself calc over the summer and I'm already stumped.

find the limit

 lim sec^(2)[(sqrt2)(p)]-1 p-->0 --------------------- 1-sec^(2)[(sqrt3)(p)] 
I'm Ann.

Harley Weston lui répond.
Multiplying by Nine - Chismbop Style 1998-04-27
Noria Jones pose la question :
About a year ago a grade 5 teacher at my son's school taught the children how to multiply the 9 times table on their fingers quickly.

It was part of a kind of finger math kind of thing...
Patrick Maidorn lui répond.

Clock Arithmetic. 1998-03-09
Joann Dixon pose la question :
What is clock mathematics?
Patrick Maidorn lui répond.
(50^20)(20^50) 1998-02-24
fion pose la question :
50 power of 20 X 20 power of 50?

How many zero can be found in the answer and why?
Penny Nom lui répond.

A Tightrope Walker. 1998-02-19
Amy Zitron pose la question :
A tightrope is stretched 30 feet above the ground between the Jay and the Tee buildings, which are 50 feet apart. A tightrope walker, walking at a constant rate of 2 feet per second from point A to point B, is illuminated by a spotlight 70 feet above point A....
Harley Weston lui répond.
Area of a triangle. 1998-02-01
Jodi Blucher pose la question :
Is there a formula for the area of an equilateral triangle knowing the length of the sides?
Chris Fisher and Harley Weston lui répond.
The two-digit numbers from 10-99 1998-01-21
Alexis Riddle pose la question :
My name is Alexis. I'm a student in 8th grade and I'm taking Algebra I Honors. My question is: In a string of numbers, two adjacent digits are considered as a two-digit number. For instance, the string 11012 contains the numbers 10, 11, and 12. What is the number of digits in the smallest string that contains all of the two-digit numbers from 10-99? Please help and thank you for any assistance you can give me.
Penny Nom lui répond.
Greatest Common Factor. 1998-01-09
Percy Stribling pose la question :
how do you figure out what the greater common factor is?
Penny Nom lui répond.
Roman Numerals 1997-10-31
Mark Curts pose la question :
I am looking for some resources for teaching Roman Numerals. I would like to expand upon the basic concepts, by locating some addition, subtraction, multiplication or division problems written with Roman Numerals.

--
Mark Curts
Chris Fisher lui répond.

Some Calculus Problems. 1997-10-30
Roger Hung pose la question :
  1. What real number exceeds its square by the greatest possible amount?

  2. The sum of two numbers is k. show that the sum of their squares is at least 1/2 k^2.

  3. .
    .
    .

Penny Nom lui répond.
Cos(x) Cos(2x) Cos(4x)=1/8 1997-09-24
Tan Wang pose la question :
How many distinct acute angles x are there for which cosx cos2x cos4x=1/8?
Chris Fisher Harley Weston and Haragauri Gupta lui répond.
A Trigonometric Limit 1997-09-18
Brian Ray pose la question :
What is the limit, as x approaches 0, or tan^23x/x^2? (read, tan squared 3x over...)?
Harley Weston lui répond.
A Limit Problem 1997-09-16
Robert Reny pose la question :
what is the limit, as x approaches 0, of 3x/2x-[x]? [] means absolute value.
Harley Weston lui répond.
Triangular Numbers. 1997-09-08
Rachel pose la question :
Show visually that the square number 16 is the sum of two triangular numbers. Which two?
Penny Nom lui répond.
Two Questions 1997-08-28
Faye Tan pose la question :
I am a trainee teacher. I would appreciate it very much if you could help me solve the following two problems which I think are meant for students at middle grade.

1. If a hen and a half lays an egg and a half in a day and a half, how many hens are needed to lay a dozen eggs in one day?

2. There are fewer than 200 passengers on a train. If they get off in pairs, one passenger will be without a partner. If they get off in groups of 3 or 4, there will still be one passenger left by himself. However, if they get off in groups of 5, no one will be left by themselves. How many passengers are there on the train?
Penny Nom lui répond.

The Division Bracket. 1997-04-09
Judy Riley pose la question :
A fellow teacher recently asked if I remembered the exact word for a division bracket (not the symbol with dots, the horizontal line in a fraction, or a solidus). I couldn't. Can you help?
Walter Whiteley and Harley Weston lui répond.
Mathematical Induction and the Derivative 1997-03-18
Shuling Chong pose la question :
"Obtain a formula for the nth derivative of the product of two functions, and prove the formula by induction on n."

Any educated tries are appreciated.
Penny Nom lui répond.

Equivalent Expressions 1997-02-26
Karyn Jones pose la question :
Josie says that s=n/(n+1) and s/(1-s)=n are two ways to write the same formula. Describe whether or not you agree with Joise. Explain how you made your decision.
Walter Whiteley and Penny Nom lui répond.
Circular Permutations 1997-02-05
Ed Varjassy pose la question :
I have an understanding of simple problems involving circular permutations but do not understand them when they start to have restrictions. Is there any advice you can give for these more complicated permutations.
Penny Nom lui répond.
Deriving the Quadratic Formula 1997-02-04
James pose la question :
How do you derive the quadratic formula? I know what it is, but the textbook doesn't say how to derive it.
Penny Nom lui répond.
Mathematics of Schedules 1997-01-16
Byron Krull pose la question :
I was asked if there was a mathematical method to work with schedules. The problem is this. There are 24 teams playing weekly on 4 sheets at 3 different times of the day as follows...
Denis Hanson lui répond.
Sides in a Regular Polygon 1996-12-06
Rick Moss pose la question :
If you are given the measure of each interior angle (162 degrees) of a regular polygon. How many sides does the polygon have?
Penny Nom lui répond.
Priority of operations 1996-11-04
Wallace pose la question :
What is the answer to this problem? Let x=-2 and y=3. Evaluate 12x / 2y (if the old division sign is used).
Chris Fisher lui répond.
A tangent to a circle is perpendicular to the radius at the point of contact. 1996-10-22
Rita Leung pose la question :
I wonder if there is any proof for this theorem - A tangent to a circle is perpendicular to the radius at the point of contact. If there is any proof for that, can you tell me please?
Chris Fisher and Harley Weston lui répond.
Could you tell me the name for the bar in a division problem? 1996-10-21
Linda pose la question :
Could you tell me the name for the bar in a division problem. Not the line with dots on either side but the line that divides the two numbers? My name is Linda. I am asking for my niece who is in 8th grade.
Chris Fisher lui répond.
How many digits are there in 2^2520? 1996-10-07
Rita Leung pose la question :
A)The number 64 is a sq., a cube, and a sixth power because 64=8*8, 4*4*4, 2*2*2*2*2*2. Find the smallest integer greater than 1 that is a sq., a cube, a 4th, 5th, 6th, 7th, 8th, 9th and 10th power. I have found that the answer is 2 to the power 2520th.
B) How many digits are in the correct answer to part A? The answer is uncertain. It is either 758 or 759. Can you give me any ideas?

Penny Nom lui répond.
Repeating decimals 1996-09-09
Alice pose la question :
What is the line called that is placed over the decimal to show that it is infinitely repeating?
The Centralizer lui répond.
Area of an annulus 1996-04-04
Ryan Levering pose la question :
What is the area of this annulus?
Penny Nom lui répond.
(-3)x(-2) 1995-10-25
Azmat pose la question :
Why is (-3)x(-2) = 6?
Herley Weston and Ed Giesbrecht lui répond.
 
Page
1/1

 

 


Centrale des maths reçoit une aide financière de l’Université de Regina et de The Pacific Institute for the Mathematical Sciences.

CMS
.

 

accueil centre de ressources accueil Société mathématique du Canada l'Université de Regina PIMS